Exam 3 PrepU Questions (Chapters 25-31)

Ace your homework & exams now with Quizwiz!

The nurse is admitting a client with frothy pink sputum. What does the nurse suspect is the primary underlying disorder of pulmonary edema? (ch 29)

decreased left ventricular pumping

A patient at the clinic describes shortness of breath, periods of feeling "lightheaded," and feeling fatigued despite a full night's sleep. The nurse obtains vital signs and auscultates a systolic click. What does the nurse suspect from the assessment findings? (ch28 pg792)

Mitral valve prolapse

An older African American client is found to have a blood pressure of 150/90 mm Hg during a work-site health screening. What should the nurse do? (ch30 pg896) a. Consider this to be a normal finding for the client's age and race. b. Recommend the client have blood pressure rechecked in 1 year. c. Recommend the client have blood pressure rechecked within 2 weeks. d. Recommend the client see a health care provider immediately for further evaluation.

Recommend the client have blood pressure rechecked within 2 weeks.

Which is a nonmodifiable risk factor for coronary artery disease (CAD)? (ch 27 pg 752) a. Hypertension b. Obesity c. Gender d. Diabetes mellitus

c. Gender

A nurse is reviewing self-care measures for a client with peripheral vascular disease. Which statement indicates proper self-care measures? (ch30 pg881) a. "I like to soak my feet in the hot tub every day." b. "I walk only to the mailbox in my bare feet." c. "I stopped smoking and use only chewing tobacco." d. "I have my wife look at the soles of my feet each day."

"I have my wife look at the soles of my feet each day."

When teaching a client with rheumatic carditis and a history of recurrent rheumatic fever, which statement by the client indicates that teaching has been successful? (ch28 pg794)

"I may have to take prophylactic antibiotics for up to 10 years."

The nurse is admitting a client with frothy pink sputum. What does the nurse suspect is the primary underlying disorder of pulmonary edema? (ch 29)

"I will watch my urine output to be sure that the medication is not affecting my kidneys." "If I take my digoxin I should have limited episodes of shortness of breath."

The nurse is admitting a client with heart failure. What client statement indicates that fluid overload was occurring at home? (ch29 pg829)

"I'm having trouble going up the steps during the day."

A client with peripheral arterial disease asks the nurse about using a heating pad to warm the feet. The nurse's best response is which of the following? (ch30 pg853) a. "It is better to put the heating pad on your abdomen, which causes vasodilation and warmth to your feet." b. "A heating pad to your feet is fine as long as the temperature stays below 105 degrees F." c. "A heating pad to your feet is a good idea because it increases the metabolic rate." d. "It is better to soak your feet in hot water as long as the water temperature is below 110 degrees F."

"It is better to put the heating pad on your abdomen, which causes vasodilation and warmth to your feet."

The nurse completes an assessment of a client with mitral regurgitation. What statement represents the appropriate physical finding for a client with this condition? (ch28 pg793)

"The high-pitched blowing sound at the apex is indicative of a systolic murmur."

The physician prescribed a Tegapore dressing to treat a venous ulcer. What should the nurse expect that the ankle-brachial index (ABI) will be if the circulatory status is adequate? (ch30 pg847) a. 0.10 b. 0.25 c. 0.35 d. 0.50

0.50

The nurse is evaluating the types of medications prescribed for a client's hypertension. Which of the following medication classifications establishes an action on vasoconstrictive hormones in the blood stream?

ACE inhibitor

The nurse is teaching a client diagnosed with hypertension about the DASH diet. How many servings of meat, fish, and poultry should the client consume per day?

2 or fewer

The nurse is assessing the blood pressure for a patient who has hypertension and the nurse does not hear an auscultatory gap. What outcome may be documented in this circumstance?

A high diastolic or low systolic reading

A nurse suspects the presence of an abdominal aortic aneurysm. What assessment data would the nurse correlate with a diagnosis of abdominal aortic aneurysm? (Select all that apply.) (ch30 pg863) a. A pulsatile abdominal mass b. Low back pain c. Lower abdominal pain d. Decreased bowel sounds e. Diarrhea

A pulsatile abdominal mass Low back pain Lower abdominal pain

During a teaching session, a client who is having a valvuloplasty tomorrow asks the nurse about the difference between a mechanical valve replacement and a tissue valve. What answer from the nurse is correct? (Chapter 28, Page 800) A) "A mechanical valve is thought to be more durable and so requires replacement less often." B) "Mechanical valves are used for women of childbearing age." C) "A mechanical valve is less likely to generate blood clots, so long-term anticoagulation therapy is not required." D) "Mechanical valves are not always available and are very expensive."

A) "A mechanical valve is thought to be more durable and so requires replacement less often."

The nurse is educating the client about a transvenous pacemaker. What is the best statement to explain why the client will have a transvenous pacemaker? (Chapter 26, Page 740) A) "A transvenous pacemaker is used to manage transient bradydysrhythmias like those that occur during acute MIs." B) "A transvenous pacemaker is used for a ventricular tachydysrhythmias." C) "A transvenous pacemaker is used in place of a transarterial pacemaker." D) "A transvenous pacemaker is a permanent pacemaker that is asynchronous."

A) "A transvenous pacemaker is used to manage transient bradydysrhythmias like those that occur during acute MIs."

The nurse admits an adult female client with a medical diagnosis of "rule out MI." The client is very frightened and expresses surprise that a woman would have heart problems. What response by the nurse will be most appropriate? (Chapter 25, Page 678) A) "A woman's heart is smaller and has smaller arteries that become occluded more easily." B) "A woman's resting heart rate is lower than a man's." C) "It takes longer for an electrical impulse to travel from the sinoatrial node to the atrioventricular node in a woman." D) "The stroke volume from a woman's heart is lower than from a man's heart."

A) "A woman's heart is smaller and has smaller arteries that become occluded more easily."

The nurse cares for a client with diabetes who is scheduled for a cardiac catheterization. Prior to the procedure, it is most important for the nurse to ask which question? (Chapter 25, Page 702) A) "Are you allergic to shellfish?" B) "Are you having chest pain?" C) "When was the last time you ate or drank?" D) "What was your morning blood sugar reading?"

A) "Are you allergic to shellfish?"

A client with known coronary artery disease reports intermittent chest pain, usually on exertion. When teaching the client about nitroglycerin administration, which instruction should the nurse provide? (Chapter 27, Page 759) A) "Be sure to take safety precautions because nitroglycerin may cause dizziness when you stand up." B) "Replace leftover sublingual nitroglycerin tablets every 9 months to make sure your pills are fresh." C) "A burning sensation after administration indicates that the nitroglycerin tablets are potent." D) "You may take a sublingual nitroglycerin tablet every 30 minutes, if needed. You may take as many as four doses."

A) "Be sure to take safety precautions because nitroglycerin may cause dizziness when you stand up."

A client with a strong family history of coronary artery disease asks the nurse how to reduce the risk of developing the disorder. Which is the best response by the nurse? (Chapter 27, Page 752) A) "Exercise, keep your cholesterol in check, and manage your stress." B) "Ask your physician to prescribe the new reverse lipid drug." C) "Moderation is the key to everything." D) "Increase the soy in your diet."

A) "Exercise, keep your blood sugar in check, and manage your stress."

A client with high blood pressure is receiving an antihypertensive drug. When developing a client teaching plan to minimize orthostatic hypotension, which instruction should the nurse include? (Chapter 31) A) "Flex your calf muscles, avoid alcohol, and change positions slowly." B) "Rest between demanding activities, eat plenty of fruits and vegetables, and drink 6 to 8 cups of fluid daily." C) "Wear elastic stockings, change positions quickly, and hold onto a stationary object when rising." D) "Avoid drinking alcohol and straining at stool, and eat a low-protein snack at night."

A) "Flex your calf muscles, avoid alcohol, and change positions slowly."

You are doing an admission assessment on a client who is having outpatient testing done for cardiac problems. What should you ask this client during your assessment? (Chapter 25) A) "Have you had any episodes of dizziness or fainting?" B) "Have you had any episodes when you are too nauseous?" C) "Have you had any episodes of mottling in your hands?" D) "Have you had any episodes of pain radiating into your lower extremities?"

A) "Have you had any episodes of dizziness or fainting?"

The nurse is caring for a client who is being discharged after insertion of a permanent pacemaker. Which question by the client indicates a need for clarification? (Chapter 26, Page 744) A) "I should ask for a handheld device search when I go through airport security." B) "I should avoid large magnetic fields, such as an MRI machine or large motors." C) "I should avoid contact sports." D) "I'll watch the incision for swelling or redness and will report if either occurs."

A) "I should ask for a handheld device search when I go through airport security."

A client with endocarditis is being discharged home. What statement indicates effectiveness of client teaching about preventing recurrence of the infection? (Chapter 28, Page 801) A) "I will ask for antibiotics whenever I have dental work done." B) "I am going to take an aspirin a day to prevent lesions around my valve." C) "I will always be on antibiotic therapy." D) "I will start an antibiotic when I am exposed to anyone with infections."

A) "I will ask for antibiotics whenever I have dental work done."

The nurse is providing discharge teaching with a client about pacemaker surveillance. Which client statement indicates a need for further teaching? (Chapter 26, Page 739) A) "I will take acetaminophen prior to the appointment to lessen the interrogation pain." B) "If possible, I would like to use the transtelephonic method for a follow-up." C) "The surveillance frequency of the follow-up varies with each person." D) "The surveillance checks will determine how much battery life is available."

A) "I will take acetaminophen prior to the appointment to lessen the interrogation pain."

The nurse is teaching a client with heart failure about digoxin. What statements by the client indicate the teaching is effective? Select all that apply. (Chapter 29, pg. 826) A) "I will watch my urine output to be sure that the medication is not affecting my kidneys." B) "If I take my digoxin I should have limited episodes of shortness of breath." C) "The digoxin will increase my appetite, so I should weight myself daily." D) "The medication will increase my heart rate and my blood pressure." E) "Digoxin therapy requires monthly drug levels."

A) "I will watch my urine output to be sure that the medication is not affecting my kidneys." B) "If I take my digoxin I should have limited episodes of shortness of breath."

A nurse is performing discharge teaching with a client who has an implantable cardioverter defibrillator (ICD) placed. Which client statement indicates effective teaching? (Chapter 26, Page 747) A) "I'll keep a log of each time my ICD discharges." B) "I can't wait to get back to my football league." C) "I have an appointment for magnetic resonance imaging of my knee scheduled for next week." D) "I need to stay at least 10 inches away from the microwave."

A) "I'll keep a log of each time my ICD discharges."

A client needs to have a cardiac valve replacement. The nurse offers client education about the procedures involved—including the benefits and risks. Which client statement indicates the need for more education? (Chapter 28, Page 799) A) "I'm anxious because I'll need to have cardiopulmonary bypass." B) "Since the procedure is minimally invasive, there is less surgical trauma." C) "Since the procedure is minimally invasive, there is less postoperative pain than with other techniques." D) "I might lose some blood, but not likely a large quantity of it."

A) "I'm anxious because I'll need to have cardiopulmonary bypass."

The nurse is administering a stool softener to a client who experienced a myocardial infarction. The client says, "I had a heart attack; I don't have a problem with constipation." What explanation will the nurse use to answer the client's question? (Chapter 25) A) "If you strain to have a bowel movement, you can cause a drop in your heart rate that can be dangerous." B) "The heart attack sets you up for limited activity, so constipation is often a problem for clients after a heart attack." C) "Please talk this over with your healthcare provider for further information." D) "The prescribed stool softener will decrease stress with a bowel movement and protect your heart from further injury."

A) "If you strain to have a bowel movement, you can cause a drop in your heart rate that can be dangerous."

Two nursing students are reading EKG strips. One of the students asks the instructor what the P-R interval represents. The correct response should be which of the following? (Chapter 26) A) "It shows the time needed for the SA node impulse to depolarize the atria and travel through the AV node." B) "It shows the time it takes the AV node impulse to depolarize the septum and travel through the Purkinje fibers." C) "It shows the time it takes the AV node impulse to depolarize the atria and travel through the SA node." D) "It shows the time it takes the AV node impulse to depolarize the ventricles and travel through the SA node."

A) "It shows the time needed for the SA node impulse to depolarize the atria and travel through the AV node."

A nurse is preparing a client for a scheduled adenosine stress test. Which statement made by the client indicates a need for further education? (Chapter 25, Page 700) A) "My family is bringing me a cup of coffee to drink before the test." B) "The effects of this medication will wear off quickly." C) "The medication will have an effect on my heart similar to exercise." D) "I may feel some flushing or nausea with this medication."

A) "My family is bringing me a cup of coffee to drink before the test."

The nurse receives a telephone call from a client with an implanted pacemaker who reports a pulse of 68 beats per minute, but the pacemaker rate is set at 72 beats per minute. What is the nurse's best response? (Chapter 26, Page 744) A) "Please come to the clinic right away so that we may interrogate the pacemaker to see if it is malfunctioning." B) "Don't worry. The pacemaker's rate is often higher than the client's actual heart rate." C) "Try walking briskly for about 5 minutes to see if that gets your heart rate to increase." D) "This is okay as long as you are not having any symptoms."

A) "Please come to the clinic right away so that we may interrogate the pacemaker to see if it is malfunctioning."

The nursing student asks the nurse to describe the difference between sinus rhythm and sinus bradycardia on the electrocardiogram strip. What is the nurse's best reply? (Chapter 26, Page 718) A) "The only difference is the heart rate." B) "The P waves will be shaped differently." C) "The QRS complex will be smaller in sinus bradycardia." D) "The P-R interval will be prolonged in sinus bradycardia."

A) "The only difference is the heart rate."

The nurse is assessing vital signs on a client who is 3 months status post myocardial infarction (MI). While the healthcare provider is examining the client, the client's spouse approaches the nurse and states "We are too afraid he will have another heart attack, so we just don't have sex anymore." What is the nurse's best response? (Chapter 25, Page 684) A) "The physiologic demands are greatest during orgasm and are equivalent to walking 3 to 4 miles per hour on a treadmill." B) "It is usually better to just give up sex after a heart attack." C) "Having an orgasm is very strenuous and your husband must be in excellent physical shape before attempting it." D) "The medications will prevent your husband from having an erection."

A) "The physiologic demands are greatest during orgasm and are equivalent to walking 3 to 4 miles per hour on a treadmill."

A client with a second-degree atrioventricular heart block, Type II is admitted to the coronary care unit. How will the nurse explain the need to monitor the client's electrocardiogram (ECG) strip to the spouse? (Chapter 26, Page 713) A) "The small box will transmit the heart rhythm to the central monitor all the time." B) "When your spouse needs help, an alarm will go off at the desk." C) "The box is recording the heart's electrical activity, and a physician will review the tracing later." D) "The heart's electrical activity will be recorded when the heart rate exceeds 60 beats per minute."

A) "The small box will transmit the heart rhythm to the central monitor all the time."

A nurse is teaching a client about valve replacement surgery. Which statement by the client indicates an understanding of the benefit of an autograft replacement valve? (Chapter 28, Page 800) A) "The valve is made from my own heart valve, and I will not need to take any blood-thinning drugs when I am discharged." B) "The valve is made from a pig tissue, and I will not need to take any blood-thinning drugs when I am discharged." C) "The valve is from a tissue donor, and I will not need to take any blood-thinning drugs when I am discharged." D) "The valve is mechanical, and it will not deteriorate or need replacing."

A) "The valve is made from my own heart valve, and I will not need to take any blood-thinning drugs when I am discharged."

The nurse is caring for a client scheduled for a transesophageal echocardiogram with a diagnosis of atrial fibrillation. The client's spouse asks the nurse to explain the purpose of the test. What is the nurse's best response? (Chapter 26, Page 725) A) "This test will show any blood clots in the heart, and help us determine if it is safe to do a cardioversion." B) "This test will show the specific area causing the atrial fibrillation and what can be done to stop it." C) "This test will show if the client needs a cardiac catheterization." D) "This test will let the doctor know if the client is at risk for hypotension."

A) "This test will show any blood clots in the heart, and help us determine if it is safe to do a cardioversion."

The nursing student asks the nurse how to tell the difference between ventricular tachycardia and ventricular fibrillation on an electrocardiogram strip. What is the best response? (Chapter 26, Page 719) A) "Ventricular fibrillation is irregular with undulating waves and no QRS complex. Ventricular tachycardia is usually regular and fast, with wide QRS complexes." B) "The two look very much alike; it is difficult to tell the difference." C) "The QRS complex in ventricular fibrillation is always narrow, while in ventricular tachycardia the QRS is of normal width." D) "The P-R interval will be prolonged in ventricular fibrillation, while in ventricular tachycardia the P-R interval is normal."

A) "Ventricular fibrillation is irregular with undulating waves and no QRS complex. Ventricular tachycardia is usually regular and fast, with wide QRS complexes."

A monitor technician on the telemetry unit asks a charge nurse why every client whose monitor shows atrial fibrillation is receiving warfarin. Which response by the charge nurse is best? (Chapter 26) A) "Warfarin prevents clot formation in the atria of clients with atrial fibrillation." B) "Warfarin controls heart rate in the client with atrial fibrillation." C) "Warfarin prevents atrial fibrillation from progressing to a lethal arrhythmia." D) "It's just a coincidence; most clients with atrial fibrillation don't receive warfarin."

A) "Warfarin prevents clot formation in the atria of clients with atrial fibrillation."

The nurse is caring for a client who had a permanent pacemaker surgically placed and is now ready for discharge. What statement made by the client indicates the need for more education? (Chapter 26, Page 744) A) "We will be getting rid of our microwave oven so it will not affect my pacemaker." B) "I will check my pulse every day and report to the doctor if the rate is below the pacemaker setting." C) "I will call the doctor if my incision becomes swollen and red." D) "I will avoid any large magnets that may affect my pacemaker."

A) "We will be getting rid of our microwave oven so it will not affect my pacemaker."

A client with a mechanical valve replacement asks the nurse, "Why do I have to take antibiotics before getting my teeth cleaned?" What is the nurse's best response? (Chapter 28, Page 801) A) "You are at risk of developing an infection in your heart." B) "Your teeth will not bleed as much if you have antibiotics." C) "This procedure may cause your valve to malfunction." D) "Antibiotics will prevent vegetative growth on your valves."

A) "You are at risk of developing an infection in your heart."

The nurse is working on a telemetry unit. When the nurse is interpreting a client's heart rhythm, the nurse counts each large block on graph paper as how many seconds? (Chapter 26, Page 715) A) 0.2 B) 0.1 C) 0.3 D) 0.4

A) 0.2

The nurse knows that what PR interval presents a first-degree heart block? (Chapter 26, Page 732) A) 0.24 seconds B) 0.18 seconds C) 0.14 seconds D) 0.16 seconds

A) 0.24 seconds

When administering heparin anticoagulant therapy, the nurse needs to make certain that the activated partial thromboplastin time (aPTT) is within the therapeutic range of: (Chapter 30, Page 870) A) 1.5 to 2.5 times the baseline control B) 2.5 to 3.0 times the baseline control C) 3.5 times the baseline control D) 4.5 times the baseline control

A) 1.5 to 2.5 times the baseline control

A client asks the nurse how long to wait after taking nitroglycerin before experiencing pain relief. What is the best answer by the nurse? (Chapter 27) A) 5 minutes B) 15 minutes C) 30 minutes D) 60 minutes

A) 5 minutes

Following a percutaneous transluminal coronary angioplasty, a client is monitored in the postprocedure unit. The client's heparin infusion was stopped 2 hours earlier. There is no evidence of bleeding or hematoma at the insertion site, and the pressure device is removed. With regards to partial thromboplastin time (PTT), when should the nurse plan to remove the femoral sheath? (Chapter 27, Page 772) A) 50 seconds or less B) 75 seconds or less C) 100 seconds or less D) 125 seconds or less

A) 50 seconds or less

A client is admitted to the emergency department reporting chest pain and shortness of breath. The nurse notes an irregular rhythm on the bedside electrocardiograph monitor. The nurse counts 9 RR intervals on the client's 6-second rhythm tracing. The nurse correctly identifies the client's heart rate as: (Chapter 26, Page 717) A) 90 bpm B) 80 bpm C) 70 bpm D) 100 bpm

A) 90 bpm

The nurse is assessing a patient who reports feeling "light-headed." When obtaining orthostatic vital signs, what does the nurse determine is a significant finding? (Chapter 25, Page 687) A) A heart rate of more than 20 bpm above the resting rate B) An unchanged systolic pressure C) An increase of 10 mm Hg blood pressure reading D) An increase of 5 mm Hg in diastolic pressure

A) A heart rate of more than 20 bpm above the resting rate

A nurse reviews an ECG strip for a client who is admitted with symptoms of an acute MI. The nurse should recognize what classic ECG changes that occur with an MI? Select all that apply. (Chapter 28) A) Abnormal Q-waves B) Absent P-waves C) U-wave elevations D) T-wave hyperactivity and inversions E) ST-segment elevations

A) Abnormal Q-waves D) T-wave hyperactivity and inversions E) ST-segment elevations

The nurse is teaching a client about the functionality of heart muscle. What factor may decrease a client's myocardial contractility? (Chapter 25, Page 678) A) Acidosis B) Alkalosis C) Sympathetic activity D) Administration of digoxin

A) Acidosis

The nurse is caring for a ventilated client after coronary artery bypass graft surgery. What are the criterions for extubation for the client? Select all that apply. (Chapter 27, Page 775, 778) A) Adequate cough and gag reflexes B) Breathing without assistance of the ventilator C) Acceptable arterial blood gas values D) Labile vital signs E) Inability to speak

A) Adequate cough and gag reflexes B) Breathing without assistance of the ventilator C) Acceptable arterial blood gas values

A client is receiving intravenous heparin to prevent blood clots. The order is for heparin 1,200 units per hour. The pharmacy sends 25,000 units of heparin in 500 mL of D5W. At how many milliliters per hour will the nurse infuse this solution? Record your answer using a whole number. (ch 27 pg 760)

24

The nurse is working on a telemetry unit, caring for a client who develops dizziness and a second-degree heart block, Mobitz Type 1. What will be the initial nursing intervention? (Chapter 26, Page 718) A) Administer an IV bolus of atropine B) Send the client to the cardiac catheterization laboratory C) Prepare to client for cardioversion D) Review the client's medication record

A) Administer an IV bolus of atropine

The nurse is preparing to administer warfarin to a client with a mechanical valve replacement. The client's international normalized ratio is 2.7. What action will the nurse take? (Chapter 28, Page 801) A) Administer the medication as ordered B) Asses the client for abnormal bleeding C) Prepare to administer vitamin K D) Hold the medication and notify the HCP

A) Administer the medication as ordered

A client's Holter monitor strip reveals a heart rate with normal conduction but with a rate consistently above 105 beats/minute. What other conditions can cause this response in a healthy heart? (Chapter 26, Page 719) A) All options are correct B) Elevated temperature C) Shock D) Strenuous exercise

A) All options are correct

A patient is admitted to a special critical care unit for the treatment of an arterial thrombus. The nurse is aware that the preferred drug of choice for clot removal, unless contraindicated, would be: (Chapter 30, Page 871) A) Alteplase B) Reteplase C) Urokinase D) Streptokinase

A) Alteplase

A nurse is reevaluating a client receiving IV fibrinolytic therapy. Which finding requires immediate intervention by the nurse? (Chapter 27, Page 766) A) Altered level of consciousness B) Minimal oozing of blood from the IV site C) Presence of reperfusion dysrhythmias D) Chest pain 2 of 10 (on a 1-to-10 pain scale)

A) Altered level of consciousness

Which would the nurse stress as a periodic lifelong necessity for a client managing infective endocarditis? (Chapter 28, Page 812) A) Antibiotic therapy B) Exercise regimen C) Potassium replacement D) Antihypertensive medication

A) Antibiotic therapy

The client states, "My doctor says that because I am now taking this water pill, I need to eat more foods that contain potassium. Can you give me some ideas about what foods would be good for this?" What is the appropriate response by the nurse? (Chapter 25, Page 694) A) Apricots, dried peas and beans, dates B) Asparagus, blueberries, green beans C) Cranberries, apples, popcorn D) Bok choy, cooked leeks, alfalfa sprouts

A) Apricots, dried peas and beans, dates

A home health nurse is seeing an elderly female client for the first time. During the physical assessment of the client's feet, the nurse notes several circular ulcers around the tips of the toes on both feet. The bases of the ulcers are pale, and the client reports the ulcers to be very painful. From these assessment findings, the nurse suspects that the cause of the ulcers is which of the following? (Chapter 30, Page 875) A) Arterial insufficiency B) Venous insufficiency C) Neither venous nor arterial D) Trauma

A) Arterial insufficiency

A nurse is caring for a client who is exhibiting signs and symptoms characteristic of a myocardial infarction (MI). Which statement describes priorities the nurse should establish while performing the physical assessment? (Chapter 27, Page 765) A) Assess the client's level of pain and administer prescribed analgesics B) Assess the client's level of anxiety and provide emotional support C) Prepare the client for pulmonary artery catheterization D) Ensure that the client's family is kept informed of the client's status

A) Assess the client's level of pain and administer prescribed analgesics

A patient had a cardiac catheterization and is now in the recovery area. What nursing interventions should be included in the plan of care? Select all that apply. (Chapter 25) A) Assessing the peripheral pulses in the affected extremity B) Checking the insertion site for hematoma formation C) Evaluating temperature and color in the affected extremity D) Assisting the patient to the bathroom after the procedure E) Assessing vital signs every 8 hours

A) Assessing the peripheral pulses in the affected extremity B) Checking the insertion site for hematoma formation C) Evaluating temperature and color in the affected extremity

A client has recently undergone a coronary artery bypass graft (CABG). The nurse should be alert to which respiratory complication? (Chapter 27, Page 775) A) Atelectasis B) Elevated blood glucose level C) Hyperkalemia D) Urinary tract infection (UTI)

A) Atelectasis

The nurse is caring for a client diagnosed with coronary artery disease (CAD). What condition most commonly results in CAD? (Chapter 27, Page 751) A) Atherosclerosis B) Diabetes mellitus C) Myocardial infarction D) Renal failure

A) Atherosclerosis

A client is experiencing an irregular heartbeat. The client asks the nurse how a heartbeat occurs. The nurse explains the conduction system of the heart beginning with the sinoatrial node (SA node). Place the conduction sequence of the heart in order beginning with the SA node. Use all options. (Chapter 25, Page 675) A) Atrial cell stimulation B) Bundle of His C) Purkinje fibers D) AV node E) Bundle branches

A) Atrial cell stimulation D) AV node B) Bundle of His E) Bundle branches C) Purkinje fibers

The nurse cares for a client with a dysrhythmia and understands that the P wave on an electrocardiogram (ECG) represents which phase of the cardiac cycle? (Chapter 26, Page 715) A) Atrial depolarization B) Early ventricular repolarization C) Ventricular depolarization D) Ventricular repolarization

A) Atrial depolarization

Which dysrhythmia has an atrial rate between 250 and 400, with saw-toothed P waves? (Chapter 26, Page 727) A) Atrial flutter B) Atrial fibrillation C) Ventricular fibrillation D) Ventricular tachycardia

A) Atrial flutter

When starting a client on oral or I.V. diltiazem, for which potential complication should the nurse monitor? (Chapter 27) A) Atrioventricular block B) Renal failure C) Flushing D) Hypertension

A) Atrioventricular block

Which term describes the ability of the heart to initiate an electrical impulse? (Chapter 25, Page 675) A) Automaticity B) Contractility C) Conductivity D) Excitability

A) Automaticity

A client tells the nurse "my heart is skipping beats again; I'm having palpitations." After completing a physical assessment, the nurse concludes the client is experiencing occasional premature atrial complexes (PACs). The nurse should instruct the client to: (Chapter 26, Page 720) A) Avoid caffeinated beverages B) Request sublingual nitroglycerin C) Apply supplemental oxygen D) Lie down and elevate the feet

A) Avoid caffeinated beverages

A client has undergone cardiac catheterization and will be discharged today. What information should the nurse emphasize during discharge teaching? (Chapter 25, Page 705) A) Avoid heavy lifting for the next 24 hours B) Take a tub bath, rather than a shower C) New bruising at the puncture site is normal D) Bend only at the waist

A) Avoid heavy lifting for the next 24 hours

A client has undergone cardiac catheterization and will be discharged today. What information should the nurse emphasize during discharge teaching? (Chapter 25, Page 705) A) Avoid heavy lifting for the next 24 hours B) Take a tub bath, rather than a shower C) New bruising at the puncture site is normal D) Bend only at the waist

A) Avoid heavy lifting for the next 24 hours

A client has had a pacemaker implanted and the nurse will begin client education upon the client becoming alert. Which postimplantation instructions must be provided to the client with a permanent pacemaker? (Chapter 26, Page 740) A) Avoid sources of electrical interference B) Keep the arm on the side of the pacemaker higher than the head C) Delay activities such as swimming and bowling for at least 3 weeks D) Keep moving the arm on the side where the pacemaker is inserted

A) Avoid sources of electrical interference

A nurse provides morning care for a client in the intensive care unit (ICU). Suddenly, the bedside monitor shows ventricular fibrillation and the client becomes unresponsive. After calling for assistance, what action should the nurse take next? (Chapter 26, Page 731) A) Begin cardiopulmonary resuscitation B) Prepare for endotracheal intubation C) Provide electrical cardioversion D) Administer intravenous epinephrine

A) Begin cardiopulmonary resuscitation

The nurse in the intensive care unit (ICU) hears an alarm sound in the patient's room. Arriving in the room, the patient is unresponsive, without a pulse, and a flat line on the monitor. What is the first action by the nurse? (Chapter 26, Page 731) A) Begin cardiopulmonary resuscitation (CPR) B) Administer epinephrine C) Administer atropine 0.5 mg D) Defibrillate with 360 joules (monophasic defibrillator)

A) Begin cardiopulmonary resuscitation (CPR)

Which of the following is a characteristic of an arterial ulcer? (Chapter 30, Page 875) A) Border regular and well demarcated B) Brawny edema C) Ankle-brachial index (ABI) > 0.90 D) Edema may be severe

A) Border regular and well demarcated

Which is not a likely origination point for cardiac dysrhythmias? (Chapter 26, Page 713) A) Bundle of His B) Atrioventricular node C) Atria D) Ventricles

A) Bundle of His

A client with CAD thinks diltiazem (Cardizem) has been causing nausea. Diltiazem (Cardizem) is categorized as which type of drug? (Chapter 27, Page 759) A) Calcium-channel blocker B) Beta-adrenergic blocker C) Nitrate D) Diuretic

A) Calcium-channel blocker

A patient has had an implantable cardioverter defibrillator inserted. What should the nurse be sure to include in the education of this patient prior to discharge? Select all that apply. (Chapter 26, Page 742-743) A) Call for emergency assistance if feeling dizzy B) Record events that trigger a shock sensation C) Avoid magnetic fields such as metal detection booths D) The patient will have to schedule monthly chest x-rays to make sure the device is patent E) The patient may have a throbbing pain that is normal

A) Call for emergency assistance if feeling dizzy B) Record events that trigger a shock sensation C) Avoid magnetic fields such as metal detection booths

The nurse accompanies a client to an exercise stress test. The client can achieve the target heart rate, but the electrocardiogram indicates ST-segment elevation. Which procedure will the nurse prepare the client for next? (Chapter 25, Page 699) A) Cardiac catheterization B) Telemetry monitoring C) Transesophageal echocardiogram D) Pharmacologic stress test

A) Cardiac catheterization

Which complication of cardiac surgery occurs when fluid and clots accumulate in the pericardial sac, which compresses the heart, preventing blood from filling the ventricles? (Chapter 27, Page 776) A) Cardiac tamponade B) Hypothermia C) Hypertension D) Fluid overload

A) Cardiac tamponade

A nurse is caring for a client in the cardiovascular intensive care unit following a coronary artery bypass graft. Which clinical finding requires immediate intervention by the nurse? (Chapter 25, Page 776) A) Central venous pressure reading of 1 B) Pain score 5/10 C) Blood pressure 110/68 mm Hg D) Heart rate 66 bpm

A) Central venous pressure reading of 1

The nurse is caring for an elderly client with a diagnosis of hypertension, who is taking several antihypertensive medications. Which safety precaution is the nurse most likely to reinforce? (Chapter 30, Page 896) A) Changing positions slowly related to possible hypotension B) Eating extra potassium due to loss of potassium related to medications C) Being sure to keep follow-up appointments D) Walking as far as the client is able every day

A) Changing positions slowly related to possible hypotension

The nurse is performing an assessment on a patient to determine the effects of hypertension on the heart and blood vessels. What specific assessment data will assist in determining this complication? Select all that apply. (Chapter 31) A) Character of apical and peripheral pulses B) Heart rhythm C) Lung sounds D) Respiratory rate E) Heart rate

A) Character of apical and peripheral pulses B) Heart rhythm E) Heart rate

The nurse is assisting with a bronchoscopy at the bedside in a critical care unit. The client experiences a vasovagal response. What should the nurse do next? (Chapter 27, Page 773) A) Check blood pressure B) Suction the airway C) Prepare to administer intravenous fluids D) Assess pupils for reactiveness

A) Check blood pressure

A middle-aged client presents to the ED reporting severe chest discomfort. Which finding is most indicative of a possible myocardial infarction (MI)? (Chapter 27, Page 763) A) Chest discomfort not relieved by rest or nitroglycerin B) Intermittent nausea and emesis for 3 days C) Cool, clammy skin and a diaphoretic, pale appearance D) Anxiousness, restlessness, and lightheadedness

A) Chest discomfort not relieved by rest or nitroglycerin

A middle-aged client presents to the ED reporting severe chest discomfort. Which finding is most indicative of a possible myocardial infarction (MI)? (Chapter 27, Page 763) A) Chest discomfort not relieved by rest or nitroglycerin B) Intermittent nausea and emesis for 3 days C) Cool, clammy skin and a diaphoretic, pale appearance D) Anxiousness, restlessness, and lightheadedness

A) Chest discomfort not relieved by rest or nitroglycerin

A client who has been diagnosed with Prinzmetal's angina will present with which symptom? (Chapter 27, Page 757) A) Chest pain that occurs at rest and usually in the middle of the night B) Radiating chest pain that lasts 15 minutes or less C) Prolonged chest pain that accompanies exercised D) Chest pain of increased frequency, severity, and duration

A) Chest pain that occurs at rest and usually in the middle of the night

The nurse is caring for a client with a damaged tricuspid valve. The nurse knows that the tricuspid valve is held in place by which of the following? (Chapter 25, Page 675) A) Chordae tendineae B) Atrioventricular tendons C) Semilunar tendineae D) Papillary tendons

A) Chordae tendineae

Health teaching includes advising patients on ways to reduce PAD. The nurse should always emphasize that the strongest risk factor for the development of atherosclerotic lesions is: (Chapter 30, Page 851) A) Cigarette smoking B) Lack of exercise C) Obesity D) Stress

A) Cigarette smoking

Pentoxifylline (Trental) is a medication used for which of the following conditions? (Chapter 30, Page 859) A) Claudication B) Thromboemboli C) Hypertension D) Elevated triglycerides

A) Claudication

The nurse prepares to apply ECG electrodes to a male client who requires continuous cardiac monitoring. Which action should the nurse complete to optimize skin adherence and conduction of the heart's electrical current? (Chapter 25, Page 696) A) Clip the client's chest hair prior to applying the electrodes B) Apply baby powder to the client's chest prior to placing the electrodes C) Clean the client's chest with alcohol prior to application of the electrodes D) Once the electrodes are applied, change them every 72 hours

A) Clip the client's chest hair prior to applying the electrodes

What is the term for the ability of the cardiac muscle to shorten in response to an electrical impulse? (Chapter 25, Page 672) A) Contractility B) Depolarization C) Repolarization D) Diastole

A) Contractility

What is the term for the ability of the cardiac muscle to shorten in response to an electrical impulse? (Chapter 25, Page 672) A) Contractility B) Depolarization C) Repolarization D) Diastole

A) Contractility

Which of the following is a diagnostic test that involves injection of a contrast media into the venous system through a dorsal vein in the foot? (Chapter 30, Page 849) A) Contrast phlebography B) Air plethysmography C) Lymphangiography D) Lymphoscintigraphy

A) Contrast phlebography

A client comes to the emergency department (ED) complaining of precordial chest pain. In describing the pain, the client describes it as pressure with a sudden onset. What disease process would the nurse suspect in this client? (Chapter 27, Page 750) A) Coronary artery disease B) Raynaud's disease C) Cardiogenic shock D) Venous occlusive disease

A) Coronary artery disease

According to the DASH diet, how many servings of vegetables should a person consume each day?

4 or 5

After evaluating a client for hypertension, a health care provider orders atenolol, 50 mg P.O. daily. Which therapeutic effect should atenolol have in treating hypertension? (Chapter 26, Page 721) A) Decreased cardiac output and decreased systolic and diastolic blood pressure B) Decreased blood pressure with reflex tachycardia C) Increased cardiac output and increased systolic and diastolic blood pressure D) Decreased peripheral vascular resistance

A) Decreased cardiac output and decreased systolic and diastolic blood pressure

A cardiac patient with a magnesium lab result of 2.5 mEq/L would most likely evidence which of the following? (Chapter 25) A) Depressed myocardial contractility B) Atrial tachycardia C) Increased cardiac excitability. D) Ventricular arrhythmias

A) Depressed myocardial contractility

The nurse is providing education about the nutrient content of the Therapeutic Lifestyle Changes (TLC) diet to a community group. What information will the nurse provide? Select all that apply. (Chapter 28) A) Dietary fiber should be 20 to 30 grams per day B) Cholesterol should be less than 1 gram per day C) Total fat should make up only 5% of the total calories D) Protein should make up approximately 15% of total calories E) Carbohydrates should make up 50% to 60% of the total calories

A) Dietary fiber should be 20 to 30 grams per day D) Protein should make up approximately 15% of total calories E) Carbohydrates should make up 50% to 60% of the total calories

A patient has been diagnosed with congestive heart failure (CHF). The health care provider has ordered a medication to enhance contractility. The nurse would expect which medication to be prescribed for the patient? (Chapter 25) A) Digoxin B) Clopidrogrel C) Enoxaparin D) Heparin

A) Digoxin

The nurse is caring for a geriatric client. The client is ordered Lanoxin (digoxin) tablets 0.125mg daily for a cardiac dysrhythmia. Which of the following assessment considerations is essential when caring for this client? (Chapter 25, Page 678) A) Digoxin level B) Cardiac output C) Activity level D) Dyspnea

A) Digoxin level

A nurse evaluates a client with a temporary pacemaker. The client's ECG tracing shows each P wave followed by the pacing spike. What is the nurse's best response? (Chapter 26, Page 741) A) Document the findings and continue to monitor the client B) Reposition the extremity and turn the client to left side C) Obtain a 12-lead ECG and a portable chest x-ray D) Check the security of all connections and increase the milliamperage

A) Document the findings and continue to monitor the client

A patient who had a colon resection 3 days ago is complaining of discomfort in the left calf. How should the nurse assess Homan's sign to determine if the patient may have a thrombus formation in the leg? (Chapter 30, Page 870) A) Dorsiflex the foot while the leg is elevated to check for calf pain B) Elevate the patient's legs for 20 minutes and then lower them slowly while checking for areas of inadequate blood return C) Extend the leg, plantar flex the foot, and check for the patency of the dorsalis pedis pulse D) Lower the patient's legs and massage the calf muscles to note any areas of tenderness

A) Dorsiflex the foot while the leg is elevated to check for calf pain

A client has been living with an internal, fixed-rate pacemaker. When checking the client's readings on a cardiac monitor the nurse notices an absence of spikes. What should the nurse do? (Chapter 26, Page 743) A) Double-check the monitoring equipment B) Do nothing; there is no cause for alarm C) Suggest the need for a new beta-blocker to the doctor D) Measure the client's blood pressure

A) Double-check the monitoring equipment

The nurse is preparing a client for upcoming electrophysiology (EP) studies and possible ablation for treatment of atrial tachycardia. What information will the nurse include in the teaching? (Chapter 26, Page 739) A) During the procedure, the dysrhythmia will be reproduced under controlled conditions B) The procedure will occur in the operating room under general anesthesia C) The procedure takes less time than a cardiac catheterization D) After the procedure, the dysrhythmia will not recur

A) During the procedure, the dysrhythmia will be reproduced under controlled conditions

The client with a diagnosis of heart failure reports frequently awakening during the night with the need to urinate. What explanation will the nurse offer to explain the urination? (Chapter 25, Page 683) A) Edema is collected in dependent extremities during the day; at night when the client lays down, it is reabsorbed into the circulation and excreted by the kidneys B) When the client is in the recumbent position, more pressure is put on the bladder, with the result of increased need to urinate C) The blood pressure is lower when the client is recumbent, which causes the kidneys to work harder; therefore, more urine is produced D) Fluid that is held in the lungs during the day becomes part of the circulation at night, causing the kidneys to produce an increased amount of urine

A) Edema is collected in dependent extremities during the day; at night when the client lays down, it is reabsorbed into the circulation and excreted by the kidneys

A client with an atrial dysrhythmia has come to the clinic for a follow-up appointment and to talk with the health care provider about options to stop this dysrhythmia. What procedure could be used to treat this client? (Chapter 26, Page 725) A) Elective electrical cardioversion B) Chemical cardioversion C) Mace procedure D) Elective electrical defibrillation

A) Elective electrical cardioversion

A client reports light-headedness, chest pain, and shortness of breath. They physician orders tests to ascertain what is causing the client's problems. Which test is used to identify cardiac rhythms? (Chapter 26, Page 713) A) Electrocardiogram B) Electroencephalogram C) Echocardiogram D) Electrocautery

A) Electrocardiogram

A client reporting heart palpitations is diagnosed with atrial fibrillation caused by mitral valve prolapse. To relieve the symptoms, the nurse should teach the client which dietary intervention? (Chapter 28, Page 793) A) Eliminate caffeine and alcohol B) Decrease the amount of sodium and saturated fat C) Eliminate dairy products and carbonated beverages D) Decrease the amount of acidic beverages and fruits

A) Eliminate caffeine and alcohol

Within the heart, several structures and several layers all play a part in protecting the heart muscle and maintaining cardiac function. The inner layer of the heart is composed of a thin, smooth layer of cells, the folds of which form heart valves. What is the name of this layer of cardiac tissue? (Chapter 25, Page 673) A) Endocardium B) Myocardium C) Pericardium D) Epicardium

A) Endocardium

The nurse assesses a client returning from the post anesthesia unit with a new onset of sinus tachycardia with a heart rate of 138 beats per minute and a blood pressure of 128/80mmHg after elevating the head of the bed. What intervention does the nurse consider? (Chapter 26) A) Evaluating laboratory values B) Removing anti-embolism stockings C) Assessing blood glucose level D) Decreasing intravenous fluids

A) Evaluating laboratory values

The nurse prepares to auscultate heart sounds. What nursing intervention will be most effective to assist with this procedure? (Chapter 25, Page 690) A) Explain to the client that the nurse will be listening to different areas of the chest and may listen for a long time, but that does not mean that anything abnormal is heard B) Ask the client to sit on the edge of the bed and hold breath while the nurse listens C) Insist that the family members leave the room if they must speak to each other while the nurse is auscultating heart sounds D) Ask the client to take deep breaths through the mouth while the nurse auscultates heart sounds

A) Explain to the client that the nurse will be listening to different areas of the chest and may listen for a long time, but that does not mean that anything abnormal is heard

A client presents to the emergency room with characteristics of atherosclerosis. What characteristics would the client display? (Chapter 27, Page 751) A) Fatty deposits in the lumen of arteries B) Cholesterol plugs in the lumen of veins C) Blood clots in the arteries D) Emboli in the veins

A) Fatty deposits in the lumen of arteries

The nurse suspects a client has developed pericarditis after a week of cold-like symptoms. Which of the client's signs and symptoms indicate pericarditis? (Chapter 28, Page 814) A) Fever, chest discomfort, and elevated erythrocyte sedimentation rate (ESR) B) Low urine output secondary to left ventricular dysfunction C) Lethargy, anorexia, and heart failure D) Pitting edema, chest discomfort, and nonspecific ST-segment elevation

A) Fever, chest discomfort, and elevated erythrocyte sedimentation rate (ESR)

Which class of medication lyses and dissolves thrombi? (Chapter 30, Page 871) A) Fibrinolytic B) Anticoagulant C) Platelet inhibitors D) Factor XA inhibitors

A) Fibrinolytic

The nurse is caring for a client who has premature ventricular contractions. What sign or symptom is observed in this client? (Chapter 26, Page 729) A) Fluttering B) Nausea C) Hypotension D) Fever

A) Fluttering

The nurse is completing a cardiac assessment. Upon auscultation, the nurse hears a grating sound using the diaphragm of the stethoscope. How will the nurse best document this finding? (Chapter 25, Page 691) A) Friction rub B) Murmur C) Snap D) Click

A) Friction rub

The nurse is administering propranolol to a client on a telemetry unit. What will the nurse monitor the client for? (Chapter 26, Page 729) A) Heart block B) Tachycardia C) Bleeding D) Change in level of consciousness

A) Heart block

The nurse cares for a client in the emergency department who has a B-type natriuretic peptide (BNP) level of 115 pg/mL. The nurse recognizes that this finding is most indicative of which condition? (Chapter 25, Page 695) A) Heart failure B) Ventricular hypertrophy C) Pulmonary edema D) Myocardial infarction

A) Heart failure

A nurse is monitoring the vital signs and blood results of a client who is receiving anticoagulation therapy. What does nurse identify as a major indication of concern? (Chapter 27, Page 760) A) Hematocrit of 30% B) Hemoglobin of 16 g/dL C) Heart rate of 87 bpm D) Blood pressure of 129/72 mm Hg

A) Hematocrit of 30%

The nurse is reviewing the laboratory results for a client diagnosed with coronary artery disease (CAD). The client's low-density lipoprotein (LDL) level is 115 mg/dL. The nurse interprets this value as: (Chapter 27, Page 753) A) High B) Low C) Within normal limits D) Critically high

A) High

A client's lipid profile reveals an LDL level of 122 mg/dL. This is considered a: (Chapter 27, Page 752) A) High LDL level B) Low LDL level C) Normal LDL level D) Fasting LDL level

A) High LDL level

The nurse suspects a diagnosis of mitral valve regurgitation when what type of murmur is heard on auscultation? (Chapter 28, Page 793) A) High-pitched blowing sound at the apex B) Diastolic murmur at the left sternal border C) Mitral click D) Low-pitched diastolic murmur at the apex

A) High-pitched blowing sound at the apex

A client reports chest pain. Which questions related to the client's history are most important to ask? Select all that apply. (Chapter 25) A) How would you describe your symptoms? B) How did your mother die? C) Do you have any children? D) Are you allergic to any medications or foods?

A) How would you describe your symptoms? B) How did your mother die? D) Are you allergic to any medications or foods?

When the postcardiac surgery client demonstrates restlessness, nausea, weakness, and peaked T waves, the nurse reviews the client's serum electrolytes, anticipating which abnormality? (Chapter 27) A) Hyperkalemia B) Hyponatremia C) Hypomagnesemia D) Hypercalcemia

A) Hyperkalemia

Understanding atherosclerosis, the nurse identifies which of the following to be both a risk factor for the development of the disorder and an outcome? (Chapter 27, Page 751) A) Hypertension B) Hyperlipidemia C) Obesity D) Glucose intolerance

A) Hypertension

Which describes a situation in which blood pressure is severely elevated and there is evidence of actual or probable target organ damage? (Chapter 30, Page 897) A) Hypertensive emergency B) Hypertensive urgency C) Primary hypertension D) Secondary hypertension

A) Hypertensive emergency

Which term describes a situation in which blood pressure is very elevated but there is no evidence of impending or progressive target organ damage? (Chapter 31) A) Hypertensive urgency B) Hypertensive emergency C) Primary hypertension D) Secondary hypertension

A) Hypertensive urgency

A nurse reviewing a client's echocardiogram report reads the following statements: "The heart muscle is asymmetrically thickened and the overall size and mass are increased, especially along the septum. The ventricular walls are thickened, reducing the size of the ventricular cavities. Several areas of the myocardium show evidence of scarring." The nurse knows these manifestations are indicative of which type of cardiomyopathy? (Chapter 28, Figure 28-8, Page 802) A) Hypertrophic B) Arrhythmogenic right ventricular cardiomyopathy C) Restrictive D) Dilated

A) Hypertrophic

The nurse identifies which of the following as a potential cause of premature ventricular complexes (PVCs)? (Chapter 26, Page 729) A) Hypokalemia B) Alkalosis C) Hypovolemia D) Bradycardia

A) Hypokalemia

The nurse assesses a client with a heart rate of 120 beats per minute. What are the known causes of sinus tachycardia? (Chapter 26, Page 719) A) Hypovolemia B) Vagal stimulation C) Hypothyroidism D) Digoxin

A) Hypovolemia

What is the treatment of choice for ventricular fibrillation? (Chapter 26, Page 731) A) Immediate bystander CPR B) Pacemaker C) Implanted defibrillator D) Atropine

A) Immediate bystander CPR

The nurse reviews a client's lab results and notes a serum calcium level of 7.9 mg/dL. It is most appropriate for the nurse to monitor the client for what condition? (Chapter 25, Page 694) A) Impaired myocardial contractility B) Enhanced sensitivity to digitalis C) Increased risk of heart block D) Inclination to ventricular fibrillation

A) Impaired myocardial contractility

When the nurse notes that, after cardiac surgery, the client demonstrates low urine output (less than 25 mL/h) with high specific gravity (greater than 1.025), the nurse suspects which condition? (Chapter 27, Page 783) A) Inadequate fluid volume B) Normal glomerular filtration C) Overhydration D) Anuria

A) Inadequate fluid volume

Which is the most important postoperative assessment parameter for a client recovering from cardiac surgery? (Chapter 27, Page 769) A) Inadequate tissue perfusion B) Mental alertness C) Blood glucose concentration D) Activity intolerance

A) Inadequate tissue perfusion

The nurse notes that the post cardiac surgery client demonstrates low urine output (< 25 mL/hr) with high specific gravity (> 1.025). What will the nurse anticipate the health care provider will order? (Chapter 27, Page 781) A) Increase intravenous fluids B) Decrease intravenous fluids C) Irrigate the urinary catheter D) Prepare the client for dialysis

A) Increase intravenous fluids

A client is diagnosed with deep vein thrombosis (DVT). Which nursing diagnosis should receive highest priority at this time? (Chapter 30) A) Ineffective peripheral tissue perfusion related to venous congestion B) Impaired gas exchange related to increased blood flow C) Risk for injury related to edema D) Excess fluid volume related to peripheral vascular disease

A) Ineffective peripheral tissue perfusion related to venous congestion

A client asks the nurse what causes the heart to be an effective pump. The nurse informs the client that this is due to the: (Chapter 26, Page 713) A) Inherent rhythmicity of cardiac muscle tissue B) Inherent rhythmicity of all muscle tissue C) Sufficient blood pressure D) Inherent electrons in muscle tissue

A) Inherent rhythmicity of cardiac muscle tissue

Which of the following is the hallmark symptom for peripheral arterial disease (PAD) in the lower extremity? (Chapter 30) A) Intermittent claudication B) Dizziness C) Acute limb ischemia D) Vertigo

A) Intermittent claudication

A client with dilated cardiomyopathy is having frequent episodes of ventricular fibrillation. What medical treatment does the nurse anticipate the client will have to terminate the episode of ventricular fibrillation? (Chapter 26, Page 744) A) Internal cardioverter defibrillator insertion B) Pacemaker insertion C) Radiofrequency ablation D) Electrophysiological study

A) Internal cardioverter defibrillator insertion

A client is being seen at the clinic on a monthly basis for assessment of blood pressure. The client has been checking blood pressure at home as well and has reported a systolic pressure of 158 and a diastolic pressure of 64. What does the nurse suspect this client is experiencing? (Chapter 31) A) Isolated systolic hypertension B) Secondary hypertension C) Hypertensive urgency D) Primary hypertension

A) Isolated systolic hypertension

The nurse is educating a patient diagnosed with angina pectoris about the difference between the pain of angina and a myocardial infarction (MI). How should the nurse describe the pain experienced during an MI? Select all that apply. (Chapter 27, Page 763) A) It is substernal in location B) It is sudden in onset and prolong in duration C) It is viselike and can radiate to the shoulders and arms D) It is relieved by rest and inactivity E) It subsides with Nitroglycerin

A) It is substernal in location B) It is sudden in onset and prolonged in duration. C) It is viselike and radiates to the shoulders and arms

A nurse is educating about lifestyle modifications for a group of clients with newly diagnosed hypertension. While discussing dietary changes, which point would the nurse emphasize? (Chapter 31) A) It takes 2 to 3 months for the taste buds to adapt to decreased salt intake B) There is usually no need to change alcohol consumption for clients with hypertension C) The taste buds never adapt to decreased salt intake D) A person with hypertension should never consume alcohol

A) It takes 2 to 3 months for the taste buds to adapt to decreased salt intake

Vasodilation or vasoconstriction produced by an external cause will interfere with a nurse's accurate assessment of a client with peripheral vascular disease (PVD). Therefore, the nurse should: (Chapter 30) A) Keep the client warm B) Maintain room temperature at 78° F (25.6° C) C) Keep the client uncovered D) Match the room temperature to the client's body temperature

A) Keep the client warm

The nurse is caring for a client who is scheduled for a transesophageal echocardiogram. What nursing intervention is a priority after the procedure? (Chapter 25, Page 703) A) Keep the head of the bed elevated 45 degrees and keep NPO until return of the gag reflex B) Monitor the puncture site and assess the affected extremity C) Keep the client turned to the right side and watch for bleeding from the site D) Observe for bloody urine and stools

A) Keep the head of the bed elevated 45 degrees and keep NPO until return of the gag reflex

Each chamber of the heart has a particular role in maintaining cellular oxygenation. Which chamber is responsible for pumping blood to all the cells and tissues of the body? (Chapter 25, Page 673) A) Left ventricle B) Left atrium C) Right ventricle D) Right atrium

A) Left ventricle

The nurse is caring for an older adult client who has come to the clinic for a yearly physical. When assessing the client, the nurse notes the blood pressure (BP) is 140/93. The nurse knows that in older clients what happens that may elevate the systolic BP? (Chapter 31, Page 887) A) Loss of arterial elasticity B) Decrease in blood volume C) Increase in calcium intake D) Decrease in cardiac output

A) Loss of arterial elasticity

A client has had an echocardiogram to measure ejection fraction. The nurse explains that ejection fraction is the percentage of blood the left ventricle ejects upon contraction. What is the typical percentage of blood a healthy heart ejects? (ch29, pg819)

55%

While the nurse is preparing a client for a cardiac catheterization, the client states that they have allergies to seafood. Which of the following medications may the nurse give prior to the procedure? (Chapter 25, Page 701) A) Methylprednisolone B) Furosemide C) Lorazepam D) Phenytoin

A) Methylprednisolone

A client is diagnosed with peripheral arterial disease. Review of the client's chart shows an ankle-brachial index (ABI) on the right of 0.45. This indicates that the right foot has which of the following? (Chapter 30, Page 847) A) Moderate to severe arterial insufficiency B) No arterial insufficiency C) Very mild arterial insufficiency D) Tissue loss to that foot

A) Moderate to severe arterial insufficiency

Which assessments should a nurse perform when caring for a client following a cardiac catheterization? Select all that apply. (Chapter 25, Page 703) A) Monitor BP and pulse frequently B) Inspect pressure dressing for signs of bleeding C) Palpate the pulse in different locations D) Inspect the color in every extremity E) Palpate the insertion site for tenderness

A) Monitor BP and pulse frequently B) Inspect pressure dressing for signs of bleeding C) Palpate the pulse in different locations

The nurse expects to see which of the following characteristics on an ECG strip for a patient who has third-degree AV block? (Chapter 26, Page 734) A) More P waves than QRS complexes B) Shortened QRS duration. C) Atrial rate of 60 bpm or below D) Extended PR interval

A) More P waves than QRS complexes

The client's heart rate is observed to be 140 bpm on the monitor. The nurse knows to monitor the client for what condition? (Chapter 25) A) Myocardial ischemia B) A pulmonary embolism C) Right-sided heart failure D) A stroke

A) Myocardial ischemia

A client presents to the ED reporting anxiety and chest pain after shoveling heavy snow that morning. The client says that nitroglycerin has not been taken for months but upon experiencing this chest pain did take three nitroglycerin tablets. Although the pain has lessened, the client states, "They did not work all that well." The client shows the nurse the nitroglycerin bottle; the prescription was filled 12 months ago. The nurse anticipates which order by the physician? (Chapter 27, Page 759) A) Nitroglycerin SL B) Chest x-ray C) Serum electrolytes D) Ativan 1 mg orally

A) Nitroglycerin SL

The nurse analyzes a 6-second electrocardiogram (ECG) tracing. The P waves and QRS complexes are regular. The PR interval is 0.18 seconds long, and the QRS complexes are 0.08 seconds long. The heart rate is calculated at 70 bpm. The nurse correctly identifies this rhythm as: (Chapter 26, Page 718) A) Normal sinus rhythm B) Sinus tachycardia C) Junctional tachycardia D) First-degree atrioventricular block

A) Normal sinus rhythm

A nurse is assessing a client's right lower leg, which is wrapped with an elastic bandage. Which signs and symptoms suggest circulatory impairment? (Chapter 30) A) Numbness, cool skin temperature, and pallor B) Swelling, warm skin temperature, and drainage C) Redness, cool skin temperature, and swelling D) Numbness, warm skin temperature, and redness

A) Numbness, cool skin temperature, and pallor

What risk factors would cause the nurse to become concerned that the client may have atherosclerotic heart disease? Select all that apply. (Chapter 31) A) Obesity B) Family history of early cardiovascular events C) Hypertension D) Lowered triglyceride levels E) Active lifestyle F) Diabetes

A) Obesity B) Family history of early cardiovascular events C) Hypertension F) Diabetes

The nurse cares for a client with clubbing of the fingers and toes. The nurse should complete which action given these findings? (Chapter 25, Page 687) A) Obtain an oxygen saturation level B) Assess the client's capillary refill C) Assess the client for pitting edema D) Obtain a 12-lead ECG tracing

A) Obtain an oxygen saturation level

The school nurse is providing care to a child with a sore throat. With any sign of throat infection, the nurse stresses which of the following? (Chapter 28, Page 810) A) Obtaining a throat culture B) Warm, salt water gargling C) Administering antiseptic lozenges D) Fluid increase to 2500cc

A) Obtaining a throat culture

To evaluate a client's atrial depolarization, the nurse observes which part of the electrocardiogram waveform? (Chapter 26, Page 715) A) P wave B) PR interval C) QRS complex D) T wave

A) P wave

A client has been diagnosed with peripheral arterial occlusive disease. Which of the following instructions is appropriate for the nurse to give the client for promoting circulation to the extremities? (Chapter 30, Page 858) A) Participate in a regular walking program B) Keep the extremities elevated slightly C) Use a heating pad to promote warmth D) Massage the calf muscles if pain occurs

A) Participate in a regular walking program

The nurse completes an assessment of a client admitted with a diagnosis of right-sided heart failure. What will be a significant clinical finding related to right-sided heart failure? (ch29 pg823) A) Pitting edema B) Decreased O2 saturation levels C) S4 ventricular gallop sign D) Oliguria

A) Pitting edema

A 26-year-old client, who has been diagnosed with paroxysmal supraventricular tachycardia (PSVT), is treated in the emergency department. The client is experiencing occasional runs of PSVT lasting up to several minutes at a time. During these episodes, the client becomes lightheaded but does not lose consciousness. Which maneuver(s) may be used to interrupt the client's atrioventricular nodal reentry tachycardia (AVNRT)? Select all that apply. (Chapter 26, Page 728) A) Placing the client's face in cold water B) Instructing the client to breathe deeply C) Performing carotid massage D) Stimulating the client's gag reflex E) Instructing the client to vigorously exercise

A) Placing the client's face in cold water C) Performing carotid massage D) Stimulating the client's gag reflex

The nurse is teaching a client diagnosed with coronary artery disease about nitroglycerin. What is the cardiac premise behind administration of nitrates? (Chapter 27, Page 759) A) Preload is reduced B) More blood returns to the heart C) It increases myocardial oxygen consumption D) It functions has a vasoconstrictor

A) Preload is reduced

A healthy adult client is seeing a health care provider for an annual physical examination. While the nurse is taking the client's vital signs, the client states, "Occasionally, my heart skips a beat." The nurse believes that the client is experiencing what condition? (Chapter 26, Page 720) A) Premature atrial complex B) Atrial flutter C) Sinus tachycardia D) Ventricular fibrillation

A) Premature atrial complex

A client presents to the ED with a myocardial infarction. Prior to administering a prescribed thrombolytic agent, the nurse must determine whether the client has which absolute contraindication to thrombolytic therapy? (Chapter 27, Page 867) A) Prior intracranial hemorrhage B) Shellfish allergy C) Use of heparin D) Recent consumption of a meal

A) Prior intracranial hemorrhage

The nurse is caring for a client following a coronary artery bypass graft (CABG). The nurse notes persistent oozing of bloody drainage from various puncture sites. The nurse anticipates that the physician will order which medication to neutralize the unfractionated heparin the client received? (Chapter 27, Page 776) A) Protamine sulfate B) Alteplase C) Clopidogrel D) Aspirin

A) Protamine sulfate

The nurse is assessing a patient's blood pressure. What does the nurse document as the difference between the systolic and the diastolic pressure? (Chapter 25, Page 687) A) Pulse pressure B) Auscultatory gap C) Pulse deficit D) Korotkoff sound

A) Pulse pressure

The nurse analyzes the electrocardiogram (ECG) tracing of a client newly admitted to the cardiac step-down unit with a diagnosis of chest pain. Which finding indicates the need for follow-up? (Chapter 26, Page 716) A) QT interval that is 0. 46 seconds long B) PR interval that is 0.18 seconds long C) QRS complex that is 0.10 seconds long D) ST segment that is isoelectric in appearance

A) QT interval that is 0. 46 seconds long

A 28-year-old client presents to the emergency department, stating severe restlessness and anxiety. Upon assessment, the client's heart rate is 118 bpm and regular, the client's pupils are dilated, and the client appears excitable. Which action should the nurse take next? (Chapter 26, Page 719) A) Question the client about alcohol and illicit drug use B) Instruct the client to hold the breath and bear down C) Prepare to administer a calcium channel blocker D) Place the client on supplemental oxygen

A) Question the client about alcohol and illicit drug use

The clinic nurse is assessing a client's pulse before outpatient diagnostic testing. What should the nurse document when assessing the client's pulse? (Chapter 25, Page 688) A) Rate, quality, and rhythm B) Pressure, rate, and rhythm C) Rate, rhythm, and volume D) Quality, volume, and rate

A) Rate, quality, and rhythm

Which term is refers to hypertension in which blood pressure that is controlled with therapy becomes uncontrolled (abnormally high) when the therapy is discontinued? (Chapter 31) A) Rebound B) Secondary C) Essential D) Primary

A) Rebound

The physician orders medication to treat a client's cardiac ischemia. What is causing the client's condition? (Chapter 25, Page 675) A) Reduced blood supply to the heart B) Pain on exertion C) High blood pressure D) Indigestion

A) Reduced blood supply to the heart

The nurse is assessing a client taking an anticoagulant. What nursing intervention is most appropriate for a client at risk for injury related to side effects of medication enoxaparin? (Chapter 25, Page 683) A) Report any incident of bloody urine, stools, or both B) Administer calcium supplements C) Assess for hypokalemia D) Assess for clubbing of the fingers

A) Report any incident of bloody urine, stools, or both

Which finding indicates that hypertension is progressing to target organ damage? (Chapter 31) A) Retinal blood vessel damage B) Chest x-ray showing pneumonia C) Urine output of 60 mL over 2 hours D) Blood urea nitrogen concentration of 12 mg/dL

A) Retinal blood vessel damage

A client comes to the clinic reporting fever, chills, and sore throat and is diagnosed with streptococcal pharyngitis. A nurse knows that early diagnosis and effective treatment is essential to avoid which preventable disease? (Chapter 28, Page 794) A) Rheumatic fever B) Pericarditis C) Mitral stenosis D) Cardiomyopathy

A) Rheumatic fever

A nurse is obtaining a history from a new client in the cardiovascular clinic. When investigating for childhood diseases and disorders associated with structural heart disease, which finding should the nurse consider significant? (Chapter 28, Page 809) A) Rheumatic fever B) Croup C) Medullary sponge kidney (MSK) D) Severe staphylococcal infection

A) Rheumatic fever

Central venous pressure is measured in which heart chamber? (Chapter 25, Page 688) A) Right atrium B) Left atrium C) Left ventricle D) Right ventricle

A) Right atrium

The nurse is placing electrodes for a 12-lead electrocardiogram (ECG). The nurse would be correct in placing an electrode on which area for V1? (Chapter 26, Page 715) A) Right side of sternum, fourth intercostal space B) Left side of sternum, fourth intercostal space C) Midway between V2 and V4 D) Mid-clavicular line, fifth intercostal space

A) Right side of sternum, fourth intercostal space

Each chamber of the heart has a particular role in maintaining cellular oxygenation. Which chamber is responsible for pumping blood to the lungs to be oxygenated? (Chapter 25, Page 673) A) Right ventricle B) Left ventricle C) Right atrium D) Left atrium

A) Right ventricle

A client in the ICU has a central venous pressure (CVP) line placed. The CVP reading is 10 mm Hg. To what condition does the nurse correlate the CVP reading? (Chapter 25) A) Right-sided heart failure B) Hypovolemia C) Left-sided heart failure D) Reduction in preload

A) Right-sided heart failure

Which of the following is the most effective intervention for preventing progression of vascular disease? (Chapter 30, Page 862) A) Risk factor modification B) Use neutral soaps C) Avoid trauma D) Wear sturdy shoes

A) Risk factor modification

A client, newly admitted to the nursing unit, has a primary diagnosis of renal failure. When assessing the client, the nurse notes a blood pressure (BP) of 180/100. The nurse knows that this is what kind of hypertension? (Chapter 31) A) Secondary B) Essential C) Primary D) Malignant

A) Secondary

A client reports recent onset of chest pain that occurs sporadically with exertion. The client also has fatigue and mild ankle swelling, which is most pronounced at the end of the day. The nurse suspects a cardiovascular disorder. What other client report increases the likelihood of a cardiovascular disorder? (Chapter 25, Page 678) A) Shortness of breath B) Insomnia C) Irritability D) Lower substernal abdominal pain

A) Shortness of breath

The nurse uses which term for the normal pacemaker of the heart? (Chapter 25) A) Sinoatrial (SA) node B) Purkinje fibers C) Atrioventricular (AV) node D) Bundle of His

A) Sinoatrial (SA) node

The nurse is teaching a beginning EKG class to staff nurses. As the nurse begins to discuss the parts of the EKG complex, one of the students asks what the normal order of conduction through the heart is. What order does the nurse describe? (Chapter 26, Page 713) A) Sinoatrial (SA) node, atrioventricular (AV) node, bundle of His, right and left bundle branches, and the Purkinje fibers B) AV node, SA node, bundle of His, right and left bundle branches, and the Purkinje fibers C) SA node, AV node, right and left bundle branches, bundle of His, and the Purkinje fibers D) SA node, AV node, bundle of His, the Purkinje fibers, and the right and left bundle branches

A) Sinoatrial (SA) node, atrioventricular (AV) node, bundle of His, right and left bundle branches, and the Purkinje fibers

Which dysrhythmia is common in older clients? (Chapter 26, Page 718) A) Sinus bradycardia B) Sinus tachycardia C) Sinus arrhythmia D) Ventricular tachycardia

A) Sinus bradycardia

What should the nurse do to manage persistent swelling in a client with severe lymphangitis and lymphadenitis? (Chapter 30, Page 880) A) Teach the client how to apply a graduated compression stocking B) Inform the physician if the client's temperature remains low C) Avoid elevating the area D) Offer cold applications to promote comfort and to enhance circulation

A) Teach the client how to apply a graduated compression stocking

The client is prescribed nadolol for hypertension. What is the reason the nurse will teach the client not to stop taking the medication abruptly? (Chapter 27, Page 759) A) The abrupt stop can cause a myocardial infarction B) The abrupt stop can lead to formation of blood clots C) The abrupt stop will precipitate internal bleeding D) The abrupt stop can trigger a migraine headache

A) The abrupt stop can cause a myocardial infarction

The client is admitted for a scheduled cardiac catheterization. On the morning of the procedure, while assessing the client's morning laboratory values, the nurse notes a blood urea nitrogen (BUN) of 34 mg/dL and a creatinine of 4.2 mg/dL. What priority reason will the nurse notify the healthcare provider? (Chapter 25, Page 703) A) The client is at risk for renal failure due to the contrast agent that will be given during the procedure B) These values show a risk for dysrhythmias C) The client is overhydrated, which puts him at risk for heart failure during the procedure D) The client is at risk for bleeding

A) The client is at risk for renal failure due to the contrast agent that will be given during the procedure

The nurse checks the synchronizer switch before using a defibrillator to terminate ventricular fibrillation for what important reason? (Chapter 26, Page 737) A) The defibrillator won't deliver a shock if the synchronizer switch is turned on B) The delivered shock must be synchronized with the client's QRS complex C) The shock must be synchronized with the client's T wave D) The defibrillator won't deliver a shock if the synchronizer switch is turned off

A) The defibrillator won't deliver a shock if the synchronizer switch is turned on

A nurse is caring for a young female adult client diagnosed with atrial fibrillation who has just had a mitral valve replacement. The client is being discharged with prescribed warfarin. The client mentions to the nurse that she relies on the rhythm method for birth control. What education will be a priority for the nurse to provide to this client? (Chapter 28, Page 800) A) The high risk for complications if she becomes pregnant while taking warfarin B) Foods to limit (green leafy vegetables) while taking warfarin C) Symptoms to report of worsening tachycardia related to atrial fibrillation D) Instructions for using the rhythm method

A) The high risk for complications if she becomes pregnant while taking warfarin

A client with heart failure asks the nurse how dobutamine affects the body's circulation. What is the nurse's best response? (Chapter 26, Page 713) A) The medication increases the force of the myocardial contraction B) The medication causes the kidneys to retain fluid and increase intravascular volume C) The medication increases the heart rate D) The medication helps the kidneys produce more urine

A) The medication increases the force of the myocardial contraction

A client has been diagnosed with atrial fibrillation and has been prescribed warfarin therapy. What should the nurse prioritize when providing health education to the client? (Chapter 26) A) The need to have regular blood levels drawn B) The need to sit upright for 30 minutes after taking the medication C) The importance of taking the medication 1 hour before or 2 hours after a meal D) The importance of adequate fluid intake

A) The need to have regular blood levels drawn

The nurse is caring for a patient with a diagnosis of pericarditis. Where does the nurse understand the inflammation is located? (Chapter 25) A) The thin fibrous sac encasing the heart B) The heart's muscle fibers C) The inner lining of the heart and valves D) The exterior layer of the heart

A) The thin fibrous sac encasing the heart

A patient recently diagnosed with pericarditis asks the nurse to explain what area of the heart is involved. How does the nurse best describe the pericardium to the client? (Chapter 25, Page 673) A) Thin fibrous sac that encases the heart B) Inner lining of the heart and valves C) Heart's muscle fibers D) Exterior layer of the heart

A) Thin fibrous sac that encases the heart

Which of the following is the most common site for a dissecting aneurysm? (Chapter 30, Page 862) A) Thoracic area B) Lumbar area C) Sacral area D) Cervical area

A) Thoracic area

Which of the following is the most common complication of prosthetic valves? (Chapter 28) A) Thromboembolism B) Infection C) Arrhythmias D) Hemolysis

A) Thromboembolism

The nurse working in the emergency department places a client in anaphylactic shock on a cardiac monitor and sees the cardiac rhythm shown. Which dysrythmia should the nurse document? (Chapter 26, Page 730) A) Ventricular tachycardia B) Ventricular asystole C) Sinus rhythm D) Junctional rhythm E) Atrial fibrillation

A) Ventricular tachycardia

During auscultation of the lungs, what would a nurse note when assessing a client with left-sided heart failure? (Chapter 25, Page 692) A) Wheezes with wet lung sounds B) Stridor C) High-pitched sounds D) Laborious breathing

A) Wheezes with wet lung sounds

Which nursing actions would be of greatest importance in the management of a client preparing for angioplasty (Chapter 27, Page 771) A) Withhold anticoagulant therapy B) Inform client of diagnostic tests C) Remove hair from skin insertion sites D) Assess distal pulses

A) Withhold anticoagulant therapy

A client is admitted to the emergency department with chest pain and doesn't respond to nitroglycerin. The health care team obtains an electrocardiogram and administers I.V. morphine. The health care provider also considers administering alteplase. This thrombolytic agent must be administered how soon after onset of myocardial infarction (MI) symptoms? (Chapter 27, Page 766) A) Within 6 hours B) Within 12 hours C) Within 24 to 48 hours D) Within 5 to 7 days

A) Within 6 hours

The cardiologist has scheduled a client for drug-induced stress testing. What instructions should the nurse provide to prepare the client for this test? (Chapter 25, Page 700) A) You will receive medication via IV administration B) You will need to wear comfortable shoes to the test C) You will begin exercising at a slow speed D) You may experience an onset of dizziness during the test

A) You will receive medication via IV administration

The nurse is preparing a client for transesophageal echocardiography (TEE). This procedure is used for which indication? (Chapter 25) A) determination of atrial thrombi B) evaluation of myocardial perfusion at rest and after exercise C) determination of electrical activity of the heart D) evaluation of the response of the cardiovascular system to increased oxygen demands

A) determination of atrial thrombi

The nurse is performing an assessment for an older adult client and auscultates an S3 heart sound. What condition does the nurse determine may correlate with this finding? (Chapter 25) A) heart failure B) coronary artery disease C) congenital heart disease D) aortic stenosis

A) heart failure

On a routine visit to the physician, a client with chronic arterial occlusive disease reports that he's stopped smoking after 34 years. To relieve symptoms of intermittent claudication, a condition associated with chronic arterial occlusive disease, which additional measure should the nurse recommend? A) Reducing daily fat intake to less than 45% of total calories B) Engaging in anaerobic exercise C) Abstaining from foods that increase levels of high-density lipoproteins (HDLs) D) Taking daily walks

D) Taking daily walks

A client needs additional information about her heart condition. The client states to the nurse, "What is considered the pacemaker of the heart?" (Chapter 25, Page 675) A) The AV node B) The bundle of HIS C) The Purkinje fibers D) The SA node

D) The SA node

What is considered the pacemaker of the heart? (Chapter 25) A) The AV node B) The bundle of HIS C) The Purkinje fibers D) The SA node

D) The SA node

The nurse is discussing the cardiac system with a client admitted with heart failure. The client asks "What determines the heart rate?" What is the nurse's best response? (Chapter 25, Page 819) A) Force of contractility controls the heart rate B) Preload controls the heart rate C) Stroke volume controls the heart rate D) The autonomic nervous system controls the heart rate

D) The autonomic nervous system controls the heart rate

You are evaluating the expected outcomes on a client who is recovering from a cardiac catheterization. What is an expected outcome that you would evaluate? (Chapter 25, Page 705) A) The client and family understands the client's CV diagnosis B) The client and family understands the need for medication C) The client and family understands the need to restrict activity for 72 hours D) The client and family understands the discharge instructions

D) The client and family understands the discharge instructions

A nurse is evaluating a client who had a myocardial infarction (MI) 7 days earlier. Which outcome indicates that the client is responding favorably to therapy? (Chapter 27) A) The client verbalizes the intention of making all necessary lifestyle changes except for stopping smoking B) The client exhibits a heart rate above 100 beats/minute C) The client states that sublingual nitroglycerin usually relieves his chest pain D) The client demonstrates ability to tolerate more activity without chest pain

D) The client demonstrates ability to tolerate more activity without chest pain

A nurse is caring for a client with pericarditis and auscultates a pericardial friction rub. What action does the nurse ask the client to do to distinguish a pericardial friction rub from a pleural friction rub? (Chapter 28, Page 815) A) The nurse places the client flat for at least 4 minutes B) The nurse has the client stand during auscultation C) There is really no question to ask the client to tell the difference D) The nurse asks the client to hold the breath during auscultation

D) The nurse asks the client to hold the breath during auscultation

A nurse is caring for a client who had a three-vessel coronary bypass graft 4 days earlier. The client's cholesterol profile is as follows: total cholesterol 265 mg/dl, low-density lipoprotein (LDL) 139 mg/dl, and high-density lipoprotein (HDL) 32 mg/dl. The client asks the nurse how to lower his cholesterol. What is the best response by the nurse? (Chapter 27, Page 754) A) Cholesterol is within the recommended guidelines and the client doesn't need to lower it B) Client should take statin medication and not worry about cholesterol C) Client should begin a running program, working up to 2 miles per day D) The nurse will ask the dietitian to talk with the client about modifying the diet

D) The nurse will ask the dietitian to talk with the client about modifying the diet

The nurse documents that a client is having a normal sinus rhythm. What characteristics of this rhythm has the nurse assessed? (Chapter 26, Page 718) A) Heart rate between 60 and 150 beats per minute B) Impulse travels to the atrioventricular (AV) node in 0.15 to 0.5 seconds C) The ventricles depolarize in 0.5 seconds or less D) The sinoatrial (SA) node initiates the impulse

D) The sinoatrial (SA) node initiates the impulse

The most important factor regulating the caliber of blood vessels, which determines resistance to flow, is: (Chapter 30, Page 843) A) Hormonal secretion B) Independent arterial wall activity C) The influence of circulating chemicals D) The sympathetic nervous system

D) The sympathetic nervous system

The nurse is teaching a client with cardiomyopathy. Which statement is a valid teaching point? (Chapter 28, Page 803) A) Clients with cardiomyopathy have a goal to improve lung function B) The disease was inherited, so there are no reversal treatments C) Clients with cardiomyopathy often need to live in a skilled nursing care facility D) The treatments include medications, medical devices, surgery, or transplantation

D) The treatments include medications, medical devices, surgery, or transplantation

When no atrial impulse is conducted through the AV node into the ventricles, the client is said to be experiencing which type of AV block? (Chapter 26, Page 732) A) Second degree, type I B) Second degree, type II C) First degree D) Third degree

D) Third degree

The nurse knows which heart rhythm occurs when the atrial and ventricular rhythms are both regular, but independent of each other? (Chapter 26) A) Asystole B) First-degree AV block C) Second-degree heart block D) Third-degree atrioventricular (AV) heart block

D) Third-degree atrioventricular (AV) heart block

Which electrocardiogram (ECG) characteristic is usually seen when a client's serum potassium level is low? (Chapter 26, Page 712) A) T wave B) QT interval C) P wave D) U wave

D) U wave

What symptoms should the nurse assess for in a client with lymphedema as a result of impaired nutrition to the tissue? (Chapter 30) A) Cyanosis B) Evident scarring C) Loose and wrinkled skin D) Ulcers and infection in the edematous area

D) Ulcers and infection in the edematous area

A client who is resting quietly reports chest pain to the nurse. The cardiac monitor indicates the presence of reversible ST-segment elevation. What type of angina is the client experiencing? (Chapter 27, Page 757) A) Silent angina B) Stable angina C) Intractable angina D) Variant angina

D) Variant angina

A patient who had a myocardial infarction is experiencing severe chest pain and alerts the nurse. The nurse begins the assessment but suddenly the patient becomes unresponsive, no pulse, with the monitor showing a rapid, disorganized ventricular rhythm. What does the nurse interpret this rhythm to be? (Chapter 26, Page 730) A) Ventricular tachycardia B) Atrial fibrillation C) Third-degree heart block D) Ventricular fibrillation

D) Ventricular fibrillation

The nurse is proving discharge instructions for a client with a new arrhythmia. Which statement should the nurse include? (Chapter 26, Page 735) A) It is not necessary to learn how to take your own pulse B) Do not be concerned if you experience symptoms of lightheadedness and dizziness C) If you miss a dose of your antiarrhythmic medication, double up on the next dose D) Your family and friends may want to take a CPR class

D) Your family and friends may want to take a CPR class

A client experiences a faster-than-normal heart rate when drinking more than two cups of coffee in the morning. What does the nurse identify on the electrocardiogram as an indicator of sinus tachycardia? (Chapter 26) A) PR interval of 0.1 seconds B) QRS duration of 0.16 seconds C) Q wave of 0.04 seconds D) heart rate of 118 bpm

D) heart rate of 118 bpm

The nurse correctly identifies which data as an example of blood pressure and heart rate measurements in a client with postural hypotension? (Chapter 25) A) supine: BP 114/82 mm Hg, HR 90 bpm; sitting: BP 110/76 mm Hg, HR 95 bpm; standing: BP 108/74 mm Hg, HR 98 bpm B) supine: BP 140/78 mm Hg, HR 72 bpm; sitting: BP 145/78 mm Hg, HR 74 bpm; standing: BP 144/78 mm Hg, HR 74 bpm C) supine: BP 130/70 mm Hg, HR 80 bpm; sitting: BP 128/70 mm Hg, HR 80 bpm; standing: BP 130/68 mm Hg, HR 82 bpm D) supine: BP 120/70 mm Hg, HR 70 bpm; sitting: BP 100/55 mm Hg, HR 90 bpm; standing: BP 98/52 mm Hg, HR 94 bpm

D) supine: BP 120/70 mm Hg, HR 70 bpm; sitting: BP 100/55 mm Hg, HR 90 bpm; standing: BP 98/52 mm Hg, HR 94 bpm

Your client is being prepared for echocardiography when they ask you why they need to have this test. What would be your best response? (Chapter 25) A) "Echocardiography will tell your doctor if you have cancer of the heart." B) "This test will find any congenital heart defects." C) "This test can tell us a lot about your heart." D)"Echocardiography is a way of determining the functioning of the left ventricle of your heart."

D)"Echocardiography is a way of determining the functioning of the left ventricle of your heart."

A patient is undergoing a pericardiocentesis. Following withdrawal of pericardial fluid, which assessment by the nurse indicates that cardiac tamponade has been relieved? (ch29 pg836)

Decrease in central venous pressure (CVP)

The nurse assessing a client who has arterial insufficiency of the legs and an ulcer on the left great toe would expect to find which characteristic? (ch30 pg845) a. Diminished or absent pulses b. Superficial ulcer c. Aching, cramping pain d. Pulses that are present but difficult to palpate

Diminished or absent pulses

Which is a characteristic of arterial insufficiency? (ch30 pg845) a. Diminished or absent pulses b. Superficial ulcer c. Aching, cramping pain d. Pulses are present but may be difficult to palpate

Diminished or absent pulses

Which aneurysm results in bleeding into the layers of the arterial wall? (ch30 pg862) a. Saccular b. Dissecting c. False d. Anastomotic

Dissecting

Which is a cerebrovascular manifestation of heart failure? (ch29 pg822)

Dizziness

A patient with long-standing hypertension is admitted to the hospital with hypertensive urgency. The physician orders a chest x-ray, which reveals an enlarged heart. What diagnostic test does the nurse anticipate preparing the patient for to determine left ventricular enlargement?

Echocardiography

Which diagnostic method is recommended to determine whether left ventricular hypertrophy has occurred?

Echocardiography

The nurse is caring for a client with cardiac compromise related to mitral valve impairment. Which outcome of the eroding of the mitral valve is most significant? (ch28 pg792)

Heart failure

Providing postoperative care to a patient who has percutaneous transluminal angioplasty (PTA), with insertion of a stent, for a femoral artery lesion, includes assessment for the most serious complication of: (ch30 pg852) a. Hemorrhage. b. Thrombosis of the graft. c. Decreased motor function. d. Stent dislodgement.

Hemorrhage.

A client is receiving enoxaparin and warfarin therapy for a venous thromboembolism (VTE). Which laboratory value indicates that anticoagulation is adequate and enoxaparin can be discontinued? (ch30 pg872) a. Activated partial thromboplastin time (aPPT) is half of the control value b. Prothrombin time (PT) is 0.5 times normal. c. International normalized ratio (INR) is 2.5. d. K+ level is 3.5.

International normalized ratio (INR) is 2.5.

Which is a characteristic of right-sided heart failure? (ch29 pg823)

Jugular vein distention

A client who suffered blunt chest trauma in a motor vehicle accident reports chest pain during deep inspiration. On auscultation, the nurse detects a pericardial friction rub — a classic sign of acute pericarditis. To relieve this chest pain, which position should the nurse encourage the client to assume? (ch28 pg814)

Leaning forward while sitting

The clinical manifestations of cardiogenic shock reflect the pathophysiology of heart failure (HF). By applying this correlation, the nurse notes that the degree of shock is proportional to which of the following? (ch29 pg835)

Left ventricular function

The nurse is preparing a client for a multiple gated acquisition (MUGA) scan. What would be an important instruction for the nurse to give a client who is to undergo a MUGA scan? (ch29)

Lie very still at intermittent times during the test.

Which feature is the hallmark of systolic heart failure? (ch29 pg819)

Low ejection fraction (EF)

The nurse is providing discharge instructions to a client with heart failure preparing to leave the following day. What type of diet should the nurse request the dietitian to discuss with the client? (ch29 pg826)

Low-sodium diet

A nurse is completing an assessment on a client and discovers an enlarged, red, and tender lymph node. The nurse will describe and document the lymph node using which term? (ch30 pg880) a. Lymphadenitis b. Lymphangitis c. Lymphedema d. Elephantiasis

Lymphadenitis

Which action will the nurse include in the plan of care for a client admitted with acute decompensated heart failure (ADHF) who is receiving milrinone? (ch29 pg826)

Monitor blood pressure frequently

The nurse is instructing a student on the proper technique for measuring blood pressure (BP). Which student action indicates a need for further teaching?

Positions the arm at waist level

An older adult client has newly diagnosed stage 2 hypertension. The health care provider has prescribed Chlorothiazide and Benazepril. What will the nurse monitor this client for?

Postural hypotension and resulting injury

A nurse in the emergency department is caring for a client with acute heart failure. Which laboratory value is most important for the nurse to check before administering medications to treat heart failure? (ch29 pg826)

Potassium

A client is taking amiloride and lisinopril for the treatment of hypertension. What laboratory studies should the nurse monitor while the client is taking these two medications together?

Potassium level

Which term describes the degree of stretch of the ventricular cardiac muscle fibers at the end of diastole? (ch29 pg677)

Preload

A patient with pericarditis is experiencing cardiac tamponade. Which collaborative intervention should the nurse anticipate for this patient?

Prepare for pericardiocentesis.

A physician orders blood coagulation tests to evaluate a client's blood-clotting ability. The nurse knows that such tests are important in assessing clients at risk for thrombi, such as those with a history of atrial fibrillation, infective endocarditis, prosthetic heart valves, or myocardial infarction. Which test determines a client's response to oral anticoagulant drugs? (ch30 pg871-872) a. Bleeding time b. Platelet count c. Prothrombin time (PT) d. Partial thromboplastin time (PTT)

Prothrombin time (PT)

A hospitalized client with heart failure puts on the call light and states, "I've become very short of breath, and I've been coughing up this pink frothy sputum." The nurse immediately suspects which of the following complications? (ch29 pg833)

Pulmonary edema

A 35-year-old client has been diagnosed with hypertension. The client is a stock broker, smokes daily, and has diabetes. During a follow-up appointment, the client states that regular visits to the doctor just to check blood pressure (BP) are cumbersome and time consuming. As the nurse, which aspect of client teaching would you recommend?

Purchasing a self-monitoring BP cuff

A client with infective endocarditis is assessed by the nurse for the presence of Janeway lesions. On inspection, the nurse recognizes these lesions by identifying which characteristic sign? (ch28 pg810-811)

Red or purple macules found on the palms of the hands

When assessing a client with cellulitis of the right leg, which finding should the nurse expect to observe? (ch30 pg881) a. Painful skin that is swollen and pale in color b. Cold, red skin c. Small, localized blackened area of skin d. Red, swollen skin with inflammation spreading to surrounding tissues

Red, swollen skin with inflammation spreading to surrounding tissues

A client is brought to the emergency department with reports of a bad headache and an increase in blood pressure. The blood pressure reading obtained by the nurse is 260/180 mm Hg. What is the therapeutic goal for reduction of the mean blood pressure?

Reduce the blood pressure by 20% to 25% within the first hour of treatment.

Which intervention should the nurse include in the plan of care for a client with valvular heart disease who is experiencing pulmonary congestion? (ch28 pg796)

Rest and sleep in a chair or sit in bed with head elevated

Which type of cardiomyopathy are characterized by diastolic dysfunction caused by rigid ventricular walls that impair diastolic filling and ventricular stretch. (ch 28)

Restrictive cardiomyopathy (RCM)

Which type of cardiomyopathy are characterized by diastolic dysfunction caused by rigid ventricular walls that impair diastolic filling and ventricular stretch. (ch28 pg802)

Restrictive cardiomyopathy (RCM)

What is the most common cause of mitral stenosis? (ch28 pg 794)

Rheumatic endocarditis

The nurse is caring for a client prescribed bumetanide for the treatment of stage 2 hypertension. Which finding indicates the client is experiencing an adverse effect of the medication?

Serum potassium value of 3.0 mEq/L

The nurse is assessing a patient with suspected acute venous insufficiency. What clinical manifestations would indicate this condition to the nurse? (Select all that apply.) a. Cool and cyanotic skin b. Initial absence of edema c. Sharp pain that may be relieved by the elevation of the extremity d. Full superficial veins e. Brisk capillary refill of the toes

Sharp pain that may be relieved by the elevation of the extremity Full superficial veins Cool and cyanotic skin

What are the first symptoms of cardiac tamponade? Select all that apply. (ch28 pg815)

Shortness of breath Chest tightness Dizziness

A nurse providing education about hypertension to a community group is discussing the high risk for cardiovascular complications. What are risk factors for cardiovascular problems in clients with hypertension? Select all that apply.

Smoking Diabetes mellitus Physical inactivity

A physician admits a client to the health care facility for treatment of an abdominal aortic aneurysm. When planning this client's care, which goal should the nurse keep in mind as she formulates interventions? (ch30 pg864) a. Decreasing blood pressure and increasing mobility b. Increasing blood pressure and reducing mobility c. Stabilizing heart rate and blood pressure and easing anxiety d. Increasing blood pressure and monitoring fluid intake and output

Stabilizing heart rate and blood pressure and easing anxiety

During the physical assessment of a client with hypertension, what would the nurse expect to be the most obvious finding?

Sustained increase of either one or both systolic or diastolic measurements.

What disease processes contribute to chronic heart failure? Select all that apply. (ch29 pg819)

Tachydysrhythmias Valvular disease Renal failure

What should the nurse do to manage the persistent swelling in a client with severe lymphangitis and lymphadenitis? (ch30 pg880) a. Teach the client how to apply an elastic sleeve b. Inform the physician if the client's temperature remains low c. Avoid elevating the area d. Offer cold applications to promote comfort and to enhance circulation

Teach the client how to apply an elastic sleeve

The nurse is caring for a client with cardiogenic shock in an critical care unit and the family is asking about the intra-aortic balloon pump (IABP). What will the nurse explain is the premise of using IABP? (ch28 pg805)

The IABP is reducing the workload of the heart during the shock period.

Providing postoperative care to a patient who has percutaneous transluminal angioplasty (PTA), with insertion of a stent, for a femoral artery lesion, includes assessment for the most serious complication of: (ch30 pg863) a. The aneurysm has become obstructed. b. The aneurysm may be preparing to rupture. c. The client is experiencing inflammation of the aneurysm. d. The client is experiencing normal sensations associated with this condition.

The aneurysm may be preparing to rupture.

A patient in cardiogenic shock after a myocardial infarction is placed on an intra-aortic balloon pump (IABP). What does the nurse understand is the mechanism of action of the balloon pump? (ch 29)

The balloon inflates at the beginning of diastole and deflates before systole to augment the pumping action of the heart.

Assessment of a client on a medical surgical unit finds a regular heart rate of 120 beats per minute, audible third and fourth heart sounds, blood pressure of 84/64 mm Hg, bibasilar crackles on lung auscultation, and a urine output of 5 mL over the past hour. What is the reason the nurse anticipates transferring the client to the intensive care unit? (ch29 pg835)

The client is going into cardiogenic shock.

The nurse is teaching a client scheduled for an autograft aortic valve replacement. What statements are true about autograft replacement process? Select all that apply. (Chapter 28, Page 800)

The grafts are obtained by excising the client's own pulmonic valves and a portion of the pulmonary artery. The autograft is an alternative for children and women of child-bearing age. Aortic valve autografts have remained viable for more than 20 years.

The client asks the nurse why a stress test is needed. What statement best explains the rationale for the health care provider to order a cardiac stress test? (ch 29)

The health care provider wants to identify if the heart failure is from coronary artery disease.

The nurse is caring for a client with essential hypertension. The nurse reviews lab work and assesses kidney function. Which action of the kidney would the nurse evaluate as the body's attempt to regulate high blood pressure?

The kidney excretes sodium and water.

What is the main difference between Class I and Class II heart failure as defined by the New York Heart Association (NYHA)? (ch29 pg819)

The level of physical activity each allows

A patient is being seen in a clinic to rule out mitral valve stenosis. Which assessment data would be most significant?

The patient reports shortness of breath when walking.

A client with aortic valve endocarditis develops dyspnea, crackles in the lungs, and restlessness. The novice nurse discusses this development with the nurse manager. What statement indicates the novice nurse is applying the assessment findings? (ch 28) a. "I anticipated this complication and I will call the health care provider right now." b. "I instructed the patient to do coughing and deep breathing and I will reassess in 30 minutes." c. "I told the patient that this is a normal complication and to take deep breaths." d. "I placed the patient in a semi-Fowler's position and started an NPO diet."

a. "I anticipated this complication and I will call the health care provider right now."

A nurse working at a pediatric clinic is teaching a group of parents. A parent asks the nurse if it is okay to let the young child recover from a sore throat naturally, rather than bringing the child to the clinic for diagnosis and treatment. What is the nurse's best response? (ch 28) a. "It may be streptococcal sore throat. Rheumatic heart disease can be prevented with early treatment." b. "Health care providers tend to overtreat children with antibiotics so the child recovers quickly." c. "It is not a good idea to give antibiotics for every sore throat that your child has because of the overuse of antibiotics." d. "It is fine to let the child recover naturally; it will save you time and money."

a. "It may be streptococcal sore throat. Rheumatic heart disease can be prevented with early treatment."

During assessment of a client admitted for cardiomyopathy, the nurse notes the following symptoms: dyspnea on exertion, fatigue, fluid retention, and nausea. The initial appropriate nursing diagnosis is which of the following? (ch 28)Which symptom occurs in the client diagnosed with mitral regurgitation when pulmonary congestion occurs? Hypertension Shortness of breath A loud, blowing murmur Tachycardia a. Decreased cardiac output b. Autonomic dysreflexia c. Ineffective airway clearance d. Disturbed sensory perception

a. Decreased cardiac output

Which medication reverses digitalis toxicity? (ch 29) a. Digoxin immune FAB b. Warfarin c. Amlodipine d. Ibuprofen

a. Digoxin immune FAB

The nurse is developing a plan of care for a client following pericardiocentesis. Which interventions should the nurse implement? Choose all that apply. (ch 29)

a. Evaluate the cardiac rhythm. b. Assess vital signs every 15 minutes for the first hour. c. Monitor heart and lung sounds. d. Record fluid output.

Which rhythm is also termed a ventricular escape rhythm? (ch 26) a. Idioventricular rhythm b. Ventricular asystole c. First degree AV block d. Ventricular fibrillation

a. Idioventricular rhythm

The instructor is talking with a nursing student who is caring for a client with pericarditis. The instructor asks the student to name the main characteristic of pericarditis. What should be the student's answer? (ch 28) a. Precordial pain b. Respiratory symptoms c. Dyspnea d. Fever

a. Precordial pain

A client with infective endocarditis is assessed by the nurse for the presence of Janeway lesions. On inspection, the nurse recognizes these lesions by identifying which characteristic sign? (ch 28) a. Red or purple macules found on the palms of the hands b. Erythematosus modules on the pads of the fingers c. Splinter hemorrhages seen under the fingernails d. Patterns of petechiae on the chest

a. Red or purple macules found on the palms of the hands

A nurse is conducting a heath history on a client with a primary diagnosis of mitral stenosis. Which disorder reported by the client is the most common cause of mitral stenosis? (ch 28) a. Rheumatic endocarditis b. Congestive heart failure c. Atrial fibrillation d. Myocardial infarction

a. Rheumatic endocarditis

The laboratory values for a client diagnosed with coronary artery disease (CAD) have just come back from the lab. The client's low-density lipoprotein (LDL) level is 112 mg/dL. The nurse recognizes that this value is (ch 27 pg 753) a. above the optimal range. b. extremely high. c. below the optimal range. d. within the optimal range.

a. above the optimal range.

The client is asking the nurse about heart-healthy food choices for lunch. What are foods that are heart healthy? Select all that apply. (ch 27) a. broiled trout b. blueberries c. soy yogurt d. baked chicken leg e. white rice with butter

a. broiled trout b. blueberries c. soy yogurt

What are contributing causes to pericarditis? Select all that apply. (ch 28) a. chest trauma b. cardiac surgery c. common cold d. myocarditis e. tuberculosis

a. chest trauma b. cardiac surgery d. myocarditis e. tuberculosis

Primary or essential hypertension accounts for about 95% of all hypertension diagnoses with an unknown etiology. Secondary hypertension accompanies specific conditions that create hypertension as a result of tissue damage. Which condition contributes to secondary hypertension?

arterial vasoconstriction

The nurse is caring for a client with heart failure. What sign will lead the nurse to suspect right-sided and left-sided heart failure? (ch29 pg821-822)

ascites

When a patient is taking an immunosuppressant following heart transplantation, the nurse would determine which of the following as the MOST important intervention? (ch 28) a. Place the patient in an isolation room. b. Assess vital signs every 4 hours. c. Prevent exposure to potentially harmful agents such as fresh fruit. d. Educate the patient regarding signs and symptoms of infection.

b. Assess vital signs every 4 hours.

Which behavior indicates to the nurse that the cardiac client's level of anxiety has increased? (ch 28) a. Participates in support groups b. Avoids answering questions regarding status c. Verbalizes fears and concerns d. Discusses prognosis freely

b. Avoids answering questions regarding status

A nurse is caring for a client who had an aortic balloon valvuloplasty. The nurse should inspect the surgical insertion site closely for which complication(s)? (ch 28) a. Evisceration b. Bleeding and infection c. Bleeding and wound dehiscence d. Thrombosis and infection

b. Bleeding and infection

The nursing instructor is teaching their clinical group how to assess a client for congestive heart failure. How would the instructor teach the students to assess a client with congestive heart failure for nocturnal dyspnea? (ch 29) a. By measuring the client's abdominal girth b. By questioning how many pillows the client normally uses for sleep c. By observing the client's diet during the day d. By collecting the client's urine output

b. By questioning how many pillows the client normally uses for sleep

A client has been prescribed furosemide 80 mg twice daily. The asymptomatic client begins to have rare premature ventricular contractions followed by runs of bigeminy with stable signs. What action will the nurse perform next? (ch 29) a. Notify the health care provider. b. Check the client's potassium level. c. Administer potassium. d. Calculate the client's intake and output.

b. Check the client's potassium level.

A client had a percutaneous transluminal coronary angioplasty (PTCA). What medication will the nurse administer to prevent thrombus formation in the stent? (ch 27 pg 772) a. Metoprolol b. Clopidogrel c. Diltiazem d. Isosorbide mononitrate

b. Clopidogrel

A nurse is educating a client with coronary artery disease about nitroglycerin administration. The nurse tells the client that nitroglycerin has what actions? Select all that apply. (ch 27 pg 759) a. Decreases the urge to use tobacco b. Dilates blood vessels c. Reduces myocardial oxygen consumption d. Relieves pain e. Decreases ischemia

b. Dilates blood vessels c. Reduces myocardial oxygen consumption d. Relieves pain e. Decreases ischemia

A client with aortic regurgitation is admitted to the hospital. Which assessment findings would indicate left ventricular failure? a. Orthopnea, nausea, pedal edema b. Distended jugular veins, pedal edema, nausea c. Dyspnea, orthopnea, pulmonary edema d. Dyspnea, distended jugular veins, orthopnea

b. Distended jugular veins, pedal edema, nausea

The nurse is caring for a client following a coronary artery bypass graft (CABG). The nurse notes persistent oozing of bloody drainage from various puncture sites. The nurse anticipates that the physician will order which medication to neutralize the unfractionated heparin the client received? (ch 27 pg 776) a. Alteplase b. Protamine sulfate c. Clopidogrel d. Aspirin

b. Protamine sulfate

The nurse is caring for a client presenting to the emergency department (ED) reporting chest pain. Which electrocardiographic (ECG) finding would be most concerning to the nurse? (ch 27 pg 763) a. Isolated premature ventricular contractions (PVCs) b. ST elevation c. Sinus tachycardia d. Frequent premature atrial contractions (PACs)

b. ST elevation

Which symptom occurs in the client diagnosed with mitral regurgitation when pulmonary congestion occurs? (ch 28) a. Hypertension b. Shortness of breath c. A loud, blowing murmur d. Tachycardia

b. Shortness of breath

Which action will a public health nurse include when planning ways to decrease the incidence of rheumatic fever in the community? (ch 28) a. Encourage susceptible groups in the community to receive immunizations with streptococcal vaccine. b. Teach individuals in the community to seek medical treatment for streptococcal pharyngitis. c. Educate individuals in the community about the importance of monitoring temperature when infections occur. d. Provide prophylactic antibiotics to individuals with a family history of rheumatic fever.

b. Teach individuals in the community to seek medical treatment for streptococcal pharyngitis.

A nurse is preparing a teaching plan regarding biological tissue valve replacement. What is a disadvantage of this type of valve replacement? (ch 28) a. There is a low incidence of thromboembolism. b. The valve has to be replaced frequently. c. The patient must take lifelong anticoagulant therapy. d. The patient's infections are easier to treat.

b. The valve has to be replaced frequently.

The nurse is reevaluating a client 2 hours after a percutaneous transluminal coronary angioplasty (PTCA) procedure. Which assessment finding may indicate the client is experiencing a complication of the procedure? (ch 27) a. Potassium level of 4.0 mEq/L b. Urine output of 40 mL c. Heart rate of 100 bpm d. Dried blood at the puncture site

b. Urine output of 40 mL

The nurse is caring for a client after cardiac surgery. What laboratory result will lead the nurse to suspect possible renal failure? (ch 27 pg 772) a. a urine specific gravity reading of 1.021 b. a serum BUN of 70 mg/dL c. a serum creatinine of 1.0 mg/dL d. an hourly urine output of 50 to 70 mL

b. a serum BUN of 70 mg/dL

The nurse is removing a client's femoral sheath after cardiac catheterization. What medication will the nurse have available? (ch 27 pg 773) a. protamine sulfate b. atropine sulfate c. adenosine d. heparin

b. atropine sulfate

After percutaneous transluminal coronary angioplasty (PTCA), the nurse confirms that a client is experiencing bleeding from the femoral site. What will be the nurse's initial action? (ch 27 pg 772) a. Review the results of the latest blood cell count, especially the hemoglobin and hematocrit. b. Decrease anticoagulant or antiplatelet therapy. c. Apply manual pressure at the site of the insertion of the sheath. d. Notify the health care provider.

c. Apply manual pressure at the site of the insertion of the sheath.

Which medication is given to clients who are diagnosed with angina but are allergic to aspirin? (ch 27 pg 760) a. Diltiazem b. Felodipine c. Clopidogrel d. Amlodipine

c. Clopidogrel

A nurse is caring for a client after cardiac surgery. Upon assessment, the client appears restless and reports nausea and weakness. The client's ECG reveals peaked T waves. The nurse reviews the client's serum electrolytes, anticipating which abnormality? (ch 27 pg 782) a. Hyponatremia b. Hypercalcemia c. Hyperkalemia d. Hypomagnesemia

c. Hyperkalemia

A new surgical patient who has undergone a coronary artery bypass graft (CABG) is receiving opioids for pain control. The nurse must be alert to adverse effects of opioids. Which of the following effects would be important for the nurse to document? (ch 27 pg 786) a. Hyperactive bowel sounds b. Urinary incontinence c. Hypotension d. Hypertension

c. Hypotension

A client has had oral anticoagulation ordered. What should the nurse monitor for when the client is taking oral anticoagulation? (ch 27 pg 871) a. Urine output b. Vascular sites for bleeding c. Prothrombin time (PT) or international normalized ratio (INR) d. Hourly IV infusion

c. Prothrombin time (PT) or international normalized ratio (INR)

In the treatment of coronary artery disease (CAD), medications are often ordered to control blood pressure in the client. Which of the following is a primary purpose of using beta-adrenergic blockers in the nursing management of CAD? (ch 27 pg 759) a. To dilate coronary arteries b. To prevent angiotensin II conversion c. To decrease workload of the heart d. To decrease homocysteine levels

c. To decrease workload of the heart

One of the most common causes of mitral valve regurgitation in people living in developed countries is (ch 28) a. myxomatous changes. b. ischemic heart disease. c. ischemia of the left ventricle. d. rheumatic fever.

c. ischemia of left ventricle.

A client who has developed congestive heart failure must learn to make dietary adaptations. The client should avoid: (ch29 pg830)

canned peas.

The term for a diagnostic test that involves injection of a contrast media into the venous system through a dorsal vein in the foot is (ch30 pg849) a. air plethysmography. b. contrast phlebography. c. lymphangiography. d. lymphoscintigraphy.

contrast phlebography.

A client with a confirmed DVT is being discharged from the ED. Which client statement indicates that the client has received proper nursing instruction and understands how to manage this condition? (ch 28) a. "I need to ice my leg every 2 hours for about 20 minutes." b. "I should lie on my side with my knees bent when sleeping." c. "I should try not to drink too much during the daytime." d. "I need to do my leg exercises five times or more every hour."

d. "I need to do my leg exercises five times or more every hour."

The nurse is caring for a client with coronary artery disease (CAD). What is an appropriate nursing action when evaluating a client with CAD? (ch 27 pg 751) a. Assess the client's mental and emotional status. b. Assess the skin of the client. c. Assess for any kind of drug abuse. d. Assess the characteristics of chest pain.

d. Assess the characteristics of chest pain.

The nurse knows that a pacemaker is the treatment of choice for what cardiac dysrhythmia? (ch 26) a. Ventricular fibrillation b. Atrial flutter c. Supraventricular tachycardia d. Complete heart block

d. Complete heart block

Which of the following is inconsistent as a condition related to metabolic syndrome? (ch 27 pg 753) a. Dyslipidemia b. Insulin resistance c. Abdominal obesity d. Hypotension

d. Hypotension

A client has a myocardial infarction in the left ventricle and develops crackles bilaterally; 3-pillow orthopnea; an S3 heart sound; and a cough with pink, frothy sputum. The nurse obtains a pulse oximetry reading of 88%. What do these signs and symptoms indicate for this client? (ch 29) a. The development of right-sided heart failure b. The development of cor pulmonale c. The development of chronic obstructive pulmonary disease (COPD) d. The development of left-sided heart failure

d. The development of left-sided heart failure

A client asks the nurse if systolic heart failure will affect any other body function. What body system response correlates with systolic heart failure (HF)? (ch29 pg819)

decrease in renal perfusion

The nurse is providing care to a client with cardiogenic shock requiring a intra-aortic balloon pump (IABP). What is the therapeutic effect of the IABP therapy? (ch29 pg835)

decreased left ventricular workload

Nurses should implement measures to relieve emotional stress for clients with hypertension because the reduction of stress

decreases the production of neurotransmitters that constrict peripheral arterioles

A client has been having cardiac symptoms for several months and is seeing a cardiologist for diagnostics to determine the cause. How will the client's ejection fraction be measured? (ch29 pg819)

echocardiogram

A client is diagnosed with infective endocarditis. What laboratory values will the nurse assess? Select all that apply. (ch28 pg810-811)

elevated white blood cell (WBC) count elevated C-reactive protein elevated erythrocyte sedimentation rate (ESR)

A client reports headaches and "just not feeling right," which the client blames on ongoing sleep disturbances. Inspection reveals Janeway lesions on the bottoms of the client's feet. These symptoms may indicate: (ch28 pg810)

infective endocarditis

The nurse is caring for a client who has been diagnosed with aortic stenosis. The client is prescribed digoxin. What is the rationale for the medication? (ch28 pg825)

left ventricular dysfunction

The nurse is assigned to care for a client with heart failure. What medication does the nurse anticipate administering that will improve client symptoms as well as increase survival? (ch29 pg824-825)

lisinopril

A client diagnosed with heart failure presents with a temperature of 99.1° F, pulse 100 beats/minute, respirations 42 breaths/minute, BP 110/50 mm Hg; crackles in both lung bases; nausea; and pulse oximeter reading of 89%. Which finding indicates a need for immediate attention? (ch29 pg834)

lung congestion

A client has been rushed to the ED with pulmonary edema and is going to need oxygen immediately. Which oxygen delivery system should be used first? (ch 29)

mask

A client has been rushed to the ED with pulmonary edema and is going to need oxygen immediately. Which oxygen delivery system should be used first? (ch29 pg834)

mask

A client has been admitted to the cardiac step-down unit with acute pulmonary edema. Which symptoms would the nurse expect to find during assessment? (ch29 pg834)

moist, gurgling respirations

A client is prescribed digitalis medication. Which condition should the nurse closely monitor when caring for the client? (ch29 pg825)

nausea and vomiting

A nurse is caring for a client diagnosed with a mitral valve prolapse who is asymptomatic. What will nurse expect to be ordered for this client? (ch28 pg793)

no treatment

The nurse is administering digoxin to a client with heart failure. What laboratory value may predispose the client to digoxin toxicity? (ch29 pg826)

potassium level of 2.8 mEq/L

A client with a history of an anterior wall myocardial infarction is being transferred from the coronary care unit (CCU) to the cardiac step-down unit (CSU). While giving a report to the CSU nurse, the CCU nurse says, "His pulmonary artery wedge pressures have been in the high normal range." What additional assessment information would be important for the CSU nurse to obtain? (ch29 pg822)

pulmonary crackles

The nurse is caring for a client with abdominal aortic aneurysm (AAA). Which assessment finding is most likely to indicate a dissection of the aneurysm? (ch30 pg863) a. Severe back pain b. Hematemesis c. Rectal bleeding d. Hypertensive crisis

severe back pain

The nurse is teaching a client about chronic untreated hypertension. What complication will the nurse explain to the client? I

stroke

The nurse understands that asystole can be caused by several conditions. Select all that apply. (Chapter 26) A) Acidosis B) Hypothermia C) Alkalosis D) Hypoxia E) Hypovolemia

Acidosis, Hypothermia, Hypoxia, Hypovolemia

Which nursing diagnosis is most significant in planning the care for a client with Raynaud's disease? (ch30 pg867)

Acute Pain

A nurse is caring for a client with left-sided heart failure. During the nurse's assessment, the client is wheezing, restless, tachycardic, and has severe apprehension. The clients reports that these symptoms came on suddenly. The nurse knows that these are symptoms of what condition? (ch29 pg834)

Acute pulmonary edema

A client has been experiencing increasing shortness of breath and fatigue. The health care provider has ordered a diagnostic test in order to determine what type of heart failure the client is having. What diagnostic test does the nurse anticipate being ordered? (ch29 pg819)

An echocardiogram

The nurse explains to a patient that the primary cause of a varicose vein is: (ch30 pg878) a. Phlebothrombosis. b. An incompetent venous valve. c. Venospasm. d. Venous occlusion.

An incompetent venous valve

A nurse is changing a dressing on an arterial suture site. The site is red, with foul-smelling drainage. Based on these symptoms, the nurse is aware to monitor for which type of aneurysm? (ch30 pg862) a. Anastomotic b. False c. Dissecting d. Saccular

Anastomotic

The nurse is assessing a patient and feels a pulse with quick, sharp strokes that suddenly collapse. The nurse knows that this type of pulse is diagnostic for which disorder? (ch28 pg 795)

Aortic regurgitation

The nurse is discussing cardiac hemodynamics with a nursing student. The nurse explains preload to the student and then asks the student what nursing interventions might cause increased preload. Which response by the student indicates understanding? (ch29 pg834)

Application of antiembolic stockings

The nurse is auscultating the heart sounds of a patient with mitral stenosis. The pulse rhythm is weak and irregular. What rhythm does the nurse expect to see on the electrocardiogram (ECG)? (ch28 pg794)

Atrial fibrillation

The nurse understands that a client with which cardiac arrhythmia is most at risk for developing heart failure? (ch 29)

Atrial fibrillation

The nurse is teaching a group of clients with heart failure about how to decrease leg edema. What dietary advice will the nurse give to clients with severe heart failure? (ch29 pg832)

Avoid the intake of processed and commercially prepared foods.

A 77-year-old client presents to the local community center for a blood pressure (BP) screening; BP is recorded as 180/90 mm Hg. The client has a history of hypertension but currently is not taking the prescribed medications. Which question is most appropriate for the nurse to ask the client first? (Chapter 31) A) "What medications are you prescribed?" B) "Can you tell me the reasons you aren't taking your medications?" C) "Are you having trouble paying for your medications?" D) "Are you able to get to your pharmacy to pick up your medications?"

B) "Can you tell me the reasons you aren't taking your medications?"

A nurse has provided discharge instructions to a client who received an implantable cardioverter defibrillator (ICD). Which statement, made by the client, indicates the need for further teaching? (Chapter 26, Page 747) A) "I will document the date and time if my ICD fires." B) "I need to take a cardiopulmonary resuscitation (CPR) class now that I have an ICD." C) "I can play golf with my son in about 2 or 3 weeks." D) "I should tell close friends and family members that I have an ICD."

B) "I need to take a cardiopulmonary resuscitation (CPR) class now that I have an ICD."

A client, who has undergone a percutaneous transluminal coronary angioplasty (PTCA), has received discharge instructions. Which statement by the client would indicate the need for further teaching by the nurse? (Chapter 27, Page 772) A) "I should avoid taking a tub bath until my catheter site heals." B) "I should expect a low-grade fever and swelling at the site for the next week." C) "I should avoid prolonged sitting." D) "I should expect bruising at the catheter site for up to 3 weeks."

B) "I should expect a low-grade fever and swelling at the site for the next week."

The nurse is caring for a client who has had 25 mg of oral hydrochlorothiazide added to the medication regimen for the treatment of hypertension. Which instruction should the nurse give the client? (Chapter 31) A) "You may develop nasal congestion or depression while taking this medication." B) "Increase the amount of fruits and vegetables you eat." C) "You may drink alcohol while taking this medication." D) "Take this medication before going to bed."

B) "Increase the amount of fruits and vegetables you eat."

When providing discharge instructions for a client who has been prescribed sublingual nitroglycerin for angina, the nurse should plan to include which instructions? (Chapter 27, Page 759) A) "Only take one nitroglycerin tablet for each episode of angina." B) "See if rest relieves the chest pain before using the nitroglycerin." C) "Call 911 if you develop a headache following nitroglycerin use." D) "Place the nitroglycerin tablet between cheek and gum."

B) "See if rest relieves the chest pain before using the nitroglycerin."

A client is unconscious on arrival to the emergency department. The nurse in the emergency department identifies that the client has a permanent pacemaker due to which characteristic? (Chapter 26, Page 741) A) Scar on the chest B) "Spike" on the rhythm strip C) Quality of the pulse D) Vibration under the skin

B) "Spike" on the rhythm strip

The nurse is speaking with a client admitted with a dysrhythmia. The client asks the nurse to explain the "F waves" on the electrocardiogram. What is the nurse's best response? (Chapter 26, Page 727) A) "The F waves are normal parts of the heart conduction system" B) "The F waves are flutter waves representing atrial activity." C) "The F waves are most likely caused by the new medication." D) "The F waves are flutter waves representing ventricular activity."

B) "The F waves are flutter waves representing atrial activity."

A client admitted to the medical-surgical unit with a venous thromboembolism (VTE) is started on enoxaparin and warfarin. The client asks the nurse why two medications are needed. Which response by the nurse is accurate? (Chapter 30) A) "Enoxaparin will dissolve the clot, and warfarin will prevent any more clots from occurring." B) "The enoxaparin will work immediately, but the warfarin takes several days to achieve its full effect." C) "Administration of two anticoagulants decreases the risk of recurrent venous thrombosis." D) "Because of the potential for a pulmonary embolism, it is important for you to take at least two anticoagulants."

B) "The enoxaparin will work immediately, but the warfarin takes several days to achieve its full effect."

The client asks the nurse to explain what is meant by a ventricular bigeminy cardiac rhythm. What is the best response by the nurse? (Chapter 26, Page 729) A) "It is when the heart conduction is primarily from the atrioventricular node." B) "The rhythm has a normal beat, then a premature beat pattern." C) "The rhythm is regular but fast." D) "The heart rate is between 150 to 250 bpm."

B) "The rhythm has a normal beat, then a premature beat pattern."

The nurse is caring for a 32-year-old client admitted with a medical diagnosis of atrial fibrillation, related to "holiday heart" syndrome. A nursing student working with the nurse asks for information about "holiday heart" syndrome. Which is the best response by the nurse?(Chapter 26) A) "This is the association of heart dysrhythmias, especially atrial fibrillation, with extramarital sex." B) "This is the association of heart dysrhythmias, especially atrial fibrillation, with binge drinking." C) "This is the association of heart dysrhythmias, especially atrial fibrillation, with very heavy meals." D) "This is the association of heart dysrhythmias, especially atrial fibrillation, with physical activity the client is not used to."

B) "This is the association of heart dysrhythmias, especially atrial fibrillation, with binge drinking."

The nurse is working with a client with a new onset of atrial fibrillation during a three-month follow-up visit. The healthcare provider is planning a cardioversion, and the client asks the nurse why there is a wait for the treatment. What is the best response by the nurse? (Chapter 26, Page 724) A) "The doctor wants to see if your heart will switch back to its normal rhythm by itself." B) "Your atrial chambers may contain blood clots now, so you must take an anticoagulant for a few weeks before the cardioversion." C) "We have to allow your heart to rest for a few weeks before it is stressed by the cardioversion." D) "There is a long list of clients in line to be cardioverted."

B) "Your atrial chambers may contain blood clots now, so you must take an anticoagulant for a few weeks before the cardioversion."

After performing an ECG on an adult client, the nurse reports that the PR interval reflects normal sinus rhythm. What is the PR interval for a normal sinus rhythm? (Chapter 26, Page 718) A) 0.15 and 0.3 seconds B) 0.12 and 0.2 seconds C) 0.25 and 0.4 seconds D) 0.05 and 0.1 seconds

B) 0.12 and 0.2 seconds

Heparin therapy is usually considered therapeutic when the activated partial thromboplastin time (aPTT) is how many times higher than a normal value? (Chapter 30, Page 760) A) 0.5 to 1.5 B) 1.5 to 2.5 C) 2.5 to 3.5 D) 3.5 to 4.5

B) 1.5 to 2.5

A client who had a prosthetic valve replacement was taking warfarin to reduce the risk of postoperative thrombosis. The client visited the nurse at a clinic once a week. What INR level would alert the nurse to notify the health care provider? (Chapter 28, Page 801) A) 3.0 B) 3.8 C) 2.6 D) 3.4

B) 3.8

The nurse is explaining the DASH diet to a client diagnosed with hypertension. The client inquires about how many servings of fruit per day can be consumed on the diet. What is the nurse's best response? (Chapter 31) A) 2 or fewer servings per day B) 4 or 5 servings per day C) 7 or 8 servings per day D) 2 or 3 servings per day

B) 4 or 5 servings per day

To be effective, percutaneous transluminal coronary angioplasty (PTCA) must be performed within what time frame, beginning with arrival at the emergency department after diagnosis of myocardial infarction (MI)? (Chapter 27, Page 766) A) 30 minutes B) 60 minutes C) 9 days D) 6 to 12 months

B) 60 minutes

Which client with a venous stasis ulcer is a candidate for topical hyperbaric oxygen therapy? (Chapter 27, Page 877) A) A nonambulatory client B) A client with a chronic, nonhealing skin lesion C) A client whose ulcer includes necrotic tissue D) A client with an infected stasis ulcer

B) A client with a chronic, nonhealing skin lesion

The licensed practical nurse is setting up the room for a client arriving at the emergency department with ventricular arrhythmias. The nurse is most correct to place which of the following in the room for treatment? (Chapter 26, Page 738) A) A suction machine B) A defibrillator C) Cardioversion equipment D) An ECG machine

B) A defibrillator

Which assessment finding by the nurse is the most significant finding suggestive of aortic aneurysm? (Chapter 30) A) Nausea and vomiting B) Abdomen bruit C) High blood pressure D) Severe back pain

B) Abdomen bruit

The most common site of aneurysm formation is in the: (Chapter 30) A) Aortic arch, around the ascending and descending aorta B) Abdominal aorta, just below the renal arteries C) Ascending aorta, around the aortic arch D) Descending aorta, beyond the subclavian arteries

B) Abdominal aorta, just below the renal arteries

A client in the emergency department reports squeezing substernal pain that radiates to the left shoulder and jaw. The client also reports nausea, diaphoresis, and shortness of breath. What is the nurse's priority action? (Chapter 27, Page 765) A) Administer oxygen, attach a cardiac monitor, and notify the health care provider B) Administer oxygen, attach a cardiac monitor, take vital signs, and alert the cardiac catheterization team C) Complete the client's registration information, perform an electrocardiogram, gain I.V. access, and take vital signs D) Gain I.V. access, give sublingual nitroglycerin, and alert the cardiac catheterization team

B) Administer oxygen, attach a cardiac monitor, take vital signs, and alert the cardiac catheterization team

A nurse is teaching about risk factors that increase the probability of heart disease to a community group. Which risk factors will the nurse include in the discussion? Select all that apply. (Chapter 28) A) Body mass index (BMI) of 23 B) Age greater than 45 years for men C) Family history of coronary heart disease D) African-American descent E) Elevated C-reactive protein

B) Age greater than 45 years for men C) Family history of coronary heart disease D) African-American descent E) Elevated C-reactive protein

The nurse is caring for a client who is displaying a third-degree AV block on the EKG monitor. What is the priority nursing intervention for the client? (Chapter 26, Page 734) A) Assessing the client's blood pressure and heart rate frequently B) Alerting the healthcare provider of the third-degree heart block C) Identifying the client's code level status D) Maintaining the client's intravenous fluids

B) Alerting the healthcare provider of the third-degree heart block

Following a percutaneous coronary intervention (PCI), a client is returned to the nursing unit with large peripheral vascular access sheaths in place. The nurse understands that which method to induce hemostasis after sheath is contraindicated? (Chapter 27, Page 772-773) A) Application of a mechanical compression device B) Application of a sandbag to the area C) Direct manual pressure D) Application of a vascular closure device

B) Application of a sandbag to the area

The nurse understands it is important to promote adequate tissue perfusion following cardiac surgery. Which measures should the nurse complete to prevent the development of deep venous thrombosis (DVT) and possible pulmonary embolism (PE)? Select all that apply. (Chapter 27, Page 787) A) Place pillows in the popliteal space B) Apply antiembolism stockings C) Encourage the client to cross their legs D) Avoid elevating the knees on the bed E) Initiate passive exercises

B) Apply antiembolism stockings D) Avoid elevating the knees on the bed E) Initiate passive exercises

The nurse is beginning discharge teaching with a client diagnosed with a myocardial infarction (MI). The nurse will include teaching on what medications? Select all that apply. (Chapter 27) A) Morphine B) Aspirin C) Sildenafil D) Atorvastatin E) Enalapril

B) Aspirin D) Atorvastatin E) Enalapril

An older adult is postoperative day one, following a coronary artery bypass graft (CABG). The client's family members express concern to the nurse that the client is uncharacteristically confused. After reporting this change in status to the health care provider, what additional action should the nurse take? (Chapter 27, Page 784) A) Educate the family about how confusion is expected in older adults postoperatively B) Assess for factors that may be causing the client's delirium C) Document the early signs of dementia and ensure the client's safety D) Reorient the client to place and time

B) Assess for factors that may be causing the client's delirium

The nurse is caring for a client on a monitored telemetry unit. During morning assessment, the nurse notes abnormal ECG waves on the telemetry monitor. Which action would the nurse do first? (Chapter 25, Page 696) A) Call the physician with a report B) Assess the client C) Assess for mechanical dysfunction D) Reposition the client

B) Assess the client

A nurse provides evening care for a client wearing a continuous telemetry monitor. While the nurse is giving the client a back rub, the client's monitor alarm sounds and the nurse notes a flat line on the bedside monitor system. What is the nurse's first response? (Chapter 26, Page 729) A) Call a code and obtain the crash cart B) Assess the client and monitor leads C) Call for assistance and begin CPR D) Administer a pericardial thump

B) Assess the client and monitor leads

The nurse is monitoring a patient in the postanesthesia care unit (PACU) following a coronary artery bypass graft, observing a regular ventricular rate of 82 beats/min and "sawtooth" P waves with an atrial rate of approximately 300 beat/min. How does the nurse interpret this rhythm? (Chapter 26, Page 726) A) Atrial fibrillation B) Atrial flutter C) Ventricular tachycardia D) Ventricular fibrillation

B) Atrial flutter

The nurse knows that electrocardiogram (ECG) characteristics of atrial fibrillation include what? (Chapter 26, Page 722) A) Normal PR interval B) Atrial rate of 300 to 400 C) P wave resent before each QRS D) Regular rhythm

B) Atrial rate of 300 to 400

A nurse is caring for a client who's experiencing sinus bradycardia with a pulse rate of 40 beats/minute. The client's blood pressure is 80/50 mm Hg and the client reports dizziness. Which medication does the nurse anticipate administering to treat bradycardia? (Chapter 26) A) Dobutamine B) Atropine C) Lidocaine D) Amiodarone

B) Atropine

The nurse administers propranolol hydrochloride to a patient with a heart rate of 64 beats per minute (bpm). One hour later, the nurse observes the heart rate on the monitor to be 36 bpm. What medication should the nurse prepare to administer that is an antidote for the propranolol? (Chapter 27, Page 773) A) Digoxin B) Atropine C) Protamine sulfate D) Sodium nitroprusside

B) Atropine

The nurse is educating a patient with chronic venous insufficiency about prevention of complications related to the disorder. What should the nurse include in the information given to the patient? Select all that apply. (Chapter 30) A) Sit on the side of the bed and dangle the feet B) Avoid constricting garments C) Elevate the legs above the heart level for 30 minutes every 2 hours D) Sit as much as possible to rest the valves in the legs E) Sleep with the foot of the bed elevated about 6 inches

B) Avoid constricting garments C) Elevate the legs above the heart level for 30 minutes every 2 hours E) Sleep with the foot of the bed elevated about 6 inches

The nurse is caring for a client with Raynaud's disease. What is an important instruction for a client who is diagnosed with this disease to prevent an attack? (Chapter 27, Page 867) A) Report changes in the usual pattern of chest pain B) Avoid situations that contribute to ischemic episodes C) Avoid fatty foods and exercise D) Take over-the-counter decongestants

B) Avoid situations that contribute to ischemic episodes

Which postimplantation instruction must a nurse provide to a client with a permanent pacemaker? (Chapter 26) A) Keep the arm on the side of the pacemaker higher than the head B) Avoid sources of electrical interference C) Keep moving the arm on the side where the pacemaker is inserted D) Delay activities such as swimming and bowling for at least 3 weeks

B) Avoid sources of electrical interference

A nurse is checking laboratory values on a client who has crackles in the lower lobes, 2+ pitting edema, and dyspnea with minimal exertion. Which laboratory value does the nurse expect to be abnormal? (Chapter 25, Page 695) A) Potassium B) B-type natriuretic peptide (BNP) C) C-reactive protein (CRP) D) Platelet count

B) B-type natriuretic peptide (BNP)

A nurse provides morning care for a client in the intensive care unit (ICU). Suddenly, the bedside monitor shows ventricular fibrillation and the client becomes unresponsive. After calling for assistance, what action should the nurse take next? (Chapter 26, Page 731) A) Prepare for endotracheal intubation B) Begin cardiopulmonary resuscitation C) Provide electrical cardioversion D) Administer intravenous epinephrine

B) Begin cardiopulmonary resuscitation

The licensed practical nurse is monitoring the waveform pattern on the cardiac monitor of the client admitted following a myocardial infarction. The nurse notes that every other beat includes a premature ventricular contraction (PVC). The nurse notes which of the following in the permanent record? (Chapter 26) A) Multifocal PVCs B) Bigeminy C) Couplets D) R-on-T phenomenon

B) Bigeminy

Which is a diagnostic marker for inflammation of vascular endothelium? (Chapter 27) A) Triglyceride B) C-reactive protein (CRP) C) Low-density lipoprotein (LDL) D) High-density lipoprotein (HDL)

B) C-reactive protein (CRP)

The nurse cares for a client with an intra-arterial blood pressure monitoring device. The nurse recognizes the most preventable complication associated with hemodynamic monitoring includes which condition? (Chapter 25) A) Hemorrhage B) Catheter-related bloodstream infections C) Air embolism D) Pneumothorax

B) Catheter-related bloodstream infections

Which nursing intervention should a nurse perform when a client with cardiomyopathy receives a diuretic? (Chapter 28, Page 822) A) Administer oxygen B) Check regularly for dependent edema C) Maintain bed rest D) Allow unrestricted physical activity

B) Check regularly for dependent edema

A nurse evaluates a client and suspects pericarditis. What indicator is considered the most characteristic symptom of pericarditis? (Chapter 28, Page 814) A) Fatigue B) Chest pain C) Orthopnea D) Dyspnea

B) Chest pain

The nurse witnesses a client experiencing ventricular fibrillation. What is the nurse's priority action? (Chapter 26, Page 731) A) IV bolus of dobutamine B) Defibrillation C) Cardioversion D) IV bolus of atropine

B) Defibrillation

The nurse is observing the monitor of a patient with a first-degree atrioventricular (AV) block. What is the nurse aware characterizes this block? (Chapter 26, Page 732) A) An irregular rhythm B) Delayed conduction, producing a prolonged PR interval C) P waves hidden with the QRS complex D) A variable heart rate, usually fewer than 60 bpm

B) Delayed conduction, producing a prolonged PR interval

Frequently, what is the earliest symptom of left-sided heart failure? (ch29 pg822) A) Confusion B) Dyspnea on exertion C) Chest pain D) Anxiety

B) Dyspnea on exertion

The diagnosis of aortic regurgitation (AR) is confirmed by which of the following? (Chapter 28, Page 795) A) Myocardial biopsy B) Echocardiography C) Exercise stress testing D) Cardiac catheterization

B) Echocardiography

The nurse is teaching a client about hypertension and the effects on the left ventricle. What diagnostic test will the nurse describe? (Chapter 31) A) Fluorescein angiography B) Echocardiography C) Computed tomographic (CT) scan D) Positron emission tomography (PET) scan

B) Echocardiography

The nurse is caring for a client with atrial fibrillation. What procedure would be recommended if drug therapies did not control the dysrhythmia? (Chapter 26, Page 725) A) Maze procedure B) Elective cardioversion C) Defibrillation D) Pacemaker implantation

B) Elective cardioversion

A client with a history of mitral stenosis is admitted to the intensive care unit (ICU) with the abrupt onset of atrial fibrillation. The client's heart rate ranges from 120 to 140 bpm. The nurse recognizes that interventions are implemented to prevent the development of (Chapter 26) A) Renal failure B) Embolic stroke C) Myocardial infarction D) Heart failure

B) Embolic stroke

The nurse is caring for a client with coronary artery disease. What is the nurse's priority goal for the client? (Chapter 27, Page 767) A) Decrease anxiety B) Enhance myocardial oxygenation C) Administer sublingual nitroglycerin D) Educate the client about his symptoms

B) Enhance myocardial oxygenation

A patient is having an angiography to detect the presence of an aneurysm. After the contrast is administered by the interventionist, the patient begins to complain of nausea and difficulty breathing. What medication is a priority to administer at this time? (Chapter 30, Page 849) A) Metoprolol (Lopressor) B) Epinephrine C) Hydrocortisone (Solu-Cortef) D) Cimetidine (Tagamet)

B) Epinephrine

A nurse is caring for a client with aortic stenosis whose compensatory mechanisms of the heart have begun to fail. The nurse will monitor the client carefully for which initial symptoms? (Chapter 28, Page 795) A) Syncope, fever, vomiting B) Exertional dyspnea, orthopnea, pulmonary edema C) Nausea, vomiting, exertional fatigue D) Dizziness, nausea, diarrhea

B) Exertional dyspnea, orthopnea, pulmonary edema

Which symptom is an early warning sign of acute coronary syndrome (ACS) and heart failure (HF)? (Chapter 25, Page 680) A) Hypotension B) Fatigue C) Change in level of consciousness D) Weight gain

B) Fatigue

Which condition(s) indicates target organ damage from untreated/undertreated hypertension? Select all that apply. (Chapter 31) A) Hyperlipidemia B) Heart failure C) Stroke D) Retinal damage E) Diabetes

B) Heart failure C) Stroke D) Retinal damage

An adult client with a tentative diagnosis of infective endocarditis is admitted to an acute care facility. The medical history reveals diabetes mellitus, hypertension, and pernicious anemia. The client underwent an appendectomy 20 years earlier and an aortic valve replacement 2 years before this admission. What history finding is a major risk factor for infective endocarditis? (Chapter 28, Page 801) A) Race B) History of aortic valve replacement C) History of diabetes mellitus D) Age

B) History of aortic valve replacement

A patient with diabetes is being treated for a wound on the lower extremity that has been present for 30 days. What option for treatment is available to increase diffusion of oxygen to the hypoxic wound? (Chapter 30) A) Surgical debridement B) Hyperbaric oxygen C) Enzymatic debridement D) Vacuum-assisted closure device

B) Hyperbaric oxygen

When administering benazepril with spironolactone, the nurse should be aware that which electrolyte imbalance may occur? (Chapter 31) A) Hypercalcemia B) Hyperkalemia C) Hypokalemia D) Hypocalcemia

B) Hyperkalemia

Which is a modifiable risk factor for coronary artery disease (CAD)? (Chapter 28) A) Race B) Hypertension C) Gender D) Increasing age

B) Hypertension

During the auscultation of a client's heart sounds, the nurse notes an S4. The nurse recognizes that an S4 is associated with which condition? (Chapter 25, Page 691) A) Heart failure B) Hypertensive heart disease C) Turbulent blood flow D) Diseased heart valves

B) Hypertensive heart disease

The nurse is assessing a client with crackling breath sounds or pulmonary congestion. What is the cause of the congestion? (ch 29) A) Nocturia B) Inadequate cardiac output C) Hepatomegaly D) Ascites

B) Inadequate cardiac output

A nurse is teaching a client about mitral stenosis. What is the key teaching point regarding the disruption to the normal flow of blood through the heart due to mitral stenosis? (Chapter 28, Page 794) A) Inadequate left and right ventricle filling B) Increased resistance of a narrowed orifice between the left atrium and the left ventricle C) Pulmonary circulation congestion D) Atrial hypertrophy

B) Increased resistance of a narrowed orifice between the left atrium and the left ventricle

A client is treated in the intensive care unit (ICU) following an acute myocardial infarction (MI). During the nursing assessment, the client reports shortness of breath and chest pain. In addition, the client's blood pressure (BP) is 100/60 mm Hg with a heart rate (HR) of 53 bpm, and the electrocardiogram (ECG) tracing shows more P waves than QRS complexes. Which action should the nurse complete first? (Chapter 26, Page 734) A) Prepare for defibrillation B) Initiate transcutaneous pacing C) Administer 1 mg of IV atropine D) Obtain a 12-lead ECG

B) Initiate transcutaneous pacing

A patient has a persistent third-degree heart block and has had several periods of syncope. What priority treatment should the nurse anticipate for this patient? (Chapter 26) A) Insertion of an implantable cardioverter defibrillator (ICD) B) Insertion of a pacemaker C) Administration of atropine D) Administration of epinephrine

B) Insertion of a pacemaker

A client with a history of hypertension is receiving client education about structures that regulate arterial pressure. Which structure is a component of that process? (Chapter 31) A) Parasympathetic nervous system B) Kidneys C) Lungs D) Limbic system

B) Kidneys

A patient is 2 days postoperative after having a permanent pacemaker inserted. The nurse observes that the patient is having continuous hiccups as the patient states, "I thought this was normal." What does the nurse understand is occurring with this patient? (Chapter 26, Page 743) A) Fracture of the lead wire B) Lead wire dislodgement C) Faulty generator D) Sensitivity is too low

B) Lead wire dislodgement

After a physical examination, the provider diagnosed a patient with a grade 4 heart murmur. During assessment, the nurse expects to hear a murmur that is: (Chapter 25) A) Easily heard with no palpable thrill. B) Loud and may be associated with a thrill sound similar to (a purring cat). C) Very loud; can be heard with the stethoscope half-way off the chest. D) Quiet but readily heard.

B) Loud and may be associated with a thrill sound similar to (a purring cat).

With a severe degree of peripheral arterial insufficiency, leg pain during rest can be reduced by: (Chapter 30, Page 846) A) Elevating the limb over the heart level B) Lowering the limb so that it is dependent C) Massaging the limb after application of cold compresses D) Placing the limb in a plane horizontal to the body

B) Lowering the limb so that it is dependent

Which nursing intervention should a nurse perform to reduce cardiac workload in a client diagnosed with myocarditis? (Chapter 28, Page 813) A) Administer a prescribed antipyretic B) Maintain the client on bed rest C) Eliminate all phone calls and visitors D) Lower the client's head

B) Maintain the client on bed rest

The nurse is administering a beta blocker to a patient in order to decrease automaticity. Which medication will the nurse administer? (Chapter 25, Page 678) A) Diltiazem B) Metoprolol C) Propafenone D) Amiodarone

B) Metoprolol

The nurse is caring for a client in the cardiac intensive care unit (CICU) after a myocardial infarction (MI). Which drug will the nurse administer that will decrease contractility? (Chapter 25, Page 678) A) Dobutamine B) Metoprolol C) Digoxin D) Dopamine

B) Metoprolol

Which technique is used to surgically revascularize the myocardium? (Chapter 27, Page 755) A) Balloon bypass B) Minimally invasive direct coronary bypass C) Peripheral bypass D) Gastric bypass

B) Minimally invasive direct coronary bypass

After 2-hour onset of acute chest pain, the client is brought to the emergency department for evaluation. Elevation of which diagnostic findings would the nurse identify as suggestive of an acute myocardial infarction at this time? (Chapter 27, Page 764) A) Troponin I B) Myoglobin C) WBC (white blood cell) count D) C-reactive protein

B) Myoglobin

The nurse is performing wound care for a patient with a necrotic sacral wound. The prescribed treatment is isotonic saline solution with fine mesh gauze and a dry dressing to cover. What type of debridement is the nurse performing? (Chapter 30, Page 876) A) Selective debridement B) Nonselective debridement C) Enzymatic debridement D) Surgical debridement

B) Nonselective debridement

A nursing instructor is reviewing the parts of an EKG strip with a group of students. One student asks about the names of all the EKG cardiac complex parts. Which of the following items are considered a part of the cardiac complex on an EKG strip? Choose all that apply. (Chapter 26, Page 715) A) QRT wave B) P-R interval C) T wave D) S-Q segment E) P wave

B) P-R interval C) T wave E) P wave

The nurse is assessing a patient with severe hypertension. When performing a focused assessment of the eyes, what does the nurse understand may be observed related to the hypertension? (Chapter 31) A) Retinal detachment B) Papilledema C) Glaucoma D) Cataracts

B) Papilledema

The nurse is assessing a client with suspected post-pericardiotomy syndrome after cardiac surgery. What manifestation will alert the nurse to this syndrome? (Chapter 27, Page 787) A) Decreased erythrocyte sedimentation rate (ESR) B) Pericardial friction rub C) Hypothermia D) Decreased white blood cell (WBC) count

B) Pericardial friction rub

The nurse is reviewing the lab work of a client diagnosed with infective endocarditis. Which diagnostic study confirms the diagnosis? (Chapter 28, Page 810) A) Complete blood count B) Positive blood culture C) Serum cardiac antigens D) Immunosuppressant assay

B) Positive blood culture

While receiving heparin to treat a pulmonary embolus, a client passes bright red urine. What should the nurse do first? (Chapter 30) A) Decrease the heparin infusion rate B) Prepare to administer protamine sulfate C) Start an I.V. infusion of dextrose 5% in water (D5W) D) Monitor the partial thromboplastin time (PTT)

B) Prepare to administer protamine sulfate

When measuring the blood pressure in each arm of a healthy adult client, the nurse recognizes that which statement is true? (Chapter 31) A) Pressures may vary, with the higher pressure found in the left arm B) Pressures should not differ more than 5 mm Hg between arms C) Pressures must be equal in both arms D) Pressures may vary 10 mm Hg or more between arms

B) Pressures should not differ more than 5 mm Hg between arms

It is important for the nurse to encourage the client to rise slowly from a sitting or lying position because gradual changes in position: (Chapter 31) A) Help reduce the blood pressure to resupply oxygen to the brain B) Provide time for the heart to increase the rate of contraction to resupply oxygen to the brain C) Provide time for the heart to reduce the rate of contraction to resupply oxygen to the brain D) Help reduce the work required by the heart to resupply oxygen to the brain

B) Provide time for the heart to increase the rate of contraction to resupply oxygen to the brain

Which ECG waveform characterizes conduction of an electrical impulse through the left ventricle? (Chapter 26, Page 716) A) P wave B) QRS complex C) PR interval D) QT interval

B) QRS complex

A nurse is assigned to the medical intensive care unit. The nurse auscultates a water-hammer pulse. What will the sound resemble? (Chapter 28, Page 795) A) Low-pitched diastolic murmur at the apex B) Quick, sharp strokes that suddenly collapse C) High-pitched blowing sound at the apex D) Mitral click

B) Quick, sharp strokes that suddenly collapse

A female client is readmitted to the facility with a warm, tender, reddened area on her right calf. Which contributing factor should the nurse recognize as most important? (Chapter 30, Page 869) A) History of increased aspirin use B) Recent pelvic surgery C) An active daily walking program D) A history of diabetes mellitus

B) Recent pelvic surgery

A client with a recent history of atrial fibrillation has been prescribed warfarin. What action will the nurse take to confirm safe dosing? (Chapter 28, Page 703) A) Assess the client's radial pulse B) Review the client's international normalized ratio (INR) C) Assess the client's apical pulse D) Review the client's most recent warfarin blood levels

B) Review the client's international normalized ratio (INR)

The nurse is caring for a client with nursing diagnosis of ineffective tissue perfusion. Which area of the heart would the nurse anticipate being compromised? (Chapter 25) A) Aorta B) Right ventricle C) Right atrium D) Pulmonary artery

B) Right ventricle

A client is diagnosed with a dysrhythmia at a rate slower than 60 beats/minute with a regular interval between 0.12 and 0.20 seconds. What type of dysrhythmia does the client have? (Chapter 26) A) Heart block B) Sinus bradycardia C) None D) Atrial bradycardia

B) Sinus bradycardia

When the nurse observes that the client's heart rate increases during inspiration and decreases during expiration, the nurse reports that the client is demonstrating: (Chapter 26) A) Sinus tachycardia B) Sinus dysrhythmia C) Sinus bradycardia D) Normal sinus rhythm

B) Sinus dysrhythmia

The nurse is conducting a service project for a local elderly community group on the topic of hypertension. The nurse will relay that which risk factors and cardiovascular problems are related to hypertension? Select all that apply. (Chapter 31) A) Elevated high-density lipoprotein (HDL) cholesterol B) Smoking C) Decreased low-density lipoprotein (LDL) levels D) Age ≥65 in women E) Overweight/obesity

B) Smoking D) Age ≥65 in women E) Overweight/obesity

The nurse is reviewing the laboratory results for a client with heart failure. Which laboratory value will the nurse report to the health care provider? (Chapter 25) A) Potassium 3.9 mEq/L B) Sodium 148 mEq/L C) Calcium 9.8 mg/dL D) Magnesium 2.5 mg/dL

B) Sodium 148 mEq/L

A nurse is aware that the patient's heart rate is influenced by many factors. The nurse understands that the heart rate can be decreased by: (Chapter 25, Page 677) A) An excess level of thyroid hormone B) Stimulation of the vagus nerve C) An increased level of catecholamines D) Sympathetic nervous system stimulation

B) Stimulation of the vagus nerve

A patient tells the nurse, "I was straining to have a bowel movement and felt like I was going to faint. I took my pulse and it was so slow." What does the nurse understand occurred with this patient? (Chapter 25) A) The patient may have had a myocardial infarction B) The patient had a vagal response C) The patient was anxious about being constipated D) The patient may have an abdominal aortic aneurysm

B) The patient had a vagal response

The licensed practical nurse is co-assigned with a registered nurse in the care of a client admitted to the cardiac unit with chest pain. The licensed practical nurse is assessing the accuracy of the cardiac monitor, which notes a heart rate of 34 beats/minute. The client appears anxious and states not feeling well. The licensed practical nurse confirms the monitor reading. When consulting with the registered nurse, which of the following is anticipated? (Chapter 26, Page 718) A) The registered nurse stating to administer digoxin B) The registered nurse administering atropine sulfate intravenously C) The registered nurse stating to hold all medication until the pulse rate returns to 60 beats/minute D) The registered nurse stating to administer all medications except those which are cardiotonics

B) The registered nurse administering atropine sulfate intravenously

The client has just been diagnosed with a dysrhythmia. The client asks the nurse to explain normal sinus rhythm. What would the nurse explain is the characteristic of normal sinus rhythm? (Chapter 26, Page 719) A) The ventricles depolarize in 0.5 second or less B) The sinoatrial (SA) node initiates the impulse C) Impulse travels to the atrioventricular (AV) node in 0.15 to 0.5 second D) Heart rate between 60 and 150 beats/minute

B) The sinoatrial (SA) node initiates the impulse

The client has had biomarkers tested after reporting chest pain. Which diagnostic marker of myocardial infarction remains elevated for as long as 2 weeks? (Chapter 27, Page 764) A) Myoglobin B) Troponin C) Total creatine kinase D) CK-MB

B) Troponin

The nurse is assessing a client with severe angina pectoris and electrocardiogram changes in the emergency room. What is the most important cardiac marker for the client? (Chapter 27) A) Myoglobin B) Troponin C) Lactate dehydrogenase D) Creatine kinase

B) Troponin

Which statements are true when the nurse is measuring blood pressure (BP)? Select all that apply. (Chapter 31) A) Using a BP cuff that is too large will give a higher BP measurement B) Using a BP cuff that is too small will give a higher BP measurement C) The client's BP should be measured 1 hour before consuming alcohol D) The client should sit quietly while BP is being measured E) The client's arm should be positioned at the level of the heart

B) Using a BP cuff that is too small will give a higher BP measurement D) The client should sit quietly while BP is being measured E) The client's arm should be positioned at the level of the heart

The nurse knows that women and the elderly are at greater risk for a fatal myocardial event. Which factor is the primary contributor of this cause? (Chapter 27, Page 757) A) Chest pain is typical B) Vague symptoms C) Decreased sensation to pain D) Gender bias

B) Vague symptoms

A home health nurse is seeing an elderly male client for the first time. During the physical assessment of the skin on the lower legs, the nurse notes edema, brown pigmentation in the gaiter area, pedal pulses, and a few irregularly shaped ulcers around the ankles. From these findings, the nurse knows that the client has a problem with peripheral circulation. Which of the following does the nurse suspect? (Chapter 30, Page 874) A) Arterial insufficiency B) Venous insufficiency C) Neither venous nor arterial insufficiency D) Trauma

B) Venous insufficiency

A client is admitted to the emergency room with a blood pressure reading of 200/130 mm Hg. What are this client's therapeutic goals? Select all that apply. (chap 31 page 897) A) An immediate lowering of the blood pressure to a normotensive level within the first 30 minutes. B) Reduction of the mean blood pressure by 25% within the first hour. C)Achievement of a goal pressure of about 160/100 within 2 to 6 hours. D) Reduction to a target goal pressure over a period of days.

B)Reduction of the mean blood pressure by 25% within the first hour. C)Achievement of a goal pressure of about 160/100 within 2 to 6 hours. D)Reduction to a target goal pressure over a period of days.

A patient is seen in the emergency department (ED) with heart failure secondary to dilated cardiomyopathy. What key diagnostic test does the nurse assess to determine the severity of the patient's heart failure? (ch 29)

B-type natriuretic peptide (BNP)

A client with pulmonary edema has been admitted to the ICU. What would be the standard care for this client? (ch29 pg818)

BP and pulse measurements every 15 to 30 minutes

Which of the following assessment results is considered a major risk factor for PAD? (ch30 pg851) a. LDL of 100 mg/dL b. BP of 160/110 mm Hg c. Cholesterol of 200 mg/dL d. Triglyceride level of 150 mg/dL

BP of 160/110 mm Hg

A client with aortic stenosis is reluctant to have valve replacement surgery. A nurse is present when the health care provider talks to the client about a treatment that is less invasive than surgery which will likely relieve some of the client's symptoms. What treatment option has been discussed? (ch28 pg796)

Balloon percutaneous valvuloplasty

A client is recovering from surgical repair of a dissecting aortic aneurysm. Which assessment findings indicate possible bleeding or recurring dissection? (ch30 pg865) a. Urine output of 15 ml/hour and 2+ hematuria b. Blood pressure of 82/40 mm Hg and heart rate of 125 beats/minute c. Urine output of 150 ml/hour and heart rate of 45 beats/minute d. Blood pressure of 82/40 mm Hg and heart rate of 45 beats/minute

Blood pressure of 82/40 mm Hg and heart rate of 125 beats/minute

A client with angina pectoris must learn how to reduce risk factors that exacerbate this condition. When developing the client's care plan, which expected outcome should a nurse include? (Chapter 27, Page 762) A) "Client will verbalize an understanding of the need to call the physician if acute pain lasts more than 2 hours." B) "Client will verbalize the intention to avoid exercise." C) "Client will verbalize the intention to stop smoking." D) "Client will verbalize an understanding of the need to restrict dietary fat, fiber, and cholesterol."

C) "Client will verbalize the intention to stop smoking."

A 52-year-old female patient is going through menopause and asks the nurse about estrogen replacement for its cardioprotective benefits. What is the best response by the nurse? (Chapter 25, Page 678) A) "That's a great idea. You don't want to have a heart attack." B) "Current research determines that the replacement of estrogen will protect a woman after she goes into menopause." C) "Current evidence indicates that estrogen is ineffective as a cardioprotectant; estrogen is actually potentially harmful and is no longer a recommended therapy." D) "You need to research it and determine what you want to do."

C) "Current evidence indicates that estrogen is ineffective as a cardioprotectant; estrogen is actually potentially harmful and is no longer a recommended therapy."

The nurse reviews discharge instructions with a client who underwent a left groin cardiac catheterization 8 hours ago. Which instructions should the nurse include? (Chapter 25, Page 705) A) "You can take a tub bath or a shower when you get home." B) "Contact your primary care provider if you develop a temperature above 102°F." C) "Do not bend at the waist, strain, or lift heavy objects for the next 24 hours." D) "If any discharge occurs at the puncture site, call 911 immediately."

C) "Do not bend at the waist, strain, or lift heavy objects for the next 24 hours."

The nurse reviews discharge instructions with a client who underwent a left groin cardiac catheterization 8 hours ago. Which instructions should the nurse include? (Chapter 25, Page 705) A) "You can take a tub bath or a shower when you get home." B) "Contact your primary care provider if you develop a temperature above 102°F." C) "Do not bend at the waist, strain, or lift heavy objects for the next 24 hours." D) "If any discharge occurs at the puncture site, call 911 immediately."

C) "Do not bend at the waist, strain, or lift heavy objects for the next 24 hours."

The client is being prepared for echocardiography when he asks the nurse why he needs to have this test. What would be the nurse's best response? (Chapter 25, Page 702) A) "This test will find any congenital heart defects." B) "This test can tell us a lot about your heart." C) "Echocardiography is a way of determining the functioning of the left ventricle of your heart." D) "Echocardiography will tell your doctor if you have cancer of the heart."

C) "Echocardiography is a way of determining the functioning of the left ventricle of your heart."

A nurse is teaching the client about the causes of fast heart rates. What client statement indicates the client requires more teaching? (Chapter 26, Page 713) A) "If I take my metoprolol daily, I will be able to control my heart rate." B) "I will cut back on my smoking and drinking alcohol." C) "I will drink coffee with only two of my meals." D) "I will take my levothyroxine daily."

C) "I will drink coffee with only two of my meals."

A community health nurse teaches a group of older adults about modifiable risk factors that contribute to the development of peripheral arterial disease (PAD). The nurse knows that the teaching was effective based on which statement? (Chapter 30, Page 851) A) "Because my family is from Italy, I have a higher risk of developing peripheral arterial disease." B) "The older I get the higher my risk for peripheral arterial disease gets." C) "I will need to stop smoking because the nicotine causes less blood to flow to my hands and feet." D) "I will need to increase the amount of green leafy vegetables I eat to lower my cholesterol levels."

C) "I will need to stop smoking because the nicotine causes less blood to flow to my hands and feet."

Which client statement indicates a good understanding of the nutritional modifications needed to manage hypertension? (Chapter 31) A) "I should eliminate caffeine from my diet to lower my blood pressure." B) "A glass of red wine each day will lower my blood pressure." C) "Limiting my salt intake to 2 grams per day will improve my blood pressure." D) "If I include less fat in my diet, I'll lower my blood pressure."

C) "Limiting my salt intake to 2 grams per day will improve my blood pressure."

A client with an acute myocardial infarction is receiving nitroglycerin by continuous I.V. infusion. Which client statement indicates that this drug is producing its therapeutic effect? (Chapter 27) A) "I have a bad headache." B) "I feel a tingling sensation around my mouth." C) "My chest pain is decreasing." D) "My vision is blurred, so my blood pressure must be up."

C) "My chest pain is decreasing."

A client is ordered a nitroglycerine transdermal patch for treatment of CAD and asks the nurse why the patch is removed at bedtime. Which is the best response by the nurse? (Chapter 27, Page 759) A) "Nitroglycerine causes headaches, but removing the patch decreases the incidence." B) "You do not need the effects of nitroglycerine while you sleep." C) "Removing the patch at night prevents drug tolerance while keeping the benefits." D) "Contact dermatitis and skin irritations are common when the patch remains on all day."

C) "Removing the patch at night prevents drug tolerance while keeping the benefits."

A nurse and a nursing student are performing a physical assessment of a client with pericarditis. The client has an audible pericardial friction rub on auscultation. When leaving the room, the student asks the nurse what causes the sound. The nurse's best response is which of the following? (Chapter 28, Page 815) A) "The layers of the heart become loose from each other and rub together with each heart beat." B) "The lung surfaces lose their lubrication and rub against the myocardium with each heart beat." C) "The pericardial surfaces lose their lubricating fluid because of inflammation and rub against each other." D) "The great vessels rub against the pericardium with each heart beat."

C) "The pericardial surfaces lose their lubricating fluid because of inflammation and rub against each other."

The nurse is assisting a patient with peripheral arterial disease to ambulate in the hallway. What should the nurse include in the education of the patient during ambulation? (Chapter 30, Page 853) A) "As soon as you feel pain, we will go back and elevate your legs." B) "If you feel pain during the walk, keep walking until the end of the hallway is reached." C) "Walk to the point of pain, rest until the pain subsides, then resume ambulation." D) "If you feel any discomfort, stop and we will use a wheelchair to take you back to your room."

C) "Walk to the point of pain, rest until the pain subsides, then resume ambulation."

The nurse is caring for a client experiencing an acute MI (STEMI). The nurse anticipates the physician will prescribe alteplase. Before administering this medication, which question is most important for the nurse to ask the client? (Chapter 27, Chart 27-8, Page 766) A) "Do your parents have heart disease?" B) "What is your pain level on a scale of 1 to 10?" C) "What time did your chest pain start today?" D) "How many sublingual nitroglycerin tablets did you take?"

C) "What time did your chest pain start today?"

The nurse is caring for a client experiencing an acute MI (STEMI). The nurse anticipates the physician will prescribe alteplase. Before administering this medication, which question is most important for the nurse to ask the client? (Chapter 27, Page 766) A) "Do your parents have heart disease?" B) "What is your pain level on a scale of 1 to 10?" C) "What time did your chest pain start today?" D) "How many sublingual nitroglycerin tablets did you take?"

C) "What time did your chest pain start today?"

The nurse is caring for a client who is being discharged after insertion of a permanent pacemaker. The client, an avid tennis player, is scheduled to play in a tournament in 1 week. What is the best advice the nurse can give related to this activity? (Chapter 26, Page 742) A) "You should avoid tennis; basketball or football would be a good substitute." B) "You may resume all normal activity in 1 week; if you are used to playing tennis, you may proceed with this activity." C) "You will need to cancel this activity; you must restrict arm movement above your head for 2 weeks." D) "Cancel your tennis tournament and wait until fall, then try hockey; skating is much easier on pacemakers."

C) "You will need to cancel this activity; you must restrict arm movement above your head for 2 weeks."

The nurse knows that what PR interval presents a first-degree heart block? (Chapter 26) A) 0.18 seconds B) 0.14 seconds C) 0.24 seconds D) 0.16 seconds

C) 0.24 seconds

A client has just arrived in the ER with a possible myocardial infarction (MI). The electrocardiogram (ECG) should be obtained within which time frame of arrival to the ER? (Chapter 27, Page 763) A) 5 minutes B) 20 minutes C) 10 minutes D) 15 minutes

C) 10 minutes

A client presents to the emergency department reporting chest pain. Which order should the nurse complete first? (Chapter 28) A) 2 L oxygen via nasal cannula B) Aspirin 325 mg orally C) 12-lead ECG D) Troponin level

C) 12-lead ECG

A nurse is teaching a client who is newly diagnosed with hypertension and diabetes mellitus. What will the nurse specify about this client's target blood pressure? (Chapter 31) A) 125/85 or lower B) 150/95 or lower C) 130/80 or lower D) 145/95 or lower

C) 130/80 or lower

The nurse is reviewing the results of a total cholesterol level for a client who has been taking simvastatin. What results display the effectiveness of the medication? (Chapter 27, Table 27-2, Page 756) A) 280-300 mg/dL B) 210-240 mg/dL C) 160-190 mg/dL D) 250-275 mg/dL

C) 160-190 mg/dL

When caring for a patient who has started anticoagulant therapy with warfarin (Coumadin), the nurse knows that therapeutic benefits will not occur for: (Chapter 30) A) The first 24 hours B) At least 12 hours C) 3 to 5 days D) 2 to 3 days

C) 3 to 5 days

The nurse is teaching a client with suspected acute myocardial infarction about serial isoenzyme testing. When is it best to have isoenzyme creatinine kinase of myocardial muscle (CK-MB) tested? (Chapter 27, Page 764) A) 30 minutes to 1 hour after pain B) 2 to 3 hours after admission C) 4 to 6 hours after pain D) 12 to 18 hours after admission

C) 4 to 6 hours after pain

The following clients are in need of exercise electrocardiography. Which client would the nurse indicate as most appropriate for a drug-induced stress test? (Chapter 25, Page 699) A) A 48-year-old policemen with history of knee replacement 4 years ago B) A 68-year-old housewife with history of osteoporosis C) A 55-year-old recovering from a fall and broken femur D) A 72-year-old retired janitor obtaining a cardiac baseline

C) A 55-year-old recovering from a fall and broken femur

A nurse is discussing with a nursing student how to accurately measure blood pressure. What statement by the student indicates an understanding of the education? (Chapter 30) A) A cuff that is too small will give a false low blood pressure B) The size of the cuff does not matter as long as it fits snugly around the arm C) A cuff that is too small will give a false high blood pressure D) A cuff that is too large will give a false high blood pressure

C) A cuff that is too small will give a false high blood pressure

The nurse is obtaining a history from a client diagnosed with hypertrophic cardiomyopathy. What information obtained from the client is indicative of hypertrophic cardiomyopathy? (Chapter 28, Page 802) A) A history of alcoholism B) A history of amyloidosis C) A parent has the same disorder D) A long-standing history of hypertension

C) A parent has the same disorder

An older adult client visits the clinic for a blood pressure check. The client's hypertension is not well controlled, and a new blood pressure medicine is prescribed. What is important for the nurse to teach this client about blood pressure medicine? (Chapter 31) A) Take the medicine on an empty stomach B) There are no adverse effects from blood pressure medicine C) A possible adverse effect of blood pressure medicine is dizziness when you stand D) A severe drop in blood pressure is possible

C) A possible adverse effect of blood pressure medicine is dizziness when you stand

The nurse is caring for a ventilated client after coronary artery bypass graft surgery. What are the criterions for extubation for the client? Select all that apply. (Chapter 27, Page 775, 778) A) Labile vital signs B) Inability to speak C) Acceptable arterial blood gas values D) Adequate cough and gag reflexes E) Breathing without assistance of the ventilator

C) Acceptable arterial blood gas values D) Adequate cough and gag reflexes E) Breathing without assistance of the ventilator

The charge nurse was discussing with the nursing student that studies have been published that suggest inflammation increases the risk of heart disease. Which modifiable factor would the nursing student target in teaching clients about prevention of inflammation that can lead to atherosclerosis? (Chapter 27, Page 851) A) Avoid use of caffeine B) Encourage use of a multivitamin C) Addressing obesity D) Drink at least 2 liters of water a day

C) Addressing obesity

A client has had a 12-lead ECG completed as part of an annual physical examination. The nurse notes an abnormal Q wave on an otherwise unremarkable ECG. The nurse recognizes that this finding indicates: (Chapter 28) A) A cardiac dysrhythmia B) Variant angina C) An old MI D) An evolving MI

C) An old MI

A client with chronic arterial occlusive disease undergoes percutaneous transluminal coronary angioplasty (PTCA) for mechanical dilation of the right femoral artery. After the procedure, the client will be prescribed long-term administration of which drug? (Chapter 27, Page 760) A) Aspirin or acetaminophen B) Pentoxifylline or acetaminophen C) Aspirin or clopidogrel D) Penicillin V or erythromycin

C) Aspirin or clopidogrel

A client reports chest pain and heavy breathing when exercising or when stressed. Which is a priority nursing intervention for the client diagnosed with coronary artery disease? (Chapter 27, Page 761) A) It is not important to assess the client or to notify the physician B) Assess the client's physical history C) Assess chest pain and administer prescribed drugs and oxygen D) Assess blood pressure and administer aspirin

C) Assess chest pain and administer prescribed drugs and oxygen

A nurse is preparing a client for cardiac catheterization. The nurse knows that which nursing intervention must be provided when the client returns to the room after the procedure? (Chapter 25, Page 704) A) Withhold analgesics for at least 6 hours after the procedure B) Restrict fluids for 6 hours after the procedure C) Assess the puncture site frequently for hematoma formation or bleeding D) Inform the client that he or she may experience numbness or pain in the leg

C) Assess the puncture site frequently for hematoma formation or bleeding

You are presenting a workshop at the senior citizens center about how the changes of aging predisposes clients to vascular occlusive disorders. What would you name as the most common cause of peripheral arterial problems in the older adult? (Chapter 30, Page 850) A) Aneurysm B) Coronary thrombosis C) Atherosclerosis D) Raynaud's disease

C) Atherosclerosis

A client presents to the emergency department via ambulance with a heart rate of 210 beats/minute and a sawtooth waveform pattern per cardiac monitor. The nurse is most correct to alert the medical team of the presence of a client with which disorder? (Chapter 26, Page 726) A) Asystole B) Premature ventricular contraction C) Atrial flutter D) Ventricular fibrillation

C) Atrial flutter

The nurse is caring for a client with second-degree atrioventricular block, Type I with symptomatic bradycardia. What is the most likely medication the nurse will administer? (Chapter 26, Page 734) A) Nadolol B) Diltiazem C) Atropine sulfate D) Atenolol

C) Atropine sulfate

A client in the emergency department states, "I have always taken a morning walk, but lately my leg cramps and hurts after just a few minutes of walking. The pain goes away after I stop walking, though." Based on this statement, which priority assessment should the nurse complete? (Chapter 30, Page 845-846) A) Check for the presence of tortuous veins bilaterally on the legs B) Ask about any changes in skin color that occur in response to cold C) Attempt to palpate the dorsalis pedis and posterior tibial pulses D) Assess for unilateral swelling and tenderness of either leg

C) Attempt to palpate the dorsalis pedis and posterior tibial pulses

Which postimplantation instruction must a nurse provide to a client with a permanent pacemaker? (Chapter 26) A) Keep the arm on the side of the pacemaker higher than the head B) Delay activities such as swimming and bowling for at least 3 weeks C) Avoid sources of electrical interference D) Keep moving the arm on the side where the pacemaker is inserted

C) Avoid sources of electrical interference

The nurse is providing discharge education for a client going home after cardiac catheterization. What information is a priority to include when providing discharge education? (Chapter 25, Page 705) A) Expect increased bruising to appear at the site over the next several days B) Do not ambulate until the healthcare provider indicates it is appropriate C) Avoid tub baths, but shower as desired D) Returning to work immediately is okay

C) Avoid tub baths, but shower as desired

The nurse is performing a medication review of a client diagnosed with myocarditis. What medication may have precipitated the client to have myocarditis? (Chapter 28, Page 812) A) Furosemide B) Acetaminophen C) Azathioprine D) Ciprofloxacin

C) Azathioprine

The nurse is caring for a client with an elevated blood pressure and no previous history of hypertension. At 0900, the blood pressure was 158/90 mm Hg. At 0930, the blood pressure is 142/82 mm Hg. The nurse is most correct when relating the fall in blood pressure to which structure? (Chapter 25, Page 677) A) Chemoreceptors B) Sympathetic nerve fibers C) Baroreceptors D) Vagus nerve

C) Baroreceptors

A client is receiving nitroglycerin ointment to treat angina pectoris. The nurse evaluates the therapeutic effectiveness of this drug by assessing the client's response and checking for adverse effects. Which vital sign is most likely to reflect an adverse effect of nitroglycerin? (Chapter 27, Page 759) A) Pulse rate of 84 beats/minute B) Respiration 26 breaths/minute C) Blood pressure 84/52 mmHg D) Temperature of 100.2° F (37.9° C)

C) Blood pressure 84/52 mmHg

The nurse is caring for a patient diagnosed with pericarditis. What serious complication should this patient be monitored for? (Chapter 28, Page 814) A) Hypertension B) Left ventricular hypertrophy C) Cardiac tamponade D) Decreased venous pressure

C) Cardiac tamponade

The nurse is admitting a client to a telemetry unit with an atrial dysrhythmia. What symptoms will the nurse further evaluate? (Chapter 26) A) Leg pain B) Hypertension C) Chest pain D) Hypocarbia

C) Chest pain

Which medication is given to clients who are diagnosed with angina but are allergic to aspirin? (Chapter 27, Page 760) A) Amlodipine B) Felodipine C) Clopidogrel D) Diltiazem

C) Clopidogrel

The nurse assesses a patient with hip pain related to intermittent claudication. She knows that the area of arterial narrowing is the: (Chapter 30) A) Posterior tibial B) Common femoral artery C) Common iliac artery D) Anterior tibial

C) Common iliac artery

A nurse assessing a client who underwent cardiac catheterization finds the client lying flat on the bed. The client's temperature is 99.8° F (37.7° C). The client's blood pressure is 104/68 mm Hg. The client's pulse rate is 76 beats/minute. The nurse detects weak pulses in the leg distal to the puncture site. Skin on the leg is cool to the touch. The puncture site is dry, but swollen. What is the most appropriate action for the nurse to take? (Chapter 25, Page 686) A) Document findings and check the client again in 1 hour B) Slow the I.V. fluid to prevent any more swelling at the puncture site C) Contact the health care provider and report the findings D) Encourage the client to perform isometric leg exercise to improve circulation in the legs

C) Contact the health care provider and report the findings

A client is admitted for treatment of Prinzmetal angina. When developing this client's care plan, the nurse should keep in mind that this type of angina is a result of what trigger? (Chapter 27, Page 757) A) The same type of activity that caused previous angina episodes B) Activities that increase myocardial oxygen demand C) Coronary artery spasm D) An unpredictable amount of activity

C) Coronary artery spasm

The clinic nurse caring for a client with a cardiovascular disorder is performing an assessment of the client's pulse. Which of the following steps is involved in determining the pulse deficit? (Chapter 25, Page 688) A) Count the radial pulse for 20 to 25 seconds B) Calculate the palpated volume C) Count the heart rate at the apex D) Calculate the pauses between pulsations

C) Count the heart rate at the apex

The nurse is caring for an elderly client with left-sided heart failure. When auscultating lung sounds, which adventitious sound is expected? (Chapter 25, Page 822) A) Whistling B) Rhonchi C) Crackles D) Coarseness

C) Crackles

The nurse is caring for a client who was admitted to the telemetry unit with a diagnosis of "rule/out acute MI." The client's chest pain began 3 hours earlier. Which laboratory test would be most helpful in confirming the diagnosis of a current MI? (Chapter 27, Page 764) A) CK-MM B) Troponin C level C) Creatinine kinase-myoglobin (CK-MB) level D) Myoglobin level

C) Creatinine kinase-myoglobin (CK-MB) level

A nurse caring for a client recently admitted to the ICU observes the client coughing up large amounts of pink, frothy sputum. Lung auscultation reveals coarse crackles in the lower lobes bilaterally. Based on this assessment, the nurse recognizes this client is developing: (Chapter 29, Page 822) A) Tuberculosis B) Bilateral pneumonia C) Decompensated heart failure with pulmonary edema D) Acute exacerbation of chronic obstructive pulmonary disease

C) Decompensated heart failure with pulmonary edema

The nurse is administering a calcium channel blocker to a patient who has symptomatic sinus tachycardia at a rate of 132 bpm. What is the anticipated action of the drug for this patient? (Chapter 27, Page 759) A) Creates a positive inotropic effect B) Increases the atrioventricular node conduction C) Decreases the sinoatrial node automaticity D) Increases the heart rate

C) Decreases the sinoatrial node automaticity

The nurse recognizes which as being true of cardioversion? (Chapter 26, Page 737) A) Amount of voltage used should exceed 400 watts/second B) Electrical impulse can be discharged during the T wave C) Defibrillator should be set to deliver a shock during the QRS complex D) Defibrillator should be set in the non-synchronous mode so the nurse can hit the button at the right time

C) Defibrillator should be set to deliver a shock during the QRS complex

During an initial assessment, the nurse measures the client's apical pulse and compares it to the peripheral pulse. The difference between the two is known as pulse: (Chapter 25) A) Rhythm B) Volume C) Deficit D) Quality

C) Deficit

During an initial assessment, the nurse measures the client's apical pulse and compares it to the peripheral pulse. The difference between the two is known as pulse: (Chapter 25, Page 688) A) Volume B) Rhythm C) Deficit D) Quality

C) Deficit

The nurse is assessing a patient with a probable diagnosis of first-degree AV block. The nurse is aware that this dysrhythmia is evident on an ECG strip by what indication? (Chapter 26, Page 732) A) Variable heart rate, usually fewer than 90 bpm B) Irregular rhythm C) Delayed conduction, producing a prolonged PR interval D) P waves hidden within the QRS complex

C) Delayed conduction, producing a prolonged PR interval

For both outpatients and inpatients scheduled for diagnostic procedures of the cardiovascular system, the nurse performs a thorough initial assessment to establish accurate baseline data. Which data is necessary to collect if the client is experiencing chest pain? (Chapter 25, Page 679) A) Blood pressure in the left arm B) Pulse rate in upper extremities C) Description of the pain D) Sound of the apical pulses

C) Description of the pain

The nurse is performing an assessment of a client's peripheral pulses and indicates that the pulse quality is +1 on a scale of 0-4. What does this documented finding indicate? (Chapter 25) A) Full, easy to palpate, and cannot be obliterated with pressure B) Strong and bounding and may be abnormal C) Difficult to palpate and is obliterated with pressure D) Diminished, but cannot be obliterated with pressure

C) Difficult to palpate and is obliterated with pressure

A client who is newly diagnosed with hypertension is going to be starting antihypertensive medicine. What is one of the main things the client and the client's spouse should watch for? (Chapter 31) A) Persistent cough B) Tremor C) Dizziness D) Blurred vision

C) Dizziness

A patient is admitted with suspected cardiomyopathy. What diagnostic test will the nurse need to teach the client about for identification of this disease? (Chapter 28, Page 803) A) Serial enzyme studies B) Cardiac catheterization C) Echocardiogram D) Phonocardiogram

C) Echocardiogram

A client comes to the emergency department reporting chest pain. An electrocardiogram (ECG) reveals myocardial ischemia and an anterior-wall myocardial infarction (MI). Which ECG characteristic does the nurse expect to see? (Chapter 27, Page 763) A) Prolonged PR interval B) Absent Q wave C) Elevated ST segment D) Widened QRS complex

C) Elevated ST segment

To check for arterial insufficiency when a client is in a supine position, the nurse should elevate the extremity at a 45-degree angle and then have the client sit up. The nurse suspects arterial insufficiency if the assessment reveals: (Chapter 30, Page 845) A) Elevational rubor B) No rubor for 10 seconds after the maneuver C) Elevational pallor D) A 30-second filling time for the veins

C) Elevational pallor

You are monitoring the results of laboratory tests performed on a client admitted to the cardiac ICU with a diagnosis of myocardial infarction. Which test would you expect to show elevated levels? (Chapter 25, Page 693) A) RBC B) Platelets C) Enzymes D) WBC

C) Enzymes

Which area of the heart that is located at the third intercostal space to the left of the sternum? (Chapter 25) A) pulmonic area B) epigastric area C) Erb Point D) aortic area

C) Erb Point

Which area of the heart is located at the third intercostal (IC) space to the left of the sternum? (Chapter 25, Page 689) A) Aortic area B) Pulmonic area C) Erb point D) Epigastric area

C) Erb point

The nurse assesses a client returning from the post anesthesia unit with a new onset of sinus tachycardia with a heart rate of 138 beats per minute and a blood pressure of 128/80mmHg after elevating the head of the bed. What intervention does the nurse consider? (Chapter 26) A) Assessing blood glucose level B) Removing anti-embolism stockings C) Evaluating laboratory values D) Decreasing intravenous fluids

C) Evaluating laboratory values

A nurse teaches a client with angina pectoris that he or she needs to take up to three sublingual nitroglycerin tablets at 5-minute intervals and immediately notify the health care provider if chest pain doesn't subside within 15 minutes. What symptoms may the client experience after taking the nitroglycerin? (Chapter 27, Page 759) A) Nausea, vomiting, depression, fatigue, and impotence B) Sedation, nausea, vomiting, constipation, and respiratory depression C) Headache, hypotension, dizziness, and flushing D) Flushing, dizziness, headache, and pedal edema

C) Headache, hypotension, dizziness, and flushing

The nurse instructor is teaching a group of nursing students about adventitious heart sounds. The instructor explains that auscultation of the heart requires familiarization with normal and abnormal heart sounds. What would the instructor tell these students a ventricular gallop indicates in an adult? (Chapter 25) A) Hypertensive heart disease B) Normal functioning C) Heart failure D) Pericarditis

C) Heart failure

A client experiences a faster-than-normal heart rate when drinking more than two cups of coffee in the morning. What does the nurse identify on the electrocardigram as an indicator of sinus tachycardia? (Chapter 26, Page 719) A) Q wave of 0.04 seconds B) PR interval of 0.1 seconds C) Heart rate of 118 bpm D) QRS duration of 0.16 seconds

C) Heart rate of 118 bpm

A nurse is reviewing laboratory values for a client diagnosed with hyperlipidemia 6 months ago. Which results indicate that the client has been following a therapeutic regimen? (Chapter 25, Page 682) A) Total cholesterol level increases from 250 mg/dl to 275 mg/dl B) Low density lipoproteins (LDL) increase from 180 mg/dl to 190 mg/dl C) High density lipoproteins (HDL) increase from 25 mg/dl to 40 mg/dl D) Triglycerides increase from 225 mg/dl to 250 mg/dl

C) High density lipoproteins (HDL) increase from 25 mg/dl to 40 mg/dl

The nurse is discussing risk factors for developing CAD with a patient in the clinic. Which results would indicate that the patient is not at significant risk for the development of CAD? (Chapter 27, Page 753) A) Cholesterol, 280 mg/dL B) Low density lipoprotein (LDL), 160 mg/dL C) High-density lipoprotein (HDL), 80 mg/dL D) A ratio of LDL to HDL, 4.5 to 1.0

C) High-density lipoprotein (HDL), 80 mg/dL

A client admitted to the coronary care unit (CCU) diagnosed with a STEMI is anxious and fearful. Which medication will the nurse administer to relieve the client's anxiety and decrease cardiac workload? (Chapter 27, Page 765) A) Tenormin (atenolol) B) IV nitroglycerin C) IV morphine D) Norvasc (amlodipine)

C) IV morphine

A nurse enters a client's room and finds the client pulseless and unresponsive. What would be the treatment of choice for this client? (Chapter 26, Page 737) A) IV lidocaine B) Chemical cardioversion C) Immediate CPR D) Electric cardioversion

C) Immediate CPR

A client diagnosed with a myocardial infarction (MI) has begun an active rehabilitation program. The nurse recognizes which overall goal as a focus of rehabilitation for a client who has had an MI? (Chapter 27, Page 766) A) Prevention of another cardiac event B) Limit to the effects and progression of atherosclerosis C) Improved quality of life D) Return to work and the lifestyle experienced before the illness

C) Improved quality of life

It is appropriate for the nurse to recommend smoking cessation for clients with hypertension because nicotine: (Chapter 31) A) Decreases the heart rate, constricts arterioles, and reduces the heart's ability to eject blood B) Decreases circulating blood volume C) Increases the heart rate, constricts arterioles, and reduces the heart's ability to eject blood D) Increases the heart rate, constricts arterioles, and increases the heart's ability to eject blood

C) Increases the heart rate, constricts arterioles, and reduces the heart's ability to eject blood

A client has been recently placed on nitroglycerin. Which instruction by the nurse should be included in the client's teaching plan? (Chapter 27, Page 759) A) Instruct the client to renew the nitroglycerin supply every 3 months B) Instruct the client to place nitroglycerin tablets in a plastic pill box C) Instruct the client on side effects of flushing, throbbing headache, and tachycardia D) Instruct the client not to crush the tablet

C) Instruct the client on side effects of flushing, throbbing headache, and tachycardia

Which nursing intervention is required to prepare a client with cardiac dysrhythmia for an elective electrical cardioversion? (Chapter 26, Page 737) A) Administer digitalis and diuretics 24 hours before cardioversion B) Monitor blood pressure every 4 hours C) Instruct the client to restrict food and oral intake D) Facilitate CPR until the client is prepared for cardioversion

C) Instruct the client to restrict food and oral intake

The nurse is caring for a client who is diagnosed with an infarction of the posterior wall of the right atrium. Which assessment finding would the nurse anticipate relating to the infarction location? (Chapter 25) A) Cyanosis B) Respiratory compromise C) Irregular heart rate D) Chronic chest pain

C) Irregular heart rate

The staff educator is teaching a class in dysrhythmias. What statement is correct for defibrillation? (Chapter 26, Page 737) A) It is a scheduled procedure 1 to 10 days in advance B) The client is sedated before the procedure C) It is used to eliminate ventricular dysrhythmias D) It uses less electrical energy than cardioversion

C) It is used to eliminate ventricular dysrhythmias

The nurse is reviewing the morning laboratory test results for a client with cardiac problems. Which finding is a priority to report to the healthcare provider? (Chapter 25, Page 694) A) Na+ 140 mEq/L B) Ca++ 9 mg/dL C) K+ 3.1 mEq/L D) Mg++ 2 mEq/L

C) K+ 3.1 mEq/L

A client with no known history of peripheral vascular disease comes to the emergency department complaining of sudden onset of lower leg pain. Inspection and palpation reveal absent pulses; paresthesia; and a mottled, cyanotic, cold, and cadaverous left calf. While the physician determines the appropriate therapy, the nurse should: (Chapter 30, Page 854) A) Place a heating pad around the affected calf B) Elevate the affected leg as high as possible C) Keep the affected leg level or slightly dependent D) Shave the affected leg in anticipation of surgery

C) Keep the affected leg level or slightly dependent

Each chamber of the heart has a particular role in maintaining cellular oxygenation. Which chamber is responsible for receiving oxygenated blood from the lungs? (Chapter 25, Page 673) A) Left ventricle B) Right atrium C) Left atrium D) Right ventricle

C) Left atrium

A nurse is caring for a client with end-stage cardiomyopathy and the client's spouse asks the nurse to clarify one of the last treatment options available that the health care provider mentioned earlier. What option will the nurse most likely discuss? (Chapter 28, Page 804) A) Chordoplasty B) Annuloplasty C) Left ventricular assist device D) Open commissurotomy

C) Left ventricular assist device

A nurse is teaching a client who receives nitrates for the relief of chest pain. Which instruction should the nurse emphasize? (Chapter 27, Page 759) A) Repeat the dose of sublingual nitroglycerin every 15 minutes for three doses B) Store the drug in a cool, well-lit place C) Lie down or sit in a chair for 5 to 10 minutes after taking the drug D) Restrict alcohol intake to two drinks per day

C) Lie down or sit in a chair for 5 to 10 minutes after taking the drug

The nurse is developing a teaching plan for a client diagnosed with hypertension. What would be important for the nurse to emphasize as part of the plan of care? (Chapter 31) A) Limiting cigarette smoking to 1 pack a week B) Limiting alcohol to a can of beer to four times a day to thin the blood C) Limiting sodium intake in the diet D) Limiting activity to prevent over exertion

C) Limiting sodium intake in the diet

After a physical examination, the provider diagnosed a patient with a grade 4 heart murmur. When auscultating a murmur, what does the nurse expect to hear? (Chapter 25, Page 692) A) Easily heard with no palpable thrill B) Quiet but readily heard. C) Loud and may be associated with a thrill sound similar to (a purring cat). D) Very loud; can be heard with the stethoscope half-way off the chest.

C) Loud and may be associated with a thrill sound similar to (a purring cat)

Severe chest pain is reported by a client during an acute myocardial infarction. Which of the following is the most appropriate drug for the nurse to administer? (Chapter 27, Page 765) A) Isosorbide mononitrate (Isordil) B) Meperidine hydrochloride (Demerol) C) Morphine sulfate (Morphine) D) Nitroglycerin transdermal patch

C) Morphine sulfate (Morphine)

The nurse is admitting a client with an elevated creatine kinase-MB isoenzyme (CK-MB). What is the cause for the elevated isoenzymes? (Chapter 27, Page 764) A) Cerebral bleeding B) I.M. injection C) Myocardial necrosis D) Skeletal muscle damage due to a recent fall

C) Myocardial necrosis

Which risk factor is related to venous stasis for deep vein thrombosis (DVT) and pulmonary embolism (PE)? (Chapter 30, Page 868) A) Trauma B) Pacing wires C) Obesity D) Surgery

C) Obesity

Papilledema is a fairly common symptom of elevated blood pressure. The best way to detect this condition is through: (Chapter 31) A) An MRI B) Using a sphygmomanometer C) Ophthalmic examination D) Laboratory tests

C) Ophthalmic examination

The nurse is assessing vital signs in a patient with a permanent pacemaker. What should the nurse document about the pacemaker? (Chapter 26, Page 744) A) Model number B) Date and time of insertion C) Pacer rate D) Location of the generator

C) Pacer rate

The nurse recognizes that the treatment for a non-ST-elevation myocardial infarction (NSTEMI) differs from that for a STEMI, in that a STEMI is more frequently treated with (Chapter 27, Page 765) A) IV heparin B) Thrombolytics C) Percutaneous coronary intervention (PCI) D) IV nitroglycerin

C) Percutaneous coronary intervention (PCI)

The nurse recognizes that the treatment for a non-ST-elevation myocardial infarction (NSTEMI) differs from that for a STEMI, in that a STEMI is more frequently treated with: (Chapter 27, Page 765) A) IV nitroglycerin B) IV heparin C) Percutaneous coronary intervention (PCI) D) Thrombolytics

C) Percutaneous coronary intervention (PCI)

A nurse is caring for a client following an arterial vascular bypass graft in the leg. What should the nurse plan to assess over the next 24 hours? (Chapter 30) A) Ankle-arm indices every 12 hours B) Color of the leg every 4 hours C) Peripheral pulses every 15 minutes after surgery D) Blood pressure every 2 hours

C) Peripheral pulses every 15 minutes after surgery

The nurse is educating a patient at risk for atherosclerosis. What nonmodifiable risk factor does the nurse identify for the patient? (Chapter 25, Page 680) A) Obesity B) Stress C) Positive family history D) Hyperlipidemia

C) Positive family history

A patient in the recovery room after cardiac surgery begins to have extremity paresthesia, peaked T waves, and mental confusion. What type of electrolyte imbalance does the nurse suspect this patient is having? (Chapter 27, Page 782) A) Calcium B) Magnesium C) Potassium D) Sodium

C) Potassium

The nurse is caring for a client after cardiac surgery. What is the most immediate concern for the nurse? (Chapter 27, Page 781) A) Weight gain of 6 ounces B) Serum glucose of 124 mg/dL C) Potassium level of 6 mEq/L D) Bilateral rales and rhonchi

C) Potassium level of 6 mEq/L

The nurse observes an electrocardiogram (ECG) tracing on a cardiac monitor with a pattern in lead II as well as a bizarre, abnormal shape to the QRS complex. The nurse has likely observed which ventricular dysrhythmia? (Chapter 26, Page 729) A) Ventricular bigeminy B) Ventricular tachycardia C) Premature ventricular contraction D) Ventricular fibrillation

C) Premature ventricular contraction

When the balloon on the distal tip of a pulmonary artery catheter is inflated and the pressure is measured, the measurement obtained is referred to as the: (Chapter 25, Page 709) A) Central venous pressure B) Pulmonary artery pressure C) Pulmonary artery wedge pressure D) Cardiac output

C) Pulmonary artery wedge pressure

Following cardiac surgery, the nurse assesses the client for any common complication of hypovolemia. What significant indication of a complication should the nurse monitor? (Chapter 27, Page 776) A) Heart rate of 60 bpm B) Central venous pressure (CVP) reading of 8 mm Hg C) Pulmonary artery wedge pressure (PAWP) of 6 mm Hg D) Blood pressure reading of 130/95 mm Hg

C) Pulmonary artery wedge pressure (PAWP) of 6 mm Hg

The nurse is attempting to determine the ventricular rate and rhythm of a patient's telemetry strip. What should the nurse examine to determine this part of the analysis? (Chapter 26, Page 716) A) TP interval B) QT interval C) RR interval D) PP interval

C) RR interval

A patient is flying overseas for 1 week for business and packed antihypertensive medications in a suitcase. After arriving at the intended destination, the patient found that the luggage had been stolen. If the patient cannot take the medication, what condition becomes a concern? (Chapter 31) A) Angina B) Left ventricular hypertrophy C) Rebound hypertension D) Isolated systolic hypertension

C) Rebound hypertension

What does decreased pulse pressure reflect? (Chapter 25, Page 687) A) Tachycardia B) Reduced distensibility of the arteries C) Reduced stroke volume D) Elevated stroke volume

C) Reduced stroke volume

A triage team is assessing a client to determine if reported chest pain is a manifestation of angina pectoris or an MI. The nurse knows that a primary distinction of angina pain is? (Chapter 27, Page 758) A) Described as crushing and substernal B) Associated with nausea and vomiting C) Relieved by rest and nitroglycerin D) Accompanied by diaphoresis and dyspnea

C) Relieved by rest and nitroglycerin

The nurse is teaching a school community parent group about heart wellness. What risk factor is a common leading cause for mitral stenosis? (Chapter 28, Page 793) A) Infective endocarditis B) Marfan syndrome C) Rheumatic fever D) Dissecting aortic aneurysm

C) Rheumatic fever

It is important for a nurse to understand cardiac hemodynamics. For blood to flow from the right ventricle to the pulmonary artery, the following must occur: (Chapter 25, Page 677) A) The atrioventricular valves must open B) The pulmonic valve must be closed C) Right ventricular pressure must be higher than pulmonary arterial pressure D) Right ventricular pressure must decrease with systole

C) Right ventricular pressure must be higher than pulmonary arterial pressure

The nurse is assessing heart sounds in a patient with heart failure. An abnormal heart sound is detected early in diastole. How would the nurse document this? (Chapter 25, Page 691) A) S1 B) S2 C) S3 D) S4

C) S3

The nurse observes a client during an exercise stress test (bicycle). Which finding indicates a positive test and the need for further diagnostic testing? (Chapter 25, Page 699) A) Dizziness and leg cramping B) BP changes; 148/80 mm Hg to 166/90 mm Hg C) ST-segment changes on the ECG D) Heart rate changes; 78 bpm to 112 bpm

C) ST-segment changes on the ECG

The staff educator is presenting a class on cardiac dysrhythmias. How would the educator describe the characteristic pattern of the atrial waves in atrial flutter? (Chapter 26, Page 726) A) Square B) Triangular C) Sawtooth D) Sinusoidal

C) Sawtooth

A nurse is teaching a client with severe hypertension about the damage this condition can cause to the body. What system/organs will the nurse note are particularly targeted for damage due to severe hypertension? (Chapter 31) A) Musculoskeletal B) Gastrointestinal C) Sensory D) Integumentary

C) Sensory

A client with a myocardial infarction develops acute mitral valve regurgitation. The nurse knows to assess for which manifestation that would indicate that the client is developing pulmonary congestion? (Chapter 28, Page 792) A) A loud, blowing murmur B) Hypertension C) Shortness of breath D) Tachycardia

C) Shortness of breath

The electrical conduction system of the heart has several components, all of which are instrumental in maintaining polarization, depolarization, and repolarization of cardiac tissue. Which of the conductive structures is known as the pacemaker of the heart? (Chapter 25, Page 675) A) Atrioventricular node B) Bundle branches C) Sinoatrial node D) Bundle of His

C) Sinoatrial node

A client is diagnosed with a dysrhythmia at a rate slower than 60 beats/minute. What type of dysrhythmia does the client have? (Chapter 26, Page 718) A) Atrial bradycardia B) Heart block C) Sinus bradycardia D) None

C) Sinus bradycardia

The nurse is caring for a client who is prescribed medication for the treatment of hypertension. The nurse recognizes that which medication conserves potassium? (Chapter 31) A) Chlorthalidone B) Chlorothiazide C) Spironolactone D) Furosemide

C) Spironolactone

A patient with coronary artery disease (CAD) is having a cardiac catheterization. What indicator is present for the patient to have a coronary artery bypass graft (CABG)? (Chapter 27, Page 773) A) The patient has compromised left ventricular function B) The patient has had angina longer than 3 years C) The patient has at least a 70% occlusion of a major coronary artery D) The patient has an ejection fraction of 65%

C) The patient has at least a 70% occlusion of a major coronary artery

The patient has a heart rate of 72 bpm with a regular rhythm. Where does the nurse determine the impulse arises from? (Chapter 25, Page 675) A) The AV node B) The Purkinje fibers C) The sinoatrial node D) The ventricles

C) The sinoatrial node

When a client who has been diagnosed with angina pectoris reports experiencing chest pain more frequently, even at rest, that the period of pain is longer, and that it takes less stress for the pain to occur, the nurse recognizes that the client is describing which type of angina? (Chapter 27, Page 757) A) Intractable B) Variant C) Unstable D) Refractory

C) Unstable

Which term refers to preinfarction angina? (Chapter 27, Page 757) A) Stable angina B) Variant angina C) Unstable angina D) Silent angina

C) Unstable angina

A patient with hypertension has a newly diagnosed atrial fibrillation. What medication does the nurse anticipate administering to prevent the complication of atrial thrombi? (Chapter 26, Page 724) A) Adenosine B) Amiodarone C) Warfarin D) Atropine

C) Warfarin

Before a transesophageal echocardiogram, a nurse gives a client an oral topical anesthetic spray. When the client returns from the procedure, the nurse observes no active gag reflex. What nursing action is a priority? (Chapter 25, Page 703) A) Introduce a nasogastric (NG) tube B) Insert an oral airway C) Withhold food and fluids D) Position the client on his side

C) Withhold food and fluids

A client was chopping firewood and experienced a heaviness in the chest and dyspnea. The client arrives in the emergency department four hours after the heaviness and the health care provider diagnoses an anterior myocardial infarction (MI). What orders will the nurse anticipate? (ch 27 pg 765) A. morphine administration, stress testing, and admission to the cardiac care unit B. streptokinase, aspirin, and morphine administration C. sublingual nitroglycerin, tissue plasminogen activator (tPA), and telemetry D. serial liver enzyme testing, telemetry, and a lidocaine infusion

C. sublingual nitroglycerin, tissue plasminogen activator (tPA), and telemetry

An older adult client has newly diagnosed stage 2 hypertension. The health care provider has prescribed the client hydrochlorothiazide and enalapril. What will the nurse be sure to include in educating this client?

Change positions (lying or sitting to standing) slowly.

A client has been prescribed furosemide 80 mg twice daily. The asymptomatic client begins to have rare premature ventricular contractions followed by runs of bigeminy with stable signs. What action will the nurse perform next? (ch29, pg824)

Check the client's potassium level.

The nurse is caring for a client in the hospital with chronic heart failure who has marked limitations in his physical activity. The client is comfortable when resting in the bed or chair, but when ambulating in the room or hall, the client becomes short of breath and fatigued easily. What type of heart failure is this considered according to the New York Heart Association (NYHA)? (ch29 pg819)

Class III (Moderate)

The nurse determines that a patient has a characteristic symptom of pericarditis. What symptom does the nurse recognize as significant for this diagnosis? (ch28 pg814)

Constant chest pain

A patient has had a successful heart transplant for end-stage heart disease. What immunosuppressant will be necessary for this patient to take to prevent rejection? (ch28 pg804)

Cyclosporine

A client comes to the health care provider's office for a follow-up visit 4 weeks after suffering a myocardial infarction (MI). Which evaluation statement suggests that the client needs more instruction? (Chapter 27, Page 770) A) "Client performs relaxation exercises three times per day to reduce stress." B) "Client's 24-hour dietary recall reveals low intake of fat and cholesterol." C) "Client verbalizes an understanding of the need to seek emergency help if heart rate increases markedly while at rest." D) "Client walks 4 miles in 1 hour every day."

D) "Client walks 4 miles in 1 hour every day."

A nurse is caring for a client who has hypertension and diabetes mellitus. The client's blood pressure this morning was 150/92 mm Hg. When the client asks the nurse what his or her blood pressure should be, what is the nurse's most appropriate response? (Chapter 31) A) "The current recommendation is for everyone to have blood pressure of 140/90 mm Hg or lower." B) "Your blood pressure is fine. Just keep doing what you're doing." C) "The lower the better. Blood pressure of 130/80 mm Hg is best for everyone." D) "Clients with diabetes should have a lower blood pressure goal. You should strive for 120/80 mm Hg."

D) "Clients with diabetes should have a lower blood pressure goal. You should strive for 120/80 mm Hg."

The nurse obtains a health history from a client with a prosthetic heart valve and new symptoms of infective endocarditis. Which question by the nurse is most appropriate to ask? (Chapter 28, Page 810) A) "Do you live with any domesticated animals in your home?" B) "Do you have a family history of endocarditis?" C) "Have you recently vacationed outside of the United States?" D) "Have you been to the dentist recently?"

D) "Have you been to the dentist recently?"

The nurse is instructing on home care after placement of an automatic implanted cardioverter defibrillator (AICD). Which statement, made by the client, needs clarification by the nurse? (Chapter 26, Page 747) A) "I should opt for a hand search at the airport instead of metal detector scan." B) "I can continue to work with my power tools." C) "I need to notify my cardiologist if I feel frequent kicks to the chest." D) "I need to stay away from microwaves."

D) "I need to stay away from microwaves."

The nurse is caring for a client newly diagnosed with hypertension. Which statement by the client indicates the need for further teaching? (Chapter 31) A) "I think I'm going to sign up for a yoga class twice a week to help reduce my stress." B) "When getting up from bed, I will sit for a short period before standing up." C) "I will consult a dietician to help get my weight under control." D) "If I take my blood pressure and it is normal, I don't have to take my blood pressure pills."

D) "If I take my blood pressure and it is normal, I don't have to take my blood pressure pills."

A nurse is caring for a client who is recovering from a myocardial infarction (MI). The cardiologist refers the client to cardiac rehabilitation. Which statement by the client indicates an understanding of cardiac rehabilitation? (Chapter 27, Page 766) A) "I'll get rest during these rehabilitation classes. All I have to do is sit and listen to the instructor." B) "When I finish the rehabilitation program I'll never have to worry about heart trouble again." C) "I won't be able to jog again even with rehabilitation." D) "Rehabilitation will help me function as well as I physically can."

D) "Rehabilitation will help me function as well as I physically can."

When assessing a client who reports recent chest pain, the nurse obtains a thorough history. Which client statement most strongly suggests angina pectoris? (Chapter 27, Page 752) A) "The pain lasted about 45 minutes." B) "The pain resolved after I ate a sandwich." C) "The pain got worse when I took a deep breath." D) "The pain occurred while I was mowing the lawn."

D) "The pain occurred while I was mowing the lawn."

The nurse is defibrillating a patient in ventricular fibrillation with paddles on a monophasic defibrillator. How much paddle pressure should the nurse apply when defibrillating? (Chapter 26) A) 5 to 10 lbs B) 10 to 15 lbs C) 15 to 20 lbs D) 20 to 25 lbs

D) 20 to 25 lbs

A patient is being treated for hypertensive emergency. When treating this patient, the priority goal is to lower the mean blood pressure (BP) by up to which percentage in the first hour? (Chapter 31) A) 40% B) 35% C) 45% D) 25%

D) 25%

Approximately what percentage of the arterial lumen must be obstructed before intermittent claudication is experienced? (Chapter 30, Page 845) A) 20 B) 30 C) 40 D) 50

D) 50

In a client with a bypass graft, the distal outflow vessel must have at least what percentage patency for the graft to remain patent? (Chapter 30) A) 20 B) 30 C) 40 D) 50

D) 50

Which of the following client scenarios would be correct for the nurse to identify as a client with secondary hypertension? (Chapter 31) A) A client of advanced age B) A client experiencing depression C) A client with excessive alcohol intake D) A client diagnosed with kidney disease

D) A client diagnosed with kidney disease

The nurse is assigned the following client assignment on the clinical unit. For which client does the nurse anticipate cardioversion as a possible medical treatment? (Chapter 26, Page 737) A) A new myocardial infarction client B) A client with poor kidney perfusion C) A client with third-degree heart block D) A client with atrial dysrhythmias

D) A client with atrial dysrhythmias

The nurse is assisting the client to manage the cardiovascular risk factors of hyperlipidemia and hypertension. The client asks the nurse what type of a diet would be best to follow. What is the best response by the nurse? (Chapter 25) A) A diet with restricted fruits and fluids B) A diet with high sodium, fruits, vegetables C) A diet high in trans fats and potassium D) A diet low in sodium, fat, cholesterol

D) A diet low in sodium, fat, cholesterol

The nurse is explaining the cause of angina pain to a client. What will the nurse say most directly caused the pain? (Chapter 27, Page 751) A) Incomplete blockage of a major coronary artery B) A destroyed part of the heart muscle C) Complete closure of an artery D) A lack of oxygen in the heart muscle cells

D) A lack of oxygen in the heart muscle cells

A client is diagnosed with pericarditis. What symptom will be the nurse's priority for treatment? (Chapter 28, Page 814) A) Denial B) Anxiety C) Fatigue D) Acute pain

D) Acute pain

A client has a medical diagnosis of an advanced first-degree atrioventricular block and is symptomatic. What initial treatment will the nurse be prepared to complete? (Chapter 26, Page 734) A) Administer an IV bolus of furosemide B) Prepare the client for a cardioversion C) Prepare client for a cardiac catheterization D) Administer an IV bolus of atropine

D) Administer an IV bolus of atropine

A client is scheduled for an elective electrical cardioversion for a sustained dysrhythmia lasting for 24 hours. Which intervention is necessary for the nurse to implement prior to the procedure? (Chapter 26, Page 724) A) Administer the prescribed digitalis to the client before the scheduled procedure B) Administer anticoagulant therapy as prescribed prior to the procedure C) Maintain the client on NPO status for 8 hours prior to the procedure D) Administer moderate sedation IV and analgesic medication as prescribed

D) Administer moderate sedation IV and analgesic medication as prescribed

The nurse is caring for a client who is displaying a third-degree AV block on the EKG monitor. What is the priority nursing intervention for the client? (Chapter 26, Page 734) A) Maintaining intravenous fluids B) Identifying a code-level status C) Assessing blood pressure and heart rate frequently D) Alerting the healthcare provider of the third-degree heart block

D) Alerting the healthcare provider of the third-degree heart block

A total artificial heart (TAH) is an electrically powered pump that circulates blood into the pulmonary artery and the aorta, thus replacing the functions of both the right and left ventricles. What makes it different from an LVAD? (Chapter 28, Page 807) A) It never needs batteries B) It is designed for extremely active patients C) It is specifically designed for long-term use D) An LVAD only supports a failing left ventricle

D) An LVAD only supports a failing left ventricle

To assess the dorsalis pedis artery, the nurse would use the tips of three fingers and apply light pressure to the: (Chapter 30, Page 846) A) Inside of the ankle just above the heel B) Exterior surface of the foot near the heel C) Outside of the foot just below the heel D) Anterior surface of the foot near the ankle joint

D) Anterior surface of the foot near the ankle joint

While assessing a patient with pericarditis, the nurse cannot auscultate a friction rub. Which action should the nurse implement? (Chapter 28, Page 815) A) Document that the pericarditis has resolved B) Notify the health care provider C) Prepare to insert a unilateral chest tube D) Ask the patient to lean forward and listen again

D) Ask the patient to lean forward and listen again

A client's electrocardiogram (ECG) tracing reveals a atrial rate between 250 and 400, with saw-toothed P waves. The nurse correctly identifies this dysrhythmia as: (Chapter 26, Page 727) A) Ventricular fibrillation B) Ventricular tachycardia C) Atrial fibrillation D) Atrial flutter

D) Atrial flutter

Which describes a valve used in replacement surgery that is made from the client's own heart valve? (Chapter 28) A) Xenograft B) Allograft C) Homograft D) Autograft

D) Autograft

A nurse is caring for a client who has had an automatic cardiac defibrillator implanted. What instructions should the nurse provide to the client? (Chapter 26) A) Avoid driving for at least 3 months B) Avoid using microwave ovens C) Use digital cellular telephones D) Avoid devices with a magnetic field

D) Avoid devices with a magnetic field

The nurse is providing discharge instructions to a client after a permanent pacemaker insertion. Which safety precaution will the nurse communicate to the client? (Chapter 26, Page 743) A) Stay at least 5 feet away from microwave ovens B) Never engage in activities that require vigorous arm and shoulder movement C) Avoid going through airport metal detectors D) Avoid undergoing magnetic resonance imaging (MRI)

D) Avoid undergoing magnetic resonance imaging (MRI)

A patient has had cardiac surgery and is being monitored in the intensive care unit (ICU). What complication should the nurse monitor for that is associated with an alteration in preload? (Chapter 27, Page 776) A) Elevated central venous pressure B) Hypothermia C) Hypertension D) Cardiac tamponade

D) Cardiac tamponade

A nurse is caring for a client with a history of cardiac disease and type 2 diabetes. The nurse is closely monitoring the client's blood glucose level. Which medication is the client most likely taking? (Chapter 26, Page 721) A) Diltiazem B) Procainamide C) Amiodarone D) Carvedilol

D) Carvedilol

The nurse is administering medications on a medical-surgical unit. A client is ordered to receive 40 mg oral nadolol for the treatment of hypertension. Before administering the medication, the nurse should: (Chapter 31) A) Weigh the client B) Check the client's serum K+ level C) Check the client's urine output D) Check the client's heart rate

D) Check the client's heart rate

The nurse is caring for a client who has just returned from the cardiac catheterization laboratory following a coronary angioplasty. What is the nurse's priority assessment? (Chapter 25) A) Check the client's tolerance to ambulation to the bathroom. B) Assess when the client last had a bowel movement. C) Assess pupils for size and reactivity. D) Check the temperature, color, and capillary refill of the affected extremity.

D) Check the temperature, color, and capillary refill of the affected extremity.

A nurse is caring for a client who has been admitted to have a cardioverter defibrillator implanted. The nurse knows that implanted cardioverter defibrillators are used in which clients? (Chapter 26, Page 744-745) A) Clients with recurrent life-threatening bradycardias B) Clients with sinus tachycardia C) Clients with ventricular bradycardia D) Clients with recurrent life-threatening tachydysrhythmias

D) Clients with recurrent life-threatening tachydysrhythmias

A client has severe coronary artery disease (CAD) and hypertension. Which medication order should the nurse consult with the health care provider about that is contraindicated for a client with severe CAD? (Chapter 31) A) Methyldopa B) Amiloride C) Bumetanide D) Clonidine

D) Clonidine

A health care provider wants a cross-sectional image of the abdomen to evaluate the degree of stenosis in a patient's left common iliac artery. The nurse knows to prepare the patient for which of the following? (Chapter 30, Page 865) A) Magnetic resonance angiography (MRA) B) Angiography C) Doppler ultrasound D) Computed tomography angiography (CTA)

D) Computed tomography angiography (CTA)

A client is hospitalized for repair of an abdominal aortic aneurysm. The nurse must be alert for signs and symptoms of aneurysm rupture and thus looks for which of the following? (Chapter 30) A) Constant, intense headache and falling blood pressure B) Higher than normal blood pressure and falling hematocrit C) Slow heart rate and high blood pressure D) Constant, intense back pain and falling blood pressure

D) Constant, intense back pain and falling blood pressure

A nurse is caring for a client with a central venous pressure (CVP) of 4 mm Hg. Which nursing intervention is appropriate? (Chapter 25, Page 706) A) Call the health care provider and obtain an order for a fluid bolus B) Re-zero the equipment and take another reading C) Call the physician and obtain an order for a diuretic D) Continue to monitor the client as ordered

D) Continue to monitor the client as ordered

A client is prescribed quinine for treatment of moderate depression of depolarization. Which sign indicates the drugs' effectiveness in the client's treatment? (Chapter 26) A) Hypertension with IV administration. B) Decreased QRS duration of about 40% from baseline C) Shortened QT segment D) Decrease in cardiac contractility

D) Decrease in cardiac contractility

A nurse is caring for a client who experienced an MI. The client is ordered to received metoprolol. The nurse understands that this medication has which therapeutic effect? (Chapter 27, Page 759) A) Decreases cholesterol level B) Decreases platelet aggregation C) Increases cardiac output D) Decreases resting heart rate

D) Decreases resting heart rate

The most important reason for a nurse to encourage a client with peripheral vascular disease to initiate a walking program is that this form of exercise: (Chapter 30, Page 853) A) Reduces stress B) Aids in weight reduction C) Increases high-density lipoprotein (HDL) level D) Decreases venous congestion

D) Decreases venous congestion

As the clinic nurse caring for a client with varicose veins, what is an appropriate nursing action for this client? (Chapter 30) A) Assess for skin integrity B) Assess for the sites of bleeding C) Demonstrate how to self-administer IV infusions D) Demonstrate how to apply and remove elastic support stockings

D) Demonstrate how to apply and remove elastic support stockings

Elective cardioversion is similar to defibrillation except that the electrical stimulation waits to discharge until an R wave appears. What does this prevent? (Chapter 26, Page 737) A) Disrupting the heart during the critical period of atrial repolarization B) Disrupting the heart during the critical period of atrial depolarization C) Disrupting the heart during the critical period of ventricular depolarization D) Disrupting the heart during the critical period of ventricular repolarization

D) Disrupting the heart during the critical period of ventricular repolarization

The nurse caring for a client who is suspected of having cardiovascular disease has a stress test ordered. The client has a co-morbidity of multiple sclerosis, so the nurse knows the stress test will be drug-induced. What drug will be used to dilate the coronary arteries? (Chapter 25, Page 699) A) Thallium B) Ativan C) Diazepam D) Dobutamine

D) Dobutamine

The nurse analyzes the electrocardiogram (ECG) strip of a stable patient admitted to the telemetry unit. The client's ECG strip demonstrates PR intervals that measure 0.24 seconds. What is the nurse's most appropriate action? (Chapter 26, Page 741) A) Notify the client's primary care provider of the findings B) Apply oxygen via nasal cannula and obtain a 12-lead ECG C) Instruct the client to bear down as if having a bowel movement D) Document the findings and continue to monitor the patient

D) Document the findings and continue to monitor the patient

The nurse and student nurse are observing a cardioversion procedure. The nurse is correct to tell the student that electrical current will be initiated at which time? (Chapter 26) A) During the contraction phase B) During repolarization of the heart C) During stimulation of the SA node D) During ventricular depolarization

D) During ventricular depolarization

A client is admitted to the hospital with aortic stenosis. What assessment findings would indicate the development of left ventricular failure? (ch 28) A) Orthopnea, nausea, pedal edema B) Dyspnea, distended jugular veins, orthopnea C) Distended jugular veins, pedal edema, nausea D) Dyspnea, orthopnea, pulmonary edema

D) Dyspnea, orthopnea, pulmonary edema

A client is admitted to the hospital with possible acute pericarditis and pericardial effusion. The nurse knows to prepare the client for which diagnostic test to confirm the client's diagnosis? (Chapter 28, Page 814) A) Cardiac cauterization B) CT scan C) Chest x-ray D) Echocardiography

D) Echocardiography

Which statement is accurate regarding Raynaud disease? (Chapter 30, Page 867) A) The disease generally affects the client trilaterally B) It affects more than two digits on each hand or foot C) It is most common in men 16 to 40 years of age D) Episodes may be triggered by unusual sensitivity to cold

D) Episodes may be triggered by unusual sensitivity to cold

A nurse is completing a head to toe assessment on a client diagnosed with right-sided heart failure. To assess peripheral edema, which of the following areas should be examined? (Chapter 25, Page 686) A) Lips and earlobes B) Under the sacrum C) Shoulders and elbows D) Feet and ankles

D) Feet and ankles

The nurse auscultates the apex beat at which anatomical location? (Chapter 25, Page 675) A) Midsternum B) 5 cm to the left of the lower end of the sternum C) 2.5 cm to the left of the xiphoid process D) Fifth intercostal space, midclavicular line

D) Fifth intercostal space, midclavicular line

A nurse is instructing a client about using antiembolism stockings. Antiembolism stockings help prevent deep vein thrombosis (DVT) by: (Chapter 30, Page 872) A) Encouraging ambulation to prevent pooling of blood B) Providing warmth to the extremity C) Elevating the extremity to prevent pooling of blood D) Forcing blood into the deep venous system

D) Forcing blood into the deep venous system

What is a harsh grating sound caused by abrasion of the pericardial surfaces during the cardiac cycle? (Chapter 25, Page 691) A) Murmur B) Opening snap C) Ejection click D) Friction rub

D) Friction rub

The nurse is administering oral metoprolol. Where are the receptor sites mainly located? (Chapter 27, Page 759) A) Uterus B) Blood vessels C) Bronchi D) Heart

D) Heart

The nurse is reviewing the results of the patient's echocardiogram and observes that the ejection fraction is 35%. The nurse anticipates that the patient will receive treatment for what condition? (Chapter 25, Page 678) A) Pulmonary embolism B) Myocardial infarction C) Pericarditis D) Heart failure

D) Heart failure

The nurse reads an athletic client's electrocardiogram. What finding will be consistent with a sinus bradycardia? (Chapter 26, Page 718) A) QR interval of 0.25 seconds B) P-to-QR ratio of 1:2 C) PR interval of 0.24 seconds D) Heart rate of 42 beats per minute (bpm)

D) Heart rate of 42 beats per minute (bpm)

A nurse is caring for a client with end-stage cardiomyopathy and the client's spouse asks the nurse to clarify one of the last treatment options available that the health care provider mentioned. What is considered to be one of the last treatments for end-stage cardiomyopathy? (Chapter 28) A) Annuloplasty B) Xenograft tissue valve C) Valvuloplasty D) Heart transplantation

D) Heart transplantation

Which type of graft is used when a heart valve replacement is made of tissue from an animal heart valve? (Chapter 28, Page 800) A) Allograft B) Autograft C) Homograft D) Heterograft

D) Heterograft

When the postcardiac surgical patient demonstrates vasodilation, hypotension, hyporeflexia, slow gastrointestinal motility (hypoactive bowel sounds), lethargy, and respiratory depression, the nurse suspects which electrolyte imbalance? (Chapter 27, Page 781) A) Hypomagnesemia B) Hypokalemia C) Hyperkalemia D) Hypermagnesemia

D) Hypermagnesemia

The nurse enters the client's room and finds the client pulseless and unresponsive. What would be the treatment of choice for this client? (Chapter 26, Page 738) A) Electric cardioversion B) IV lidocaine C) Chemical cardioversion D) Immediate defibrillation

D) Immediate defibrillation

A client diagnosed with a myocardial infarction (MI) is being moved to the rehabilitation unit for further therapy. Which statement reflects a long-term goal of rehabilitation for the client with an MI? (Chapter 27, Page 766) A) Prevention of another cardiac event B) Ability to return to work and a preillness functional capacity C) Limitation of the effects and progression of atherosclerosis D) Improvement in quality of life

D) Improvement in quality of life

The critical care nurse is caring for clients in an emergency department. When caring for a variety of clients, when is the presence of a third heart sound normal? (Chapter 25, Page 691) A) In clients with heart valve replacement B) In geriatric clients C) In clients with an indwelling pacemaker D) In pediatric clients

D) In pediatric clients

The nurse cares for a client prescribed warfarin orally. The nurse reviews the client's prothrombin time (PT) level to evaluate the effectiveness of the medication. Which laboratory values should the nurse also evaluate? (Chapter 25, Page 694) A) Partial thromboplastic time (PTT) B) Complete blood count (CBC) C) Sodium D) International normalized ratio (INR)

D) International normalized ratio (INR)

The nurse is performing an assessment of the patient's heart. Where would the nurse locate the apical pulse if the heart is in a normal position? (Chapter 25, Page 689) A) Left 2nd intercostal space at the midclavicular line B) Right 2nd intercostal space at the midclavicular line C) Right 3rd intercostal space at the midclavicular line D) Left 5th intercostal space at the midclavicular line

D) Left 5th intercostal space at the midclavicular line

The nurse auscultates the PMI (point of maximal impulse) at which anatomic location? (Chapter 25, Page 675) A) 3 cm to the right of the sternum B) 5 cm to the left of the lower end of the sternum C) 2.5 cm to the left of the xiphoid process D) Left midclavicular line, fifth intercostal space

D) Left midclavicular line, fifth intercostal space

The nurse teaches the client which guidelines regarding lifestyle modifications for hypertension? (Chapter 31) A) Limit aerobic physical activity to 15 minutes, three times per week B) Reduce smoking to no more than four cigarettes per day C) Stop alcohol intake D) Maintain adequate dietary intake of fruits and vegetables

D) Maintain adequate dietary intake of fruits and vegetables

If a client were to develop rheumatic carditis, which cardiac structure would most likely be affected? (Chapter 28, Page 794) A) Coronary arteries B) Inferior vena cava C) Septum D) Mitral valve

D) Mitral valve

The nurse is caring for a client who has a suspected dysrhythmia. What most appropriate intervention should the nurse use to help detect dysrhythmias? (Chapter 26) A) Monitor blood pressure continuously B) Provide supplemental oxygen C) Palpate the client's pulse and observe the client's response D) Monitor cardiac rhythm continuously

D) Monitor cardiac rhythm continuously

Which discharge instruction for self-care should the nurse provide to a client who has undergone a percutaneous transluminal coronary angioplasty (PTCA) procedure? (Chapter 27, Page 773) A) Cleanse the site with disinfectants and dress the wound appropriately B) Normal activities of daily living can be resumed the first day after surgery C) Refrain from sexual activity for 1 month D) Monitor the site for bleeding or hematoma

D) Monitor the site for bleeding or hematoma

Which nursing intervention must a nurse perform when administering prescribed vasopressors to a client with a cardiac dysrhythmia? (Chapter 26, Page 721) A) Keep the client flat for one hour after administration B) Administer every five minutes during cardiac resuscitation C) Document heart rate before and after administration D) Monitor vital signs and cardiac rhythm

D) Monitor vital signs and cardiac rhythm

The nurse is caring for a client who is having chest pain associated with a myocardial infarction (MI). What medication will the nurse administer intravenously to reduce pain and anxiety? (Chapter 28) A) Codeine sulfate B) Fentanyl C) Hydromorphone hydrochloride D) Morphine sulfate

D) Morphine sulfate

A client who was recently diagnosed with prehypertension is to meet with a dietitian and return for a follow-up with the cardiologist in 6 months. What would this client's treatment likely include? (Chapter 31) A) Pharmacological interventions B) Observation only C) Procedural interventions D) Nonpharmacological interventions

D) Nonpharmacological interventions

When the client has increased difficulty breathing when lying flat, the nurse records that the client is demonstrating: (Chapter 28) A) Dyspnea upon exertion B) Hyperpnea C) Paroxysmal nocturnal dyspnea D) Orthopnea

D) Orthopnea

The client returns to the clinic for a follow-up appointment following a permanent pacemaker insertion and reports tenderness and throbbing around the incision. The nurse observes mild swelling, erythema, and warmth at the pacemaker insertion site. What does the nurse suspect? (Chapter 26, Page 746) A) Normal postoperative healing B) Postoperative site hematoma C) Internal bleeding at pacemaker site D) Pacemaker site infection

D) Pacemaker site infection

A client admitted to the telemetry unit has a serum potassium level of 6.6 mEq/L. Which electrocardiographic (ECG) characteristic is commonly associated with this laboratory finding? (Chapter 26) A) Flattened P waves B) Prolonged QT interval C) Occasional U waves D) Peaked T waves

D) Peaked T waves

The nurse is assessing a patient's electrocardiogram (ECG). What phase does the nurse determine is the resting phase before the next depolarization? (Chapter 25, Page 676) A) Phase 1 B) Phase 2 C) Phase 3 D) Phase 4

D) Phase 4

A patient complains of a "stabbing pain and a burning sensation" in his left foot. The nurse notices that the foot is a lighter color than the rest of the skin. The artery that the nurse suspects is occluded would be the: (Chapter 30, Page 858) A) Internal iliac B) Common femoral C) Popliteal D) Posterior tibial

D) Posterior tibial

A client is admitted to the hospital with systolic left-sided heart failure. The nurse knows to look for which assessment finding for this client? (ch29 pg822) A) Jugular venous distention B) Nausea C) Pedal edema D) Pulmonary congestion

D) Pulmonary congestion

A client is being evaluated for coronary artery disease (CAD) and is scheduled for an electron beam computed tomography (EBCT). The nurse understands that the primary advantage of this radiologic test is which of the following? (Chapter 28) A) Clear images B) Less exposure to radiation C) Less invasive procedure D) Quantifies calcified plaque

D) Quantifies calcified plaque

The nurse is caring for a client newly diagnosed with secondary hypertension. Which condition contributes to the development of secondary hypertension? (Chapter 31) A) Hepatic function B) Calcium deficit C) Acid-based imbalance D) Renal disease

D) Renal disease

The nurse completes an assessment of a client admitted with pericarditis. What client symptom will the nurse correlate with the diagnosis of pericarditis? (Chapter 28, Page 814) A) Dyspnea B) Fatigue lasting more than 1 month C) Elevated ESR and CRP D) Reports of constant chest pain

D) Reports of constant chest pain

One of the students asks what the consequences of uncorrected, left-sided heart failure would be. What would be the nursing instructor's best response? (Chapter 25) A) Distention of the jugular vein B) Blood congestion in neck veins C) Effort to lie down to breathe D) Right-sided heart failure

D) Right-sided heart failure

An operating room nurse is caring for a client who is having a pacemaker implanted. The health care provider has requested a demand mode pacemaker for this client. What is this type of pacemaker? (Chapter 26, Page 740) A) A temporary pacemaker B) Asynchronous C) A fixed-rate pacemaker D) Self-activated

D) Self-activated

A nurse is caring for a client with acute mitral regurgitation related to an acute myocardial infarction. The nurse knows to monitor the client carefully for symptoms of which initial complication or result? (Chapter 28, Page 793) A) Infarcted bowel B) Kidney failure C) Cerebral vascular accident (CVA) D) Severe heart failure

D) Severe heart failure

A patient comes to the emergency department with reports of chest pain after using cocaine. The nurse assesses the patient and obtains vital signs with results as follows: blood pressure 140/92, heart rate 128, respiratory rate 26, and an oxygen saturation of 98%. What rhythm on the monitor does the nurse anticipate viewing? (Chapter 26, Page 719) A) Sinus bradycardia B) Ventricular tachycardia C) Normal sinus rhythm D) Sinus tachycardia

D) Sinus tachycardia

A client has a heart rate greater than 155 beats/minute and the ECG shows a regular rhythm with a rate of 162 beats/minute. The client is intermittently alert and reports chest pain. P waves cannot be identified. What condition would the nurse expect the physician to diagnose? (Chapter 26, Page 729) A) Sinus tachycardia B) Atrial flutter C) Heart block D) Supraventricular tachycardia

D) Supraventricular tachycardia

A nurse is teaching a client who will soon be discharged with a prescription for warfarin (Coumadin). Which statement should the nurse include in discharge teaching? (ch30 pg871) a. "Eat more yogurt and broccoli." b. "This drug will dissolve any clots you may still have." c. "If you miss a dose, double the next dose." d. "Don't take aspirin while you're taking warfarin."

"Don't take aspirin while you're taking warfarin."

A 66-year-old client presents to the emergency department reporting severe headache and mild nausea for the past 6 hours. Upon assessment, the client's BP is 210/120 mm Hg. The client has a history of hypertension and takes 1.0 mg clonidine twice daily. Which question is most important for the nurse to ask the client next?

"Have you taken your prescribed clonidine today?"

A client with a forceful, pounding heartbeat is diagnosed with mitral valve prolapse. Which client statement indicates to the nurse a need for additional teaching?(ch28 pg735)

"I can still drink coffee and tea."

The nurse determines that a client recently diagnosed with subacute bacterial endocarditis understands discharge teaching upon which client statement? (ch28 pg801)

"I have to call my doctor so I can get antibiotics before seeing the dentist."

The nurse is educating a client about the care related to a new diagnosis of mitral valve prolapse. What statement made by the client demonstrates understanding of the teaching? (ch 28)

"I will avoid caffeine, alcohol, and smoking."

A nurse is teaching a client who is awaiting a heart transplant. Which statement indicates the client understands what is required to help minimize rejection? (ch28 pg805)

"I will need to take three different types of medications for the rest of my life to help prevent rejection."

A nurse is educating a client about monitoring blood pressure readings at home. What will the nurse be sure to emphasize? (ch30 pg894) a. "Avoid smoking cigarettes for 8 hours prior to taking blood pressure." b. "Sit quietly for 5 minutes prior to taking blood pressure." c. "Sit with legs crossed when taking your blood pressure." d. "Be sure the forearm is well supported above heart level while taking blood pressure."

"Sit quietly for 5 minutes prior to taking blood pressure."

A nurse is caring for a client newly diagnosed with mitral valve prolapse. The health care provider indicates the client has probably had this condition for years. What factor is important for the nurse to consider when teaching the client about valvular disease?

"The client with mitral valve prolapse probably had no health symptoms."

A nurse is caring for four clients on the cardiac unit. Which client has the greatest risk for contracting infective endocarditis? (ch28 pg793)

A client 4 days postoperative after mitral valve replacement

A client in the emergency room is in cardiac arrest and exhibiting pulseless electrical activity (PEA) on the cardiac monitor. What will be the nurse's next action? (ch29 pg838)

Administer epinephrine.

The nurse is administering sublingual nitroglycerin to a client with chest pain. What action will the nurse take after administering two sublingual tablets if the client continues with chest pain and has a blood pressure of 120/82 mm Hg? (ch29, pg821)

Administer the third sublingual nitroglycerin tablet.

The nurse is performing a respiratory assessment for a patient in left-sided heart failure. What does the nurse understand is the best determinant of the patient's ventilation and oxygenation status? (ch29 pg834)

Arterial blood gases

The nurse is teaching a client about mitral stenosis. What are pathophysiologic events occurring in mitral stenosis? Select all that apply. (ch28 pg794)

Amount of time for forward flow is lessened. Pulmonary pressure increases. Diastole is shortened.

A client with left-sided heart failure is in danger of impaired renal perfusion. How would the nurse assess this client for impaired renal perfusion? (ch29 pg822)

Assess for elevated blood urea nitrogen levels.

A client arrives at the ED with an exacerbation of left-sided heart failure and reports shortness of breath. Which is the priority nursing intervention? (ch29 pg822)

Assess oxygen saturation

A nurse is caring for a client receiving warfarin therapy following a mechanical valve replacement. The client had a prothrombin time and International Normalized Ratio (INR) drawn before breakfast. The laboratory report shows the client's INR reading was 4. What is the nurse's first priority ? (ch28 pg801)

Assess the client for bleeding and notify the health care provider of the results.

Which of the following medication classifications is more likely to be expected when the nurse is caring for a client with atrial fibrillation? (Chapter 26, Page 724) A) Diuretic B) Anticoagulant C) Antihypertensive D) Potassium supplement

B) Anticoagulant

A client has been diagnosed with Raynaud's disease. Which self-care strategies minimize risks associated with this disease? Select all that apply. (ch30 pg867) a. Wear gloves to protect hands from injury when performing tasks. b. Do not smoke, or stop smoking. c. Avoid over-the-counter decongestants and cold remedies. d. Refrain from going outdoors in cold weather. Limit activities that place stress on the ulnar nerve

Avoid over-the-counter decongestants and cold remedies. Do not smoke, or stop smoking. Wear gloves to protect hands from injury when performing tasks.

Which of the following is inconsistent as a condition related to metabolic syndrome? (Chapter 27, Page 753) A) Dyslipidemia B) Hypotension C) Abdominal obesity D) Insulin resistance

B) Hypotension

A nurse is teaching a client about maintaining a healthy heart. What information will the nurse include with the teaching? (Chapter 28) A) Exercise one or two times per week B) Use alcohol in moderation C) Smoke in moderation D) Consume a diet high in saturated fats

B) Use alcohol in moderation

A client is diagnosed with mitral regurgitation. What does the nurse consider with the mechanics of cardiac hemodynamics? (ch28 pg793)

Blood flows backward from the left ventricle into the left atrium during systole.

A patient in severe pulmonary edema is being intubated by the respiratory therapist. What priority action by the nurse will assist in the confirmation of tube placement in the proper position in the trachea? (ch 29)

Call for a chest x-ray.

The nurse hears the alarm sound on the telemetry monitor and observes a flat line. The patient is found unresponsive, without a pulse, and no respiratory effort. What is the first action by the nurse? (ch29 pg837)

Call for help and begin chest compressions.

A client is brought to the emergency department via rescue squad with suspicion of cardiogenic pulmonary edema. What complication should the nurse monitor for? Select all that apply. (ch 29)

Cardiac arrest Respiratory arrest Cardiac dysrhythmias

The nurse understands that which of the following medications will be administered to the client for 6 to 12 weeks following prosthetic porcine valve surgery? (Chapter 28, Page 801) A) Furosemide B) Aspirin C) Digoxin D) Warfarin

D) Warfarin

A nurse is assessing a client with congestive heart failure for jugular vein distension (JVD). Which observation is important to report to the physician? (ch29 pg828)

JVD is noted 4 cm above the sternal angle.

A client with mitral stenosis comes to the physician's office for a routine checkup. When listening to the client's heart, the nurse expects to hear which type of murmur? (ch28 pg794)

Diastolic, rumbling, low-pitched

A diabetic client visits a walk-in clinic and asks the nurse to take a blood pressure (BP) reading. The measurements are 150/90 mm Hg. Which of the following would the nurse expect as the treatment to normalize the client's BP?

Drug therapy

A client with aortic regurgitation is admitted to the hospital. Which assessment findings would indicate left ventricular failure? (ch 28)

Dyspnea, orthopnea, paroxysmal nocturnal dyspnea (PND)

The diagnosis of heart failure is usually confirmed by which of the following? (ch29 pg819)

Echocardiogram

Which diagnostic study is usually performed to confirm the diagnosis of heart failure? (ch29 pg819)

Echocardiogram

A client with venous insufficiency asks the nurse what they can do to decrease their risk of complications. What advice should the nurse provide to clients with venous insufficiency? (ch30 pg874) a. Elevate the legs periodically for at least an hour. b. Avoid foods with iodine. c. Elevate the legs periodically for at least 15 to 20 minutes. d. Refrain from sexual activity for a week.

Elevate the legs periodically for at least 15 to 20 minutes.

A client is being seen at the clinic for a routine physical when the nurse notes the client's blood pressure is 150/97. The client is considered to be a healthy, well-nourished young adult. What type of hypertension does this client have?

Essential (primary)

A client has a significant history of congestive heart failure. What should the nurse specifically assess during the client's semiannual cardiology examination? Select all that apply. (ch29 pg823)

Examine the client's neck for distended veins. Monitor the client for signs of lethargy or confusion.

A client with acute pericarditis is exhibiting distended jugular veins, tachycardia, tachypnea, and muffled heart sounds. The nurse recognizes these as symptoms of what occurrence? (ch29 pg836)

Excess pericardial fluid compresses the heart and prevents adequate diastolic filling.

The nurse is auscultating the heart of a client diagnosed with mitral valve prolapse. Which is often the first and only manifestation of mitral valve prolapse? (ch28 pg793)

Extra heart sound

When teaching a client about hypertension and lifestyle changes what does the nurse emphasizes should be included in the diet?

Fresh fruits and vegetables

The nurse is caring for a client diagnosed with infective endocarditis and awaiting blood culture results. The client asks, "Where did I pick up these bacteria?" The nurse is most safe to speculate which of the following? (ch28 pg810)

From a break in the skin

The nurse is caring for a client with suspected right-sided heart failure. What would the nurse know that clients with suspected right-sided heart failure may experience? (ch 29)

Gradual unexplained weight gain

A young mother brings her 4-year-old in to the pediatric clinic with a mild fever and a red, spotty rash that is beginning to fade. The child's heart rate is rapid, and the rhythm is abnormal. The mother states the child has been healthy until about 3 weeks ago when the child had a sore throat. The nurse suspects rheumatic carditis. What organism causes rheumatic carditis? (ch28 pg809)

Group A, beta-hemolytic streptococcus

A client with congestive heart failure is admitted to the hospital after reporting shortness of breath. How should the nurse position the client in order to decrease preload? ch29 pg830)

Head of the bed elevated 45 degrees and lower arms supported by pillows

The nurse is creating a community teaching demonstration focusing on the cause of blood pressure. When completing the visual aid, which body structures represent the mechanism of blood pressure?

Heart and blood vessels

The nurse is preparing to administer hydralazine and isosorbide dinitrate. When obtaining vital signs, the nurse notes that the blood pressure is 90/60. What is the priority action by the nurse? (ch 29)

Hold the medication and call the health care provider.

A client is taking 50 mg of oral spironolactone twice a day to assist with blood pressure control. While the nurse is performing the morning assessment, the client reports nausea, general muscle cramps, and weakness. The ECG strip shows a peaked, narrow T-wave, which is a change. What electrolyte imbalance does the nurse suspect?

Hyperkalemia

A patient arrives at the clinic for a follow-up visit for treatment of hypertension. The nurse obtains a blood pressure reading of 180/110 but finds no evidence of impending or progressive organ damage when performing the assessment on the patient. What situation does the nurse understand this patient is experiencing?

Hypertensive urgency

The nurse is assessing a patient who reports no symptoms of heart failure at rest but is symptomatic with ordinary physical activity. Under what classification does the nurse understand this patient would be categorized? (ch29 pg819)

II

Which New York Heart Association classification of heart failure has a poor prognosis and includes symptoms of cardiac insufficiency at rest? (ch29 pg819)

IV

The nurse is caring for a patient who has started anticoagulant therapy with warfarin (Coumadin). When does the nurse understand that therapeutic benefits will begin? (ch30 pg872) a. Within 12 hours b. Within the first 24 hours c. In 2 days d. In 3 to 5 days

In 3 to 5 days

Which sign or symptom suggests that a client's abdominal aortic aneurysm is extending? (ch30 pg863) a. Increased abdominal and back pain b. Decreased pulse rate and blood pressure c. Retrosternal back pain radiating to the left arm d. Elevated blood pressure and rapid respirations

Increased abdominal and back pain

The pathophysiology of pericardial effusion is associated with all of the following except: (ch29 pg836)

Increased venous return.

A nurse is developing a nursing care plan for a client with peripheral arterial disease. Which of the following will be the priority nursing diagnosis? a. Ineffective peripheral tissue perfusion b. Impaired tissue integrity c. Ineffective thermoregulation d. Ineffective self-health management

Ineffective peripheral tissue perfusion

A client experiences orthostatic hypotension while receiving furosemide to treat hypertension. How will the nurse intervene?

Instruct the client to sit for several minutes before standing.

Which of the following would be inconsistent with a hypertensive urgency?

Intracranial hemorrhage

A patient with a history of valvular disease has just arrived in the PACU after a percutaneous balloon valvuloplasty. Which intervention should the recovery nurse implement? (ch28 pg797)

Keep the patient's affected leg straight.

Aortic dissection may be mistaken for which of the following disease processes? (ch30 pg865) a. Myocardial infarction (MI) b. Stroke c. Pneumothorax d. Angina

Myocardial infarction (MI)

A nurse is teaching clients newly diagnosed with coronary heart disease (CHD) about the disease process and risk factors for heart failure. Which problem can cause left-sided heart failure (HF)? (ch 29)

Myocardial ischemia

A client is admitted to the intensive care unit (ICU) with a diagnosis of hypertension emergency/crisis. The client's blood pressure (BP) is 200/130 mm Hg. The nurse is preparing to administer IV nitroprusside. Upon assessment, which finding requires immediate intervention by the nurse?

Numbness and weakness in the left arm

The nurse is working in a long-term care facility with a group of older adults with cardiac disorders. Why would it be important for the nurse to closely monitor an older adult receiving digitalis preparations for cardiac disorders? (ch29 pg824)

Older adults are at increased risk for toxicity.

A client with infective endocarditis (IE) and a fever is admitted to the intensive care unit. Which of these physician orders should the nurse implement first? (ch28 pg811)

Order blood cultures drawn from two sites

Which describes difficulty breathing when a client is lying flat? (ch29 pg822)

Orthopnea

A client with Raynaud's disease complains of cold and numbness in the fingers. Which of the following would the nurse identify as an early sign of vasoconstriction? (ch30 pg867) a. Cyanosis b. Gangrene c. Pallor d. Clubbing of the fingers

Pallor

A client comes into the emergency department reporting about chest pain that gets worse when taking deep breaths and lying down. After ruling out a myocardial infarction, a nurse would assess for which diagnosis? (ch28 pg815)

Pericarditis

The client with cardiac failure is taught to report which symptom to the health care provider or clinic immediately? (ch29 pg832)

Persistent cough

The nurse in an oncology clinic notes that the client being treated has hypertension. What tumor is a predisposing condition for secondary hypertension?

Pheochromocytoma

Which mitral valve condition generally produces no symptoms? (ch28 pg792)

Prolapse

A postoperative client is receiving heparin after developing thrombophlebitis. The nurse monitors the client carefully for bleeding and other adverse effects of heparin. If the client starts to exhibit signs of excessive bleeding, the nurse should expect to administer an antidote that is specific to heparin. Which agent fits this description? (ch30 pg872) a. Phytonadione (vitamin K) b. Protamine sulfate c. Thrombin d. Plasma protein fraction

Protamine sulfate

A patient is admitted to the intensive care unit (ICU) with left-sided heart failure. What clinical manifestations does the nurse anticipate finding when performing an assessment? (Select all that apply.) (ch29 pg822)

Pulmonary crackles Dyspnea Cough

It is important for a nurse to be aware of the normal hemodynamics of blood flow to recognize and understand pathology when it occurs. The nurse should know that incomplete closure of the tricuspid valve results in a backward flow of blood from the: (ch28 pg792)

Right ventricle to the right atrium.

The nurse is providing discharge teaching for a client with rheumatic endocarditis but no valvular dysfunction. On which nursing diagnosis should the nurse focus her teaching? (ch28 pg809)

Risk for infection

Which term describes high blood pressure from an identified cause, such as renal disease?

Secondary hypertension

The nurse is assessing a client admitted with infective endocarditis. Which manifestation would the nurse expect to find? (ch 28)

Small painful lesions on the pads of the fingers and toes

The nurse is instructing a new graduate nurse. Together, they are caring for a client with infective endocarditis. What is a sign of infective endocarditis? (ch 28)

Splinter hemorrhage

Which is a manifestation of right-sided heart failure? (ch29 pg822)

Systemic venous congestion

The nurse documents pitting edema in the bilateral lower extremities of the client. What does this documentation mean? (ch29 pg823)

There is excess fluid volume in the interstitial space in areas affected by gravity.

A client in a clinic setting has just been diagnosed with hypertension. When the client asks what the end goal is for treatment, what is the nurse's best response?

To prevent complications/death by achieving and maintaining a blood pressure of 140/90 or less

The nurse is caring for a patient with venous insufficiency. For what should the nurse assess the patient's lower extremities? (Chapter 30, Page 874) A) Rubor B) Cellulitis C) Dermatitis D) Ulceration

Ulceration

The physician writes orders for a patient to receive an angiotensin II receptor blocker for treatment of heart failure. What medication does the nurse administer? (ch29 pg825)

Valsartan (Diovan)

The client asked the nurse to describe Stage C heart failure. What is the best explanation by the nurse? (ch29 pg820)

a client who reports no symptoms of heart failure at rest but is symptomatic with increased physical activity

A client is at risk for excess fluid volume. Which nursing intervention ensures the most accurate monitoring of the client's fluid status? (ch29 pg832)

Weighing the client daily at the same time each day

Which nursing intervention should the nurse perform when a client with valvular disorder of the heart has a heart rate less than 60 beats/min before administering beta-blockers? (ch29 pg824)

Withhold the drug and inform the primary health care provider.

A nurse plans to have an education session with a client with cardiomyopathy and the client's spouse about ways to increase activity tolerance. What instructions would the nurse provide? (ch 28) a. Alternate active periods with rest periods. b. Gradually work up to strenuous activity. c. Avoid all physical and emotional stress. d. Include isometric exercises in the daily routine.

a. Alternate active periods with rest periods.

A client with left-sided heart failure reports increasing shortness of breath and is agitated and coughing up pink-tinged, foamy sputum. The nurse should recognize these findings as signs and symptoms of what condition? (ch29, 822)

acute pulmonary edema.

The nurse is to administer morphine sulfate to a client with chest pain. What initial nursing action is required prior to administration? (ch 27 pg 765) a. Measure the blood pressure for hypertension. b. Count the respiratory rate for bradypnea. c. Check the radial pulse for dysrhythmias. d. Measure urinary output for dehydration.

b. Count the respiratory rate for bradypnea.

A nurse reviews a client's medication history before administering a cholinergic blocking agent. Adverse effects of a cholinergic blocking agent may delay absorption of what medication? (ch 27) a. Digoxin b. Nitroglycerin c. Amantadine d. Diphenhydramine

b. Nitroglycerin

The nurse is administering medications to a client with pericarditis. What medications will be commonly prescribed to treat pericarditis? Select all that apply. (ch28 pg814)

colchicine ibuprofen prednisone

The nurse is caring for a client with a blood pressure of 210/100 mm Hg in the emergency room. What is the most appropriate route of administration for antihypertensive agents?

continuous IV infusion

The nurse is assessing a client admitted with cardiogenic shock. What medication will the nurse titrate to improve blood flow to vital organs? (ch29 pg839)

dopamine

The nurse is caring for a client with advanced heart failure. What treatment will be considered after all other therapies have failed? (ch29 pg828)

heart transplant

A blood pressure (BP) of 140/90 mm Hg is considered to be

hypertension.

The nurse is caring for a client with heart failure who is receiving a diuretic medication. What implementation will help the nurse evaluate the client's response of the medication? (ch29 pg823)

measuring intake and output

A nurse reviews the client's medical record and reads in the progress notes that the client has decreased left ventricular function. What assessment will validate the diagnosis? (ch29 pg819)

orthopnea

A client is receiving captopril for heart failure. During the nurse's assessment, what sign indicates that the medication therapy is ineffective? (ch29 pg834)

peripheral edema

A client who was admitted to the hospital with a diagnosis of thrombophlebitis 1 day ago suddenly reports chest pain and shortness of breath and is visibly anxious. The nurse immediately assesses the client for other signs and symptoms of (ch29 pg836)

pulmonary embolism

A nurse suspects that a client has digoxin toxicity. The nurse should assess for: (ch 29)

vision changes.

A client with heart failure must be monitored closely after starting diuretic therapy. What is the best indicator for the nurse to monitor? (ch29 pg834)

weight

The nurse is planning the care of a patient admitted to the hospital with hypertension. What objective will help to meet the needs of this patient?

Lowering and controlling the blood pressure without adverse effects and without undue cost


Related study sets

BA 300 Unit 1: Group and Organizational Influences (Pressures T&N pp. 207, 199-201, 211, 215-218) & (Culture (T&N, pp. 128-129, 132-137, and 139-142)

View Set

google ads video certification questions

View Set

Sociology: Chapter 1: Sociology and the Real World

View Set

PHY 2020 Final - Pradhan (Abridged)

View Set